Dubai Exams Mcqs For GP

You might also like

Download as pdf or txt
Download as pdf or txt
You are on page 1of 1270

PROMETRIC MCQ QUESTIONS FOR UAE

(HAAD,DHA,MOH) QATAR AND SAUDI


ARABIA GENERAL PRACTITIONER EXAMS

ORTHOPAEDICS MCQs

1. Boy running for hours has pain in knee and mass on the upper surface of
the tibia
o Osgood schlatter disease
o Iliotibial band
2. Patient with metartasal fracture, x-ray not show exact fracture, next
investigation:
o USS
o CT
o MRI
3. 20 year old man sustained a deep laceration on the anterior surface of
the wrist. Median nerve injury would result in:
o Claw hand defect
o Wrist drop
o Sensory deficit only
o Inability to oppose the thumb to other fingers
o The ability to flex the metacarpophalangeal joints
4. All the following muscles are part of the rotator cuff, except:
o Supraspinatus
o Infraspinatus
o Deltoid
o Subscapularis
o Teres minor
5. Patient with scoliosis, you need to refer him to the orthopedic when the
degree is:
o 10
o 5
o 15
o 20
6. Patient complaining of pain at night when he elevated his arm, tingling
on lateral arm side and lateral three fingers. What is the diagnosis?
o Brachial plexus neuropathy
o Shoulder impingement syndrome
o Brachial artery thrombophebitis
o Thoracic outlet problem
7. Mid clavicle fracture:
o Surgery is always indicated if the fracture is displaced
o Figure-8-dressing has better outcomes than simple sling
o Figure-8-dressing is strongly indicated in patient with un-union
risk
o Both figure-8 and simple sling has similar outcomes
8. Young adult presented with pain on lateral elbow, tingling of lateral
arm, he plays squash:
o Carpal tunnel
o Tennis elbow
9. Patient complaining of pain along median nerve distribution and
positive tinel sign. Treatment include casting of both hand in what
position
o Dorsiflexion
o Plantar flexion
o Extension
o Adduction
o Abduction
10. Old man with bilateral knee pain and tenderness that increase with
walking and relieved by rest
o RA
o OA
11. The useful exercise for osteoarthritis in old age is to maintain muscle
and bone Low resistance and high repetition weight training:
o Conditioning and low repetition weight training
o Walking and weight exercise
12. Diet supplement for osteoarthritis
o Ginger
13. Old patient c/o bilateral knee pain with mild joint enlargement ESR and
CRP normal dx:
o Osteoarthritis
o Rheumatoid arthritis
o Gout
14. Old lady came to clinic as routine visit, she mention decrease intake of
Ca food, doctor suspects osteoporosis, next initial investigation:
o DEXA
o Ca in serum
o Thyroid function test
o Vit. D
15. Old man c/o knee pain on walking with crepitus x-ray show narrow joint
space and subchondral sclerosis:
o Rheumatoid arthritis
o Gout
o Osteoarthritis
16. Child with back pain that wake Px from sleep. Diagnosis?
o Lumbar kyphosis
o Osteoarthritis
o RA
o Scoliosis
17. 5 year old girl complaining of limping. In CT there vascular necrosis.
Treatment is:
o Surgery total hip replacement
o Splint
o Physiotherapy
18. Adult with osteoporosis. What is the treatment?
o Ca & folic acid
19. Patient with congenital hip dislocation:
o Abducting at flexed hip can cause click or tali
20. Boutonnière deformity of finger is:
o Flexion of the proximal interphalangeal joint & hyperxtension
of the distal interphalangeal joint
o Flexion of the proximal interphalangeal joint & extension of the
distal interphalangeal joint
21. Old patient with painful hip increased with walking & associated with
morning stiffness. Diagnosis:
o Osteoporosis
o Osteoarthritis
o RA
22.Old age with…, & spine x-ray showed ankylosing spondylopathy. What
is the management?
o Injection of subdural steroid
o Back splint
o Physiotherapy
23. Fracture of humerus associated with
o Radial nerve injury
24. Pseudogout is:
o CACO3
o CACL3
25.Old male complaining of right hip pain on walking, the pain increased at
the end of the day. When he wake up in the morning he complaining of
joint pain and stiffness
o Osteoarthritis
o Osteomyelitis
o Osteoporosis
26. The most common fracture in osteoporosis:
o Colles fracture (if prior 75yr)
o Fracture neck of femur
o Shaft of femur
o Hip fracture (if over 75yr)
27. 50 year old male with numbness in the little finger and has degenerative
cervicitis with restriction in neck movement, also there is numbness in
the ring finger and atrophy of the thenar muscle+ compression in the
elbow. What will you do?
o Surgical decompression
o CAT scan for survical spine
28. Which of the following is the disease improving drug for RA:
o NSAID
o Hydroxychloroquine
29. Treatment of open tibial fracture:
o Cephazolin
o Cephazolin + gentamicin
o Gentamicin
o Cephazolin, gentamicin and metronidazole
30. A football player presented with knee pain after a hit on the lateral side
of the knee. On exam, increased laxity on valgus stress, negative
lachman & Mcmurray’s test. What is the most likely diagnosis?
o Lateral collateral ligament tear
o Medial collateral ligament tear
o ACL tear
o PCL
31. Most common site of non-traumatic fracture in osteoporotic patient is:
o Head of femur
o Neck of femur
o Vertebra
o Tibia
32. 2 year old child fell over his toy, as a result of that his leg was under the
toy. In the next day he refused to walk. What is the diagnosis?
o Spiral fracture of the right femur
o Spiral fracture of the right tibia
o Cheeps fracture of the right proximal tibia
o Swelling of the soft tissue of the right leg
o Ankle
33. 50 year old male works as a constructor, 1 week ago when he started
using a hammer; he developed pain on the lateral side of the elbow.
What is your diagnosis?
o Osteoarthritis
o Rheumatoid arthritis
o Ulnar nerve compression
o Lateral epicondylitis
34. Middle age male fell down on his elbow and developed pain which is the
early manifestation (I cannot remember) but: the fat pad sign is a sign
that is sometimes seen on lateral radiographs of the elbow following
trauma. Elevation of the anterior and posterior fat pads of the elbow
joint suggest the presence of occult fracture:
o Anterior pad sign
o Posterior pad sign
35. Child came with or Toeing-in, set in W shape. When he walks both feet
and knee inward with 20 degree. Both femur inward rotations 70
degree. What is the diagnosis?
o Metatarsus adductus
o Femoral anteversion (femoral torsion)
36. Olecranon bursitis of the lbow joint caused by:
o Repeated elbow trauma
o Autoimmune disease
o Staph aureus
o Rupture of bursa
37. Mother complains of sharp pain on radial styloid when carrying her
baby. The pain increase with extension of the thumb against resistance.
Finkelstein test was positive. Diagnosis:
o Osteoarthritis of radial styloid
o De Quervain tenosynovitis
38. 4 year old baby fell down his mother pulled him by the arm & since then
he kept his arm in pronation position. What is your management?
o Sp[lint
o Do xray for the arm before any intervention
o Orthopaedic surgery
39. Polyartheralgia rheumatic. What is the thing that suggests it rather than
ESR & C-reactive protein
o Proximal muscle weakness
o Proximal muscle tenderness
40. 17 year old football player gave history of left knee giving off.
What is the most likely diagnosis?
o Lateral menisceal injury
o Medial menisceal injury
o Lateral collateral ligament
o Medial collateral ligament
o Anterior cruciate ligament
41. 10 year old by presented to clinic with 3 weeks history of limping that
worsen in the morning. This suggest which of the following:
o Septic arthritis
o Legg calve Perthes disease
o RA
o Tumor
o Slipped capital femoral epiphysis
42. 17 year old male while playing football felt on his knee “tern over”
what do you think happened:
o Medial meniscus ligament
o Lateral meniscus ligament
o Lateral collateral ligament
o Anterior cruciate ligament
43. 30 year old male with a history of pain & swelling of the right knee,
synovial fluid aspiration showed yellow color, opaque appearance,
variable varicosity, WBC 150,000, 80%poor mucin clot. Diagnosis is:
o Goutism arthritis
o Meniscal tear
o RA
o Septic arthritis
o Pseudogout arthritis
44. 25 year old male presented with single fracture in the shaft of the
femur. Treatment is:
o Open retrograde intramedullary nail
o Closed antegrade intramedullary nail
o Internal fixation
o Apply cast
o Skeletal traction
45.70 year old man fell on an outstretched hand. Omn examinationintact
radial and ulnar pulses, dinner fork deformity. Tender radial head.
Diagnosis is:
o Fracture of the distal ulna & displacement of radial head
o Fracture of the shaft of radius & displacement of head of ulna
o Colles fracture
o Fracture of Scaphoid
46. The commonest nerve injury associated with humerus fracture is:
o Radial nerve
o Ulnar
o Musculocutaneous
o Axillary
o Median
47. Baby presents with unilateral deformity in the foot appear when it is
become the weight bearing in the other foot but when bearing the
deformity disappear. The patient has defect in dorsiflexion of that foot. I
think they are talking about (club foot) treatment:
o Orthopaedic correction
o Shoe….
o Surgery…
48. Case scenario: patient present with carpal tunnel syndrome.
Treatment:
o Corticosteroid injection
49. Shoulder pain most commonly due to:
o Referred pain due to cardiac ischemia
o Infraspinatus muscle injury
o In acute cholecystitis
o Rotator cuff
50.A patient hairline metartasal fracture. The x-ray is normal. What is the
2nd line
o CT scan
o MRI
o USS
51. Mother come to you complaining that child cannot use his right arm to
take things from her and keeps his arm in pronation postion and fisted.
How will you solve this orthopedic problem:
o Orthopedic referral for possible correction
o Rapid supination of forearm
52.Patient came to you with pain in posterior neck and occipital area. No
affectation of vision, by cervical x-ray , there is decrease in joint space.
What is your diagnosis:
o Cervical spondylosis
53. Cervical spondylosis
o Plantoflexion
o Dorsiflexion
o Extension
o Abduction
54.Patient with congenital hip dislocation
o Abducting at flexed hip can cause click or tali
55. Radiological finding in lateral view elbow dislocation:
o Posterior pad sign
56.33 year old Saudi male complaining from lower back pain and
considerable morning stiffness. X-ray showed sclerosis joint. Other
criterion of this disease are all of the following except:
o Common in male
o Negative RF
o No subcutaneous nodules
o Aortic complications
57. Graph showing risk of osteoporosis with aging
o The elderly people get higher risk than young( something like I
don’t remember)
o 10% of 70 year old people will develop osteoporosis
58.About clavicular fracture in the newborn. What is true?
o Most cases develop brachial plexus injury
o Figure-8-dressing is needed
o Internal fixation is needed
o Most will heal spontaneously
59.18 year old boy with back pain. Investigation to do except:
o CBC
o ESR
o X-ray
o Bone scan
60. Old patient complaining of back pain on walking. On examination
there was stiffness of the muscle and there was some finding on the x-
ray. Best effective treatment:
o Physiotherapy
o NSAID
o Surgery
61. Female, right hand lateral two radial styloid processes pain, since month
increase progressively, CS, positive Finkelstein test. What is the initial
treatment?
o Nerve decompression
o Cast upper joint
o Cast with thumb raised
62. Cast with thumb raised
o -3.5
o -2.5
o 1
o 2
o 3.5
63. In knee examination: +ve Lachman test indicate injury:
o Anterior cruciate ligament
64. Colles fracture:
o Distal end of the radius
o Scaphoid fracture
o Around the elbow
o Head of the radius
65.A child fell on an out-stretched and flexed elbow. Exam showed swelling
around the elbow with no radial pulse. Best management:
o Closed reduction
o Closed reduction then check for radial pulse
o Open reduction
o Cuff and collar for 3 weeks
66. Flexion, adduction, and internal rotation is:
o Anterior hip dislocation
o Posterior hip dislocation
67. Old lady with osteoporosis asked for treatment for prevention:
o Vitamin D
o Vitamin E
o Retinoic acid
68. Young male with morning stiffness at back relieved with activity
and uveitis
o Ankylosing Spondylitis
69. Young female with pain her elbow (lateral epicondylitis). Best
treatment is:
o Rest + physical therapy + NSAID
70. Female presented with complain of neck pain and occipital headache,
no other symptom. On x-ray has cervical spine osteophytes and narrow
disks:
o Cervical spondylosis
71. Bursitis of the elbow joint caused by:
o Elbow trauma
o Autoimmune disease
o Staph aureus
o Rupture of bursa
72. 48 year old male complaining of lower back pain with morning stiffness
for 30 minutes only. On exam he was having spasm centrally on the
lower back. What is the appropriate management:
o Epidural steroids injection
o Back brace
o Facet lysis
o Physiotherapy
73. Old patient had history of gout and drinking alcohol heavily came with
bone pain. On examination, generalized bone tenderness and proximal
muscle weakness, x-ray of long bone shows… I cant remember…ix
shows high ca and ph.. ur dx
o Osteomalacia CA low,ph low, alp high
o Mets from prostate cancer
o Osteoarthritis
o Paget disease ca normal, ph normal, alp high
74. RTA with hip dislocation and shock so causes of shock is:
o Blood loss
o Urethral injury
o Neurogenic
75. Patient with DM presented with limited or decreased range of
movement passive and active of all directions of shoulder
o Frozen shoulder
o Impingement syndrome
o Osteoarthritis
76. Pseudogout is Ca:
o Pyrophosphate
o Sulfate
o Urate
77. An elderly female presented with a history of bilateral hand stiffness
that is worse in the morning. On examination, she had bony swellings in
the distal interphalangeal joints. These swellings are:
o Heberden nodules
o Buchard nodules
o Synovial thickening
o Synovial cysts
78. Snuff box
o In scaphoid bone
79. Female patient has morning stiffness and pain involving the
metacarpophalangeal and proximal interphalangeal joints. What is the
likely diagnosis?
o Rheumatoid arthritis
80. 74 year old female complaining of pain and stiffness in the hip and
shoulder girdle muscles. She is also experiencing low grade fever and
has depression. On examination no muscle weakness
detected(Polymyalgia rheumatic). Investigation of choice:
o RF
o Muscle CK
o ESR
81. Supracondylar fracture patient presented with swelling and cyanosis of
finger after plaster. Management:
o Removal of splint near finger
o Entire removal of all splint
82. The most common site of osteomyelitis is
o Epiphysis
o Diaphysis
o Metaphysis
o Blood flow
83. What is the initial management for a patient newly diagnosed with knee
osteoarthritis:
o Intra-articular corticosteroid
o Reduce weight
o Exercise
o Strengthening of quadriceps muscle
84. Which of the following is true regarding Perthes disease:
o Commonly seen between 11-16years of age
o Always unilateral
o May present by painless limp
o Characteristically affect the external rotation of hip
o More in female
85.A patient is asked to face the wall, bend his waist and let his hands hang
down without support. This test is used as a screening tool for which of
the following?
o Lower limb asymmetry
o Rectal prolapsed
o Scoliosis
86. Years girl, the doctor asked her to flex her waist with free hands.
This screening is for:
o Scoliosis
o Nerve compression
o Disc prolapse
o Sciatica
87. A patient presents with long time history of knee pain suggestive of
osteoarthritis. Now he complains of unilateral lower limb swelling and
on examination there is +ve pedal & tibial pitting edema. What is the
next appropriate investigation?
o CXR
o ECG
o Echocardiography
o Duplex ultrasound of lower limb
88. In lumbar disc prolapsed at L4-L5 the patient will have:
o Pain at groin & front of thigh
o Hypoesthesia around the knee
o Weakness of dorsiflexion of foot
o Absent ankle reflex
o Fasciculation at calf muscle
89. 2 year old baby was brought to the clinic because of inability to
walk straight. On examination, there was asymmetry of skin creases in
the groin. The trendelenburg’s sign was positive on the left side. Your
diagnosi:
o Fracture pelvis
o Congenital hip disclocation
o Fracture femur on the left side
o Poliomyelitis
o Rickets
90. Fractured pelvis commonly associated with:
o Bladder injury
o Penile urethra injury
o Bulbomembraneous urethra injury
o Ureter injury
91. Sickle cell anemia patient presented with unilateral hip pain. Most likely
diagnosis is:
o Septic arthritis
o Avascular necrosis
92. Avascular necrosis of the head of the femur is usually detected
clinically by:
o 3 months
o 6 months
o 11 months
o 15 months
93. Which of the following is not true regarding osteomyelitis
o Pyomyositis
o Epiphyseal plate destruction
o Septic arthritis (it can develop due to osteomyelitis)
o Septicaemia
o After bone growth
94. Congenital hip dislocation; all are true EXCEPT:
o More in girls
o Best examined after 12-36 hours from birth
o There will be limitation in abduction of thigh
o Barlow test will give click indicating CDH
o Can be treated by splint
95.Acute gait disturbance in children; all are true EXCEPT:
o Commonly self limited
o The usual presenting symptom is limping
o Radiological investigation can reveal the diagnosis
o Most often no cause can be found
96. Concerning green stick fracture in children, all are true EXCEPT:
o Extremely painful
o Most commonly involve the forearm
o Function of the limb is preserved
o Is incomplete fracture
97. Which of the following increase bone density and muscle strength
o Endurance and weigh exercise
o High repetition
o Low repetition
98. Hypertensive patient on thiazide presented at night with severe
left foot pain involving the first toe with redness extending to the mid-
leg. The Diagnosis:
o Cellulitis
o Septic arthritis
o Gouty arthritis
99. Man with back pain. X-ray show fracture at T8, L1 & L2. Bone
density T-1,9
o Osteopenia
o Osteoporosis
100. Child fall and had spiral type radial fracture. What is the
management?
o Splinting
o Refer to orthopaedics
o Refer to Pediatrics
o Open reduction with internal fixation
101. Man who has been having severe pain on his big toe with knee
pain and examination revealed negative perferingent crystals:
o Uric acid deposit secondary to synovial fluid over saturation
o Ca pyrophosphate secondary to synovial fluid over saturation
102. Patient with epilepsy came with left shoulder pain. On
examination, flattened contour of the shoulder, fixed adduction with
internal rotation. What is the diagnosis?
o Inferior dislocation
o Subacromial post dislocation
o Subglenoid ant dislocation
o Subclavicular ant dislocation
103. Child with radial head dislocation. What is the next in
management:
o Reduction and subluxation
o X-ray
o MRI
104. Fracture in the humerus affecting radial nerve lead to:
o Wrist drop

OPTHALMOLOGY QUESTIONS

1.Male patient developed corneal ulcer in his right eye after trauma, what is the management ?

a. Topical antibiotics * b. steroids


2. Old diabetic patient with mild early cataract and retinal pigmentation with Drusen formation, you
prescribed antioxidant, what to do next?

a. Urgent ophthalmology appointment b. routine ophalmology referral *


c. cataract surgery d. see him after one month to detect
improvement
3. A picture of snelling chart the ‘q’ was, how far should the patient stand

a. 3m b. 6m* c. 9m

4. Which of the following is not a sign or symptom of central retinal artery occlusion?

a. painful loss of vision* b. painless loss of vision

c. previous transient loss of vision d. dilated pupil with sluggish reaction to light

5. A female patient with right eye pain and redness with watery discharge, no history of trauma,
itching, on examination, there is diffuse congestion in the conjunctiva and watery discharge, what
will you do

a. give Antibiotics b. give antihistamine by exclusion c. topical steroid

d. refer her to ophthalmologist e. no need for further management*

6. Patient complaining of pain when moving the eye, fundoscopy is normal, what is the diagnosis?

a. optic neuritis* b. papillilodema

7. Child with large periorbital hemangioma, if this hemangioma cause obstruction to vision, when
will be permanent decrease in visual acuity after obstruction by one day

a. by 1week* b. by 3 months c. by 6 months


8. Infant born with hemangioma on the right eyelid , what is appropriate time to operate to prevent
amylopia:

a. 1 day b. 1 week* c. 3 months d. 9 months

9. 50 year old man presented to ER with sudden headache, blurred of vision , and eye pain. The
diagnosis is :

a. acute glaucoma* b. acute conjunctivitis c. corneal ulcer

10. Open globe injury, treatment is :

a.continuous antibiotic drops b continuous water and N/Saline drops

c. continuous steroids drops d. sterile cover and then referred*

11. 2 years old boy with coryza, cough and red eyes with watery discharge (a case of measles). Most
likely diagnosis of the red eyes is:

a. conjunctivitis* b. blepharitis

13. SCA patient ,the macula is cherry red and absence of afferent papillary light reflex

a. Retinal artery occlusion

14. Patient has decrease visual acuity bilateral , but more in right side, visual field is not affected, in
fundus there is irregular pigmentations and early cataract formation. What will you do

a. refer to ophthalmologist for laser therapy b. refer to ophthalmologist for cataract surgery

c. see the patient next month*

15. Male patient developed corneal ulcer in his right eye after trauma, what is the management?

a. topical antibiotics and analgesia* b. topical steroids

16. A patient have tender, redness nodule on lacrimal duct site. Before referred him to
ophthalmologist , what will you do:

a. Topical steroid b. Topical antibiotics c. Oral antibiotics *

17. A male came to you complaining of sudden progressive decreasing in vision of left eye over last
two/three days, also pain on the same eye , on fundoscopy optic disk swelling was seen. Diagnosis

a. central retical vein occlusion b. optic neuritis* c. macular degeneration

18. A gardener has recurrent conjunctivitis. He cant avoid exposure to environment. In orde to
decrease the symptoms in the evening, GP should advise him to:

a. cold compression b. eye irrigation with vinegar c. contact lenses d. antihistamines*

19. Patient medically free came with eye watery discharge , cloudy anterior chamber with red
conjunctiva. Diagnosis
a. Keratitis b. Uveitis* c. Retinitis d. corneal ulceration

20.30 year old patient presented with eye stocking in the morning , what the cause?

a. viral b. bacterial* c. fungal

21. Initial treatment of acute angle glaucoma:

a. IV acetazaolamide, topical pilocarpin and B blocker

22. Patient with lateral and vertical diplobia, he cant abduct both eyes, the affected nerve is:

a. II b. III c. VI* d. V

23. Photophobia, blurred vision, keratic behind cornea and cells in anterior chamber , the best
treatment is :

a. topical antifungal b. topical acyclovir c. antiobiotic*

24. Patient with trachoma in eye for prevention you should

a. water sanitation* b. water sanitation and eradication of organism

c. mass treatment

25. Patients come with history of flu like symptoms for many days and complain of periorbital
edema, Diagnosis

a. viral conjunctivitis* b. bacterial conjunctivitis c. keratitis

26. Pterygium in ophthalmology , treatment

a. surgery

27. Patient with ptosis , which nerve is affected?

a. 3rd CN (occulomotor)

28. Patient comes with sudden painless loss of vision before going to lose the vision see flashes and
high lights, asking for diagnosis:

a. Retinal detachment

29. Patient with URTI when he coughs or sneez see lash asking the possible causes:

a. Mechanical stimuli to retina, irritation of optic disc

30. hazy vision with subcortical of keratinizing deposition asking for management

a. systemic steroids

31. Patient with pain in ophthalmic division of trigeminal nerve and vesicle, which of the following is
used to decrease post herpetic neuralgia:
a. local steroids b. systemic acyclovir and steroids* c. acyclovir

32. Male patient developed corneal ulcer inhis right eye after trauma , what is the management:

a. topical antibiotics and analgesia b. topical steroids c. antibiotic, cycloplegia and refer to
ophthalmology*

33. Blow out fracture eyelid swelling, redness other symptoms include

a. present of air fluid level b. enophthalmos (posterior displacement of the eye)*

34. Attack rate for children whom developed pink eye, frist day 10 out of 50, second day 30 out
of 50

a. 20 b. 40 c. 60 d. 80*

35. Patient came with trauma to left eye by tennis ball examination shows anterior chamber
hemorrhage , you must exclude?

a. conjunctivitis b. blepharitis c. foreign body (most likely)* d. keratitis

36. Acute angle glaucoma, you can use all of the following drug except?

a. B blocker b. acetazolamide c. pilocarpine d. dipivefrin*

37. Patient with foreign body sensation in the eye, after the removal of the foreign body it was
insect, treatment

a. local antibiotics* b. local steroids c. systemic antibiotic d. systemic steroid

38. Mucopurulent discharge

a. bacterial conjunctivitis

39. Patient with hypertensive retinopathy grade2 AV nicking, normal BP , no decrease vision,
with cupping of optic disc, what will you do to the patient:

a. reassurance, the problem is benign b. convert him to ophthalmologist* c. laser operation

40. A 30 year old male present to ER complaining of visual deterioration for 3 days of RT. Eye
followed by light perception, the least cause is:

a. retinal detachment b. central retinal arterial embolism * c. vitreaous humor hemorrhage

d. retro-orbital neuritis e. retinitis pigmentation

41. Anterior uveitis is a character of the following except

a. RA* b. sarcoidosis c. behcet disease d. riters syndrome e. ankylosing spondolitis

42. Patient with open angle glaucoma and known case of COPD and DM , what is the treatment

a. Timelol b. betaxolol c. acetazolamide*


43. Patient with bilateral eye discharge, watery,red eyes, corneal ulceration, what is the most
common cause?

a. dust and pollen* b. hypertension c. ultra-violent light and stress

44. 70 y/o female says play puzzle but for a short period she cant play because as she develop
headache when playing what will you exam?

a. astigmatism* b. glaucoma

45. 54 y/o patient, farmer, complains of dry eye, he is a smoker for 20yrs and smokes 2 packs/
day, your recommendation:

a. advise him to exercise b. stop smoking * c. wear sunscreen

47. Patient is wearing contact lenses for vision correction since ten years , now come
complaining of excessive tearing when exposed to bright light, what will be your advice to him

a. wear hat b. wear sunglasses c. remove the lenses at night* d. saline eyedrops 4
times/day

48. Patient complains dry eyes, a moisturizing eye drops were prescribed to him 4 times daily.
What is the most appropriate method of application of these eye drops?

a. 1 drop in the lower fornix* b. 2 drops in the lower fornix c. 1 drop in the upper fornix

49. 17 yrs old school boy was playing football and he was kicked in his right eye…few hours later
he started to complain of double vision and ecchymoses around the eye, what is the most likely
diagnosis?

a. cellulitis b. orbital bone fracture* c. global eye ball rupture d. subconjunctival


hemorrhage

50. Diabetic patient have neovascularization and vitreous hemorrhage, next step

a. refer to ophthalmologist

51. 35 yrs old female complaining of acute inflammation and pain in her left eye since 2days ,
she gave history of visual blurring and use of contact lens as well, on examination, flouorescence
stain shows dentritic ulcer at the center of the cornea, what is the most likely diagnosis?

a. corneal abrasion b. herpetic central ulcer* c. central lens stress ulcer d. acute episcleritis
e. acute angle closure glaucoma

52. Patient present with corneal abrasion, treatment

a. cover the eye with a dressing b. antibiotics ointment put it in the home without covering
the eye*

53. Patient with subconjunctival hemorrhage. What you will do for him ?

a. reassurance b. send him to the ophthalmologist*


54. Patient with recent history of URTI develop severe conjunctivitis injection with redness, tearing,
photophobia, what is treatment?

a. topical antibiotics b. topical acyclovir c. oral acyclovir d. topical steroids*

55. Patient presented with constricted pupil, ciliary flushing and cloudy anterior chamber, there is no
abnormality in eyelid , vision, and lacrimal duct:

a. uveitis* b. central vein thrombosis c. central artery embolism d. acute angle closure
glaucoma

56. Newborn with eye infection:

a. oral anti*biotics b. oral steroids c. topical antibiotic

57. Man who brought a cat and now developed watery discharge from his eyes he is having:

a. allergic conjunctivitis* b. atopic dermatitis c. cat scratch disease

58. How to differentiate between uveitis and keratitis in red eye

a. redness of the eye b. blurred vision c. photophobia d. eye pain e. dark, floating spots along the
visual field in uveitis, ciliary vessel dilatation*

59. By covering test done to child the other eye turn laterally, diagnosis is

a. exotropia strabismus

60. Hypertensive came to ophthalmology doctor by exam show increase cup when asking the patient
he did not complain of anything. What is the diagnosis?

a. hypertensive retinopathy b. diabetic neuropathy* c. acute closed angle glaucoma d. retinal


detachment

61. Long use of tropical corticosteroid lead to :

a. rise intraocular pressure b. cataract* c. ptosis d. keratoconus

62. Female patient wear glass since 10yrs , she diagnosed recently type 2 DM , she should screen or
examine her eyes every :

a. 6M b. 12 M* c. 2Yrs d. 5Yrs

63. Picture (fundus of eye) ‘glaucoma’

a. increase cup to disc ratio more than A1/2

64. Diabetic patient want your advice to decrease the risk of developing diabetic retinopathy?

a. decrease HTN and obesity* b. decrease HTN and smoking c. decrease smoking and obesity

65. Patient came to you after trauma complaining of loss of the abduction of his (left or right) eye. So
which cranial nerve affected?
a. III b. IV c. V d. VI*

66. A boy 3days after flu symptom develop conjunctivitis with occipital and nick L.N enlarged , so
diagnosis is

a. adenoviruses* b. streptococcus c. HSV

67. Child came to ophthalmology clinic did cover test, during eye cover, his left eye move
spontaneously to left, the most complication is:

a. strabismus* b. glaucoma c. myobloma

68. A 45yr old male presented to the ER with sudden headache , blurring of vision, excruciating eye
pain and frequent vomiting:

a. acute glaucoma* b. acute conjunctivitis c. acuteiritis d. episcleritis e. corneal ulceration

69. Patient came to you complaining of gradual loss of vision and now he can only identify light.
Which of the following is the LEAST cause of his problem:

a. retinal detachment b. central retinal artery* c. retinitis pigmentosa d. retrobulbar neuritis

70. What is the management of uveitis?

a. topical or oral steroid

71. All the following may cause sudden unilateral blindness except:

a. retinitis pigmentosa* b. retrobulbar neuritis c. retinal detachment d. vitreous hemorrhage


e. central retinal artery embolism

72. Patient has painful red left eye associated with photophobia, what is the diagnosis?

a. glaucoma b. uveitis* c. others

73. Retinal detachment, all true except :

a. more common in hypermetropic patient than myopic

74. Acute glaucoma, all are true except:

a. refer to ophthalmologist b. give miotic before referral * c. can present with headache d. can
present with abdominal pain e. pupil size in acute glaucoma is larger than normal

75. All are true about congenital squint except:

a. there is no difference of the angle of deviation of squint eye between far and near vision*

b. asymmetry of corneal light reflex

76. TB patient suffer from painful red eye photophobia

a. glaucoma b. uveitis* c. bacterial conjunctivitis d. viral conjunctivitis


77. Regarding stye infection of the lower eyelid , all true except:

a. is infection of gland in the lower eye lid b. can be treated by topical antibiotics

c. can be treated by systemic antibiotics, needs ophthalmology referral though

d. sometimes referral is needed , but it is never the first option*

78. Which of the following is true regarding red eye:

a. more redness occur in corioscleral ‘suggest iritis’*

b. if associated with fixed mid fixed dilated pupil suggest anterior uveitis

c. in case of glaucoma treatment is mydratics

79. Picture of snelling chart, 70yrs old patient can only read to the 3rd line , what is his visual acuity?

a. 20/100 b. 20/70* c.20/50 d.20/40

80. This patient see letters at 20 feet, where normal person see it:

a. at 70 feet

81. 24yrs old female newly diagnosed type 2 DM , she is wearing glasses for 10yrs, how frequent she
should follow ophthalmologist?

a. every 5yrs b. annually*

82. Contraindicated in acute glaucoma management:

a. pilocarpine b. timolol

83. Flu like symptoms since two days and now has red eye, what is the diagnosis:

a. viral conjunctivitis* b. bacterial conjuctivis c. uveitis d. glaucoma

84. The most dangerous red eye that need urgent referral to ophthalmologist

a. associated with itching b. presence of mucopurulent discharge c. bilateral e. associated with


photophobia*

85. Patient with pterygium in one eye, the other eye is normal, what’s correct to tell:

a. it’s due to vitaminosis A b. it may affect vision c. it’s part of a systemic disease

RHEUMATOLOGY

1. An elderly lady presented with chronic knee pain bilaterally that increases with
activity and decreases with rest. The most likely diagnosis is:
a. Osteoarthritis (correct answer)
b. Rheumatoid arthritis

c. Septic arthritis

2. An old woman complaining of hip pain that increases by walking and peaks by the
end of the day and keeps her awake at night, also morning stiffness:

a. Osteoporosis

b. Osteoarthritis (correct answer)

c. Rh. Arthritis

3. Old patient with bilateral knee swelling, pain, normal ESR:

a. Gout

b. Osteoarthritis (correct answer)

c. RA

4. What is the initial management for a middle aged patient with newly diagnosed knee
osteoarthritis?

a. Intra-articular corticosteroid

b. Reduce weight (correct answer)

c. Exercise

d. Strengthening of quadriceps muscle

5. The useful exercise for osteoarthritis in old age to maintain muscle and bone:

a. Low resistance and high repetition weight training (correct answer)

b. Conditioning and low repetition weight training

c. Conditioning and low repetition weight training

6. Male patient presents with swollen erythema , tender left knee and right wrist.
Patient gives history of international travel 2 months before. Aspiration of joint reveals
gram negative diplococci. What is the most likely diagnosis?

a. Neisseria gonorrhea (correct answer)

b. Staphylococcus

c. Streptococcus

7. Triad of heart block, uveitis and sacroiliitis , diagnosis;

a. Ankylosing spondylitis (Correct answer)


b. Lumbar stenosis

c. Multiple myeloma

8. Patient had urethritis but now comes with left knee pain. Urethral swab reveals
positive pus cell but is negative for Neisseria meningitidis and Chlamydia.

a. RA

b. Reiter’s disease (answer)

c. Gonococcal

9. Patient with rheumatoid arthritis . X-ray of fingers show permanent lesion that may
lead to permanent dysfunction. What is the underlying process?

A. Substance secreted by synovial fluid

10. Which of the following favor diagnosis of SLE?

a. Joint deformity

b. Lung cavitations

c. Sever Raynaud phenomenon

d. Cystoid body in retina

e. Anti RNP+ (answer)

11. Patient with rheumatoid arthritis. Hand xray shows swelling. What will you do for
him?

a. NSAID (answer).

b. Injection Steroid

c. Positive pressure ventilation.

12. Which of these is true of dermatomyositis?

a. associated with inflammatory bowel disease

b. indicates underlying malignancy (answer)

c. presents as distal muscle weakness

13. Pseudogout;

a. Phosphate

b. Calcium

c. Flouride
d. Calcium pyrophosphate (answer)

14. Patient complains of abdominal pain and joint pains. The abdominal pain is colicky in
character, and accompanied by nausea, vomiting and diarrhea. There is blood and
mucus in the stools. The pain in the joints involves the ankle and the knees . On
examination there is purpura on the legs and buttocks:

a. Meningococcal infections

b. Rocky Mountain Spotted Fever

c. Systemic Lupus Erythematosus

d. Henoch Sconlein Purpura (answer)

15. Long scenario, bone mineral density , having T score – 3.5, so diagnosis is:

a. Osteopenia

b. Osteoporosis ( answer)

c. Normal

d. Rickets disease

16. Patient with hypertension and on medication for that. Now complains of pain and
swelling of big toe(MT). In the light of recent complaints, which of the following drugs
must be changed?

a. Thiazide No answer given

17. Elderly came with sudden loss of vision in the right eye with headache. Investigation
showed high CRP and high ESR. What is the diagnosis?

a. Temporal arteritis No answer given

18. Old female with osteoporosis. What is the exogenous cause?

a. Age

b. Decreased vitamin D (answer)

19. Patient with cervical spondylitis came with atrophy in hypothenar muscle and
decreased sensation in ulnar nerve distribution. Studies showed alertness in ulnar
nerve function in elbow. Your action is

a. Physiotherapy

b. Cubital tunnel decompression (?carpal tunnel) (answer)


20. Patient is a known case of cervical spondylosis, presented with paraesthesias of the
little finger with atrophy of the hypothenar muscles. EMG showed ulnar (?carpal) tunnel
compression of the ulnar nerve. What will be your action now?

a. Steroid injection

b. CT scan of spine

c. Ulnar nerve decompression. (answer)

21 Polymalgia Rheumatica case with elevated ESR. Other feature:

a.Proximal muscle weakness

b. proximal muscle tenderness (answer)

22. Patient came with osteoarthritis and swelling in the distal interphalangeal joint.
What is the name of this swelling?

a. Bouchard nodes

b. Heberden’s nodes (answer)

23. An 80 year old lady presented to your office with a 6 month history of stiffness in her
hand, bilaterally. This stiffness gets worse in the morning and quickly subsides as the
patient begins daily activities. She has no other significant medical problems. On
examination, the patient has bilateral bony swellings at the margins of the distal
interphalangeal joints on the 2nd – 5th digits. No other abnormalities were found on
physical examination. The swellings represent:

a. Heberden’s nodes ( answer)

b. Bouchard’s nodes

c. Synovial thickenings

d. subcutaneous nodules

e. Sesamoids

24. Regarding Boutonniere deformity, which one is true?

a. Flexion of PIP and hyperextension of DIP (answer)

b. Flexion of both PIP and DIP

c. Extension of PIP and flexion of DIP

d. Extension of both PIP and DIP


25.Patient has history of parotid and salivary gland enlargement, complains of dry eye,
mouth and skin. Lab results HLA-B8and DR3 ANA positive, Rheumatoid factor positive.
What is the course of treatment?

a. Physostigmine

b. Eye drops with saliva replacement (answer)

c. NSAID

d. Plenty of oral fluid

26. Young patient with red, tender swollen big left toe 1st metatarsal tender , tender
swollen foot and tender whole left leg. His temperature is 38. What is the diagnosis?

a. Cellulitis ( answer)

b. Vasculitis

c Vasculitis

27. Elderly patient with unilateral headache, chronic shoulder and limb pain , positive
Rheumatoid factor and positive ANA . What is the treatment?

a. Aspirin

b. Indomethacin

c. Corticosteroid ( answer)

28. Active gout management :

a. Allopurinol

b. NSAID ( answer)

c. Paracetamol

d. Gold salt

29. Treatment of acute gouty arthritis

a. Allopurinol

b. Indomethacin (answer)

c. Penicillamine

d. Steroid

30. Best investigation for giant cell arteritis.

a. Biopsy of temporal arteries (No answer given)


31. Patient with Rheumatoid arthritis came to you and asking about the most effective
way to decrease joint disability in the future. Your advice will be:

a. Cold application over joint will reduce the morning stiffness symptoms

b. Disease modifying antirhumatic drugs are sufficient alone. ( answer)

32. Osteoporosis depends on

a. Age (answer)

b. Stage

c. Gender

33. 30 year old male with history of pain and swelling of the right knee. Synovial fluid
aspiration showed yellow colour opaque appearance, variable viscosity. WBC= 150,000,
80% neutrophil, poor mucin clot. Diagnosis is ;

a. Gouty arthritis

b. Meniscal tear

c. RA

d. Septic arthritis

e. Pseudogout arthritis (answer)

34. Rheumatoid Arthritis

a. Destruction in articular cartilage (answer)

b. M = F

c. No nodules

d. Any synovial joint

e. HLA DR4

35. Triad of heart block, tis and sacroileitis. Diagnosis;

a. Ankylosing spondylitis (answer)

b. Lumbar stenosis

c. Multiple myeloma

36. Pseudogout is

a. CaCO3 (answer)

b. CaCL3
37. Juvenile Idiopathic arthritis treatment :

a. Aspirin (answer)

b. Steroid

c. Penicillamine

d. Hydrochloroquin

e. Paracetamol

38. Patient presents with SLE . The drug with least side effect;

a. Methotrexate (answer)

b. name of other chemotherapy

39. Regarding allopurinol :

a. It is a uricosuric agent

b. Decreases the development of uric acid stones (answer)

c. Useful in acute attack of gout

40. Man with pain and swelling of first metatarsophalangeal joint . Diagnosis:

a. Gout also called podagra

No answer given

41. 14 year old girl with athralgia and photosensitivity and malar flush and proteinuria.
Diagnosis;

a. RA

b. Lupus Nephritis (answer)

c. UTI

42. Which of the following is a disease removing drug for RA?

a. NSAID

b. Hydroxychloroquine (answer)

43. 27 year old male has symmetric oligoarthritis, involving knee and elbow, painful oral
ulcer for 10 years, came with a form of arthritis and abdominal pain. Diagnosis is:

a. Behcets disease (answer)

b. SLE
c. Reactive arthritis

d. UC

e. Whipple’s disease

44. Child with positive Gower sign. Which is most diagnostic test

a. Muscle biopsy

45. Patient is a 74 year old female complaining of pain and stiffness in the hip and
shoulder girdle muscles. She is also experiencing low grade fever and has depression. On
examination, no muscle weakness detected. Investigation of choice is:

a. RF

b. Muscle CK

c. ESR (answer)

46. Female patient diagnosed as polymyalgia Rheumatica. What will find in the clinical
picture to support this diagnosis?

a. Osteophyte in joint radiograph

b. Tenderness of proximal muscle (answer)

c.Weakness of proximal muscle

d. Very high ESR

47. Dermatomyositis comes with the following symptoms

a. Proximal muscle weakness ( answer)

b. Proximal muscle tenderness

48. Most important to predict a prognosis of SLE patient:

a. Degree of renal involvement (answer)

b. Sex of the patient

c. Leucocyte count

49. Patient presented with back pain relieved by ambulation . What is the best initial
treatment?

a. Steroid injection in the back

b. Back bracing

c. Physical therapy as initial treatment (answer)


50. Diet supplement for osteoarthritis

a. Ginger

No answer given

51. Which drug causes SLE like syndrome?

a. Hydralazine (answer)

b. Propranolol

c. Amoxicillin

52. In patients with rheumatoid arthritis,

a. Cold app overjoint is good

b. Bed rest is the best

c. Exercise will decrease post-inflammatory contractures (answer)

53. Gouty arthritis negative birefringent crystals. What is the mechanism?

a. Deposition of uric acid crystal in synovial fluid due to oversaturation.

No answer given

54. Old patient with history of bilateral pain and crepitations of both knee for years.
Now comes with acute right knee swelling . On examination, you found that there is
edema over dorsum and tibia of right leg . What is the best investigation for
thcondition?

a. Right limb venogram

No answer given

55. 40 year old male comes to you complaining of sudden joint swelling. No history of
trauma. No historyof chronic disease. What is the investigation you will ask?

a. CBC for WBCs

b. ESR (answer)

c. MRI of knee joint

d. Rheumatoid factor

56. Female with sudden blindness of right eye , no pain in the eye. There is temporal
rness when combing hair. What is the management?

a. Steroid eye drop


b. Oral steroid

c. IV steroids (answer)

57. Patient with oral ulcer, genital ulcer and arthritis. What is the diagnosis?

a. Behcet’s disease ( answer)

b. Syphilis

c. Herpes simplex

58. Patient with history of 5 years HTN on thiazide came to ER midnight screaming
holding his left foot . On examination, patient is afebrile with left foot tender erythema.
Swollen big toe is most tender and painful. No other joint involvement.

a. Cellulitis

b. Gouty arthritis (answer)

c. Septic arthritis

59. Joint aspirate. Gram stain reveals gram negative diplococcic(N gonorrhea). What is
the treatment?

a. Ceftriaxone IM or cefepime PO one dose

No answer given

60. Commonest organisms in septic arthritis:

a. Staphylococcus aureus (answer)

b. Streptococci

c. Streptococci

61. Child with back pain that es from sleep. So diagnosis (incomplete Q) :

a. Lumbar kyphosis (answer)

b. Osteoarthritis

c. Scoliosis

62. Patient with pain in sacroiliac joint, with morning stiffness. Xray of sacroiliac joint, all
will be found except :

a. RF negative

b. Subcutaneous nodules ( answer)

c. Male> Female
63. Allopurinol, one is true:

a. Effective in acute attack of gout

b. Decreases the chance of uric acid stone formation in the kidneys (answer)

c. Salicylates antagonize its action

64. Mechanism of destruction of joint in RA

a. Swelling of synovial fluid

b. Anti-inflammatory cytokines attacking the joint (answer)

65. 28 year old woman came to your clinic with 2 month history of flitting arthralgia.
Past medical history unremarkable. On examination, she is afebrile.Right knee joint :
mild swelling with some tenderness, otherwise no other physical findings. CBC: HB
124g/L

(12.4 g/dl, WBC 9.2* 109/l ESR 80 MM/H . Rheumatoid factor negative. VDRL positive.
Urine : RBC: 15-20/ hpf. Protein 2+.The most appropriate investigation at this time is:

a. Blood culture

b. A.S.O titre.

c. C-reactive proin

d. Double stranded DNA

e. Kidney Ultrasound

PULMONOLOGY

1. Young patients with history of cough, chest pain, and fever. CXR showed lower lobe infiltrate

A. amoxycillin B. cefuroxime C. Imipenem. D. Cipro

2. Best thing to reduce motarlity rate in COPD. A. home oxygen. B. Enalapril C STOP SMOKING

3. Patient with TB has ocular toxicity. The drug responsible is. A. ETHAMBUTOL B. Streptomycin
C. Isoniazid D. Rifampin
4. Patient treated for TB started then develop numbness. Which of the vitamin is deficient. A.
PYRIDOXINE. B . Niacin C. Vitamin C. D. Thiamin

5. A 17 year old patient with dyspnea. The PO2, PCo2, PH are increased, CXR normal. A ACUTE
ASMATHIC ATTACK. B. pulmonary embolism. C. pneumonia. D. pneumothorax

6. The most common cause of community acquired pneumonia is A. STREPT PNEUMONIA B. H.


influenza. C. klebsiella. D. mycoplasma

7. Patient presented with sorethroat, anorexia, loss of appetite. Throat exam showed enlarged
tonsils with patachie on palate and uvula, mild tenderness of spleen and liver. The causative org is.
A. EBV(INFECTIOUS MONONUCLEOSIS). B. group B streptococcus

8. Most common cause of croup is A. PARAINFLUENZA. B. influenza

9. Young patient on TB medication is having vertigo. Which of the following drug is the cause. A.
STREPTOMYCIN. B. ethambutol. C. rifampicin

10. Well known case of SCD presented with pleuritic chest pain, fever, tachypnea,
respitaotry rate was 30bpm, Oxygen saturation was 90%. What is the diagnosis? A. ACUTE CHEST
SYNDROME. B. voc. C. pericarditis

11. Child with atopic dermatitis has stridor at night with barking cough on and off from
time to time. Diagnosis is A. croup. B. ba. C. SPASMODIC CROUP

12. Patient with asthma well controlled by albuterol came complaining of asthma not
responding to albuterol. What medication would be added? A. CORTICOSTEROID INHALER. B. long
acting beta agonist C. oral corticosteroid. D. theophylline

13. An old patient with history of CVD and IHD presents with a pattern of breathing
described as ‘ a period of apnea followed by slow breathing which accelerates and becomes rapid
with hyperpnea and tachycardia and then apnea again. What is this type of breathing? A.
Hippocrates. B. CHYENE STOKE. C. kussmaul breathing. D. one type beginning with O and contains
only three letters

14. Rheumatic fever patient has streptotoccocal pharyngitis. Risk of developing another
is A. Trimes more than normal. B. 50%. C. 100%

15. Young male patient had pharyngitis then cough and fever. What is the most likely
organism? A. staph aureus. B. STREPT PNEUMO

16. 17 year old male with history of mild intermittent asthma attacks occur once or
twice weekly in the morning and no attacks in the night. What should be the initial drug to give? A.
INHALED SHORT ACTING B2 AGNONIST AS NEEDED. B. inhaled high dose corticosteroid. C. oral
steroid. D. ipratropium bromide.

17. Case scenarios about bronchial carcinoma. Which is true? A. the most common
cancer in females. B. spuamous cell carcinoma spreads faster. C. ADENOCARCINOMA IS USUALLY IN
THE UPPER PART. D. elevation of the diaphragm on the Xray means that the carcinoma has
metastasize outside the chest. D. bronchoscopy should be done

18. 39 year old HIV patient with TB receive 4 drugs of treatment after one month. A.
continue 4 drugs for 1 year. B. CONTINUE ISONIAZID FOR 9 MONTHS. C. continue isoniazid for 1year

19. Child has history URTI for few days. He developed barky cough and SOB. Your
diagnosis is A. foreign body inhalation. B. pneumonia. C. CROUP. D pertussis

20. Asthma case. What drug is prophylactic? A. B2 AGONIST. B. theophylline. C. oral


steroid

21. Male patient working in the cotton field presented with three weeks history of
cough. CXR showed bilateral hillar lymphadenopathy and biopsy (by bronchoscopy) showed non
caseating granuloma. What is your diagnois? A. sarcoidosis. Bamyloidosis. C. histiomycosis. D.
berryllosis. E. PNEUMOCONIOSIS
22. Patient with untreated bronchogenic carcinoma has dilated neck vein, facial
flushing, hoarseness and dysphagia(SVC syndrome. CXR showed small pleural effusion. What is your
immediate action? A. consult cardiologist for pericardiocentesis. B. consult thoracic surgeon for
thoracoentesis. C. CONSULT ONCOLOGIST

23. Patient with typical finding of pleural effusion management. A. CHEST TUBE

24. Old patient with DM2, EMPHYSEMA, non community pneumonia. Best to give is A.
pneumococcal vaccine and influenza vaccine now. B. pneumococcal vaccine and influenza vaccine 2
weeks after discharge. C. PNEUMOCCOCAL VACCINE AND INFLUENZA VACCINE 4 WEEKS AFTER
DISCHARGE. D. influenza vaccine only. E. pneumococcal vaccine only

25. Radiological features of military TB; A pleural effusion. B. 3-4 DIFFUSE NODULES. C.
small cavities

26. Patient ingested large amount of aspirin show nausea, vomitting and
hyperventilation. What is the diagnosis? A. metabolic alkalosis and respiratory alkalosis. B. metabolic
acidosis and respiratory acidosis. C. RESPIRATORY ALKALOSIS AND METABOLC ACIDOSIS. D.
respiratoty alkalosis and respiratory acidosis

27. Patient presented with sudden chest pain and dyspnea, tactile fremitus and chest
movement is decreased. Xray showed decreased pulmonary marking in let side. What is the
diagnosis? A. atelectasis of left lung. B. SPONTANEOUS PNEUMOTHORAX. C. pulmonary embolism

28. A 20 year old male who is a known asthmatic presented to the ER with shortness of
breath. PR 120 RR 30, PEFR 100mins. Examnation revealed very quiet chest. What is the most
probable mgt? A. NEBULISED SBUTAMOL. B. I aminophylline. C. pleural aspiration. D. hemlich
maneuver. E. chest drain.

29. Patient is a known case of moderate intermmitent bronchial asthma. He is using


ventollin nebulizer. He develops 3 attacks per week. The drug to be added is A. increase dose of
prednisolone. B. ADD LONG ACTING B2 AGONIST. C. add ipratropium C. Iv aminophylline
30. One of the following is true about COPD home mgt. A. GIVE O2 IF THE SO2 IS LESS
THAN 88%. B. give o2 if the so2 is 88-95%. C. give o2 at night only

31. Elderly male patient who is a known case of debilitating disease presented with
fever, productive of cough, and sputum culture showed growth of gram negative organisms on a
buffered charcoal yeast agar. What is the organism. A. mycoplasma pneumonia. B. klebsiella
pneumonia. C. ureaplasma. D. LEGIONELLA

32. 27year old girl came to the ER. She was breathing heavily RR 20/min. She has
numbness and tingling sensation around the mouth and tip of the fingers. What will you do? A. let
her breathe into a bag. B. ORDER SERUM ELECTROLYTES. C. first giver her 5ml of 50% glucose
solution

33. Patient with lung cancer and signs of pneumonia. What is the most common
organism? A. klebsiella. B. chlamydia. C. STREPTOCCOCUS. D. suayionhigella

34. Patient 18 years old admitted for ARDS and developped hemothorax. What is the
cause? A. central line insertion. B. HIGH NEGATIVE PRESSURE. C. high oxygen

35. COPD patient with emphysema has low oxygen, prolonged chronic high O2. The
respiratory drie maintained in this patient by. A. HYPOXEMIA. B. hypercapnea. C. patient effort
voluntary

36. The most common cause of cough in adults is. A. asthma. B. gerd. C. POSTNASAL
DRIP

37. Patient has fever, night sweating, bloody sputum, weight loss, PPD test is now
reactionary. Diagnosis is A. ACTIVATION OF PRIMARY TB. B. sarcoidosis. C. case control is backward
study

38. Best early sign to detect tension pneumothorax. A. TRACHEAL SHIFT. B. distended
neck veins. C. hypotenion
39. Holding breathing. Which of the following is true? A. mostly occur between the ages
of 5 and 10 months. B. increase risk of epilepsy. C. A KNOWN PRECIPITANT OF GENERALIZED
CONCULSION. D. diazepam may decrease attacks.

40. 58 years old male patient came with history of fever, cough with purulent foul
smelling sputum and CXR showed fluid filled cavity. What is the most likely diagnosis. A. ABSCESS. B.
tb. Bronchiectasis

41. What is the meaning of difficulty in breathing? A. DYSPNEA. B. tachycardia

42. Obese 60 year lady in the 5th day post cholecystectomy, she complains of SOB and
decreased BP 60 systolic. On examination, unilateral swelling of right leg. What is the diagnosis? A.
hypovolemic shock. B. septic shock. C.PE. D. mi. E. haemorraghic shock

43. 55 years old male patient with COPD complains of 1 week fever, productive cough.
CXR showed left upper pneumonia and culture of sputum shows positive haemophilus influenza.
What is the treatment. A. penicillin. B. doxycycline. C. CEFUROXIME. D. gentamicin. E. cabenicilln

44. Klesiella faecalis causes one of the following. A. PNEUMONIA

45. Hemoptysis, several month of PPD positive, all vaccination XRAY showed apical
filtration. PPD done again showed negative. Diagnosis is. A. sarcoidosis. B. PRIMARY OLD TB. C.
mycoplasma

46. For close contact with TB patients, what do you gie a. immunoglobulin. B anti-tb. C.
rifampicin. D. INH

47. An outbreak of TB. You should give. A. bcg vaccine. B. RIFAMPICIN. C. tetracycline.
D. h. influenza vaccine

48. Patient sustained a major trauma presented to the ER the first thing to do. A. OPEN
AIRWAY GIVE 2 BREATH. B. open airway remove foreign bodies. C. give 2 breath followed by chest
compression. D. chest compression after feeling the pulse.
49. Patient with 3 weeks history of shortness of breath with hemoptysis. The
appropriate inestigation is. A. cxr, afb, abg. B. CXR, PPD, AFB. C. ct, afb, abg

50. Treatment of community acquired pneumonia. A. AZYTHROMYCIN. B. cipro. C.


gentamicin. D. tetracycline

51. Patient had fever in the morning after he underwent surgery. What is your
diagnosis? A. ATELECTASIS. B. wound infection. C. dvt. D. uti

52. The best prophylaxis of DVT in the post op patient (safe and cost effective) is. A.
LMWH.B. warfarin. C. aspirin. D. unfractionated heparin

53. 3years old presented with shortness of breath and cough at night which resoled by
itself in 2 days. He has history of rash on his hands and allergic rhinitis. He most likely had. A. croup.
B. BRONCHIAL ASTHMA. C. epiglottitis

54. Paediatric patient came to you in ER with wheezing, dyspnea, muscle contraction(
most probably asthma). Best to give initially is. A. theophylline. B. ALBUTEROL NEBULIZER. C. oral
seroids

55. Antibiotic for community acquired pneumonia. A. gentamicin and amoxicillin. B.


ERYTHOMYCIN

56. Prohylaxis of asthma. A. oral steroids. B. INHALER STEROIDS . C. inhaler beta


agonists

57. Smoking withdrawal symptoms peak at. A. 1-2days. B. 2-4DAYS. C. 7days. D. 10-
14days

58. 6moths with cough and wheezy chest. Diagnosis is A. asthma. B. BRONCHIOLITS. C.
pneumonia. D. fb aspiration
59. Physiological cause of hypoxemia. A. hypoventilation. B. Improper alveolar diffusion.
C. PERFUSION PROBLEM(V/Q MISMATCH). D. elevated 2-3dpg

60. Child with asthma use betamethasone. Most common side effects is A. increased
intraocular pressure. B. epilepsy. C. GROWTH RETARDATION

61. The respiratory distress syndrome after injury is due to. A. pneumothorax. B.
aspiration. C. PULMONARY EDEMA. D. pulmonary embolus. E. none of the above

62. Interstitial lung disease. All is true except. A. insidious onset exertional dyspnea. B.
bibasilar inspiratory crepitations in physical examination. C. HEMOPTYSIS IS AN EARLY SIGN. D. total
lung volume is reduced.

63. The effectiveness of ventilation during CPR is measured by. A. CHEST RISE. B. pulse
oximetry. C. pulse acceleration

64. Regarding moderately severe asthma. All true except. A. PO2 <60mmHg. B.
pco2>60mmHg, early in the attack. C. pulsus paradoxus. D. iv cortisone helps in a few hours

65. What is the simplest method to diagnose fractured rib. A. posterior inferior xray. B.
lateral xray. C. ct chest. D. OBLIQUE XRAY

66. Air bronchogram is a characteristics features of pulmonary of. A. pulmonary edema.


B. hyaline membrane disease. C. LOBAR PNEUMONIA. D. lung granuloma

67. The most specific investigations for pulmonary embolism is. A. perfusion scan. B.
cxr. C. ventilation scan. D. PULMONARY ANGIOGRAPHY

68. A 62 years old male known to have BA. Hx for one month on bronchodilator
beclomethazone and been given theophylline. Side effects of theophylline is. A. g upset. B. diarrhea.
C. facial flushing. D.CARDIAC ARRHYTHMIA
69. Hx of recurrent pneumonia, foul smelling sputum with blood and clubbing. What is
the diagnosis. A. BRONCHIETASIS. B. pneumonia. C. lung abscess. D. copd

70. In mycoplasma, there will be. A. POSITIVE COLD AGGLUTININS. B. lobar


consolidation

71. Patient in ER; dyspnea, right sided chest pain, engorged neck veins and weak heart
sounds, absent air entry over right lung. Plan of treatment for this patient is. A. ivf, pain killer, o2. B.
aspiration of pericardium. C. respiratory stimulus. D. intubation. E IMMEDIATE NEEDLE
ASPIRATIONS, CHEST TUBE

72. Which of the following radiological features is a characteristic of miliary


tuberculosis. A.sparing of the lung apices. B. pleural effusion. C. septal line. D. absence of glandular
enlargement. E. PRESENCE OF SMALL CAVITY

73. A 30 years old man presents with shortness of breath after blunt injury to his chest.
RR 30/IN, CXR showed complete collapse of the left lung with pneumothorax, mediastinum was
shifted to the right. The treatment of choice is. A. CHEST TUBE INSERTION. B. chest aspirations. C.
thoracotomy and pleurectomy. D. ivf and o2 mask. E. intubation

74. Right lung. Which one is true? A. got 7 segements. B. 2 PULMONARY VEINS. C. no
relation with azygous vein

75. A 24 year old woman develops wheezing and shortness of breath when she is
exposed to cold air or when she is exercising. These symptoms are becoming worse which of the
following is the prophylactic agent of choice for the treatment of asthma in these circumstances. A.
INHALED B2 AGONISTS. B. oral aminophylline. C. inhaled anticholinergics. D. oral antihistamines. E.
oral corticosteroids

76. Which of the following regimen is the recommemeded initial treatment for most
adults with active TB. A. a two dug regimen consisting of inh and rifampicin. B a three drug regimen
consisting of inh, rifampicin ad ethambutol. C. A FOUR DRUG REGIMEN CONSISTING OF INH,
RIFAMPICIN, ETAHMBUTOL, PYRAZINAMIDE. D. no treatment for most patients until infection is
confirmed by culture. E. a five drug regimen consisting of inh, rifampicin, ethambutol, pyrazinamide
and ciprofloxacin

77. A 55 year old male presented to your office for assessment of chronic cough he
stated that he has been coughing for the last 10 years but the cough is becomig more bothersome
recently. Cough productive of mucoid sputum, occasionally becomes purulent. Past hx; 35 years
smoling 2 packs par day. On examination, 124kg, wheezes while talking. Auscultation, wheezes all
oer the lungs. The most likely diagnosis is. A. smokers cough. B. bronchiectasis. C. emphysema. D.
CHRONIC BRONCHITIS. E. fibrosing alveolitis

78. 25 years old man had fixation f fractured right femur. Two days later he became
dyspneic, chest pain and hemoptysis. ABG; Ph 7.5; pO2 65; pCO2 25. Initial treatment is. A.
furosemide. B. hydrocortisone. C. bronchoscopy. D. HEPARIN. E. warfarin

79. All of the following are true about PE except. A. NORMAL ABG. B. sinus tachycardia
is the most common finding. C. low plasma d dimer is highly predictive for excluding pe. D. spiral ct
is the investigation of choice. E. heparin should be given to all paatients with clinical suspicion of PE

80. In a child with TB, all is found except. A. hx of exposure to a Tb patient. B. cxr
findins. C. splenomegaly. D. positive culture from gastric lavage. E. ALL ARE CORRECT

81. All indicate severity of bronchial asthma except. A. intercostal and supraclavicular
retraction. B. exhaustion. Po2 60mmGh. C. Po260mmHg and PCO2 45mmHg. D. pulsusparadoxus
>20mmHg

82. Patient came with scenario of chest infection, first day of admission he was treated
with cefotaxime, next day patient case became worse with decrease perfusion and xray show
complete right side hydrothorax. Causative organism is. A. STREPTOCOCCUS PNEUMONIA. B. staph
aureus. C. h influenza. D. pseudomonas

83. Which of the following treatment is contraindicated in a asthmatic patient. A. NON


SELECTIVE BETA BLOCKERS
84. Which of the following shift oxygen dissociation to the right? A. respiratory alkalosis.
B. HYPOXIA. C. hypothermia

85. 3 years old patient has TB as a pediatrician you did PPD test after 72 hours you find
10mm induration in the child. This suggests. A. inconclusive result. B. weak positive result. C.
STRONG POSITIVE RESULT.

86. Best way to secure airway in a responsive multi injured patient is A.


NASOPHARYGEAL AIRWAY

87. Old patient with sudden onset of chest pain, cough, and hemoptysis, ecg result right
axis deviation and right bundle branch block. What is the diagnosis? A. mi. B. PE.

88. TB patient taking antiTB drugs developed color blindness. Which drug caused this
side effect? A. ETHAMBUTOL.

89. PPD positive, CXR negative. A. inh for 9 months. B. INH FOR RIFAMICIN FOR 9
MONTHS

90. Patients developed dyspnea after lying down for 2 hours frothy sputum stained with
blood, positive hepatojugular reflux, positive leg edema, oncotic pressure higher than capillary. 25%
edema is. A. INTERSTITIAL. B. venous. C. alveolar. D. capillary Patient came with cough, wheezing,
his chest monophonic sound. On xray, there is patchy shadows in the upper lobe + low volume
fibrosis. He lives in a crowded in a place. What is the injection shoud be given to the patients
contacts? A. haemophilus influenza. B.

immune globulin. C. meningococcal conjugate. D. BASIL CALAMEN

91. Patient is a known case of moderate persistent bronchial asthma. He is using


ventolin neulizer. He develpoes three attacks per week. He drug to be added is. A. increase
prednisolone. B. ADD LONG ACTING BETA AGONIST. C. ADD IPRATROPIUM. D. IV amminophyllin
92. The chromosome of CF is. A. short arm of chromosome 7. B. LONG ARM OF
CHROMOSOME 7. C. short arm of chromosome 8. D. long arm of chromosome 8. E. short arm of
chromosome 17

93. Patient presented with severe bronchial which of the following drug not
recommended . A. SODIUM GLUCONATE. B. corticosteroid inection. C. corticosteroid nebulizer

94. Lady known to have recurrent DVT came with SVC thrombosis. What is the
diagnosis? A. SLE. B. Christmas disease. C. lung cancer. D. nephrotic syndrome

95. Long scenario for patient smokes for 35years with 2 packets daily before 3 days
develop cough with yellow sputum since 3 hours became blood tinged sputum. Xray show
opacification and filtration of right hemithorax. Diagnosis. A. BRONCHOGENIC CA. B. acute
bronchitis. C.lobar pneumonia

96. Kn

97. Known case of asthma. Prevent. A. EXPOSURE TO DUST MITE

98. Patient with severe asthma. Silent chest. What is the next step. A. IV theophylline. B.
neb salbutamol

99. 82 year old female presented to ER in confusion with hypotension. Bp was


70/20mmHg. Pr 160bpm, rectal temp 37.7. the most likely of the following would suggest sepsis as a
cause of hypotension is. A. low systemic vascular resistance and high cardiac output. B. high systemic
vascular resistance and low cardiac output. C. pulmonary capillary pressure less than 25. D. ph. < 7.2.
E. serum dehydrogenase more than 22.

100. Child with picture of pneumonia treated with cefotaxime but e got worse with
cyanosis intercostal retraction and shifting of the trachea and hemothorax on xray. The organism is.
A. pneumocystis carinni. B. STREPT PNEUMO. C. staph aureus. D. pseudomonas
101. What is the most effective measure to limiting complications of COPD. A.
pneumococcal vaccination. B. SMOKING CESSATION

102. Goodpasture syndrome is associated with. A. osteoporosis. B. multiple fractures. C.


LUNG BLEEDING AND GLOMERULONEPHRITIS.

103. End stage ofCOPD. A. erythrocytosis. B. high calcium. C. LOW K.

104. Case of old male heavy smoker, on CXR there is a mass, have hyponatremia and
hyperosmolar urine. What is the cause. A. SIADH. B. pituitary failure

105. A known case of uncontrolled asthma moderately persistent on bronchodilator came


ith exacerbation and he is now okay. What will you give him to control his asthma. A. systemic
steroid. B. INHALER STEROID. C. ipratropium

106. Patient PPD test positive for TB before anti TB treatment. A. repeat ppd. B. DO
MANTOUX.

107. Old patient , smoker, copd having cough and shortness of breath in daytime and not
at night. How to treat him. A. theophylline. B. IPRATROPIUM. C. long acting.

108. Patient whith asthma use short acting beta agonist and systemic steroid.
Classification of treatment. A. mild intermittent. B. mild persistent. C. moderate. D. SEVERE.

109. Obese patient and suffering with his life. The important thing that he is snoring while
sleeping and the doctors record that he has about 80 apnea episode that extend po2 reach 75% and
no other symptoms. Exam is normal. Your action is. A. prescribe for him nasal strip. B. prescribe an
oral device. C. REFER TO AN ENT

110. Patient came with pneumocystitis carinii infection. What is your action? A. ax and
discharge. B. CHECK HIM FOR HIV.
111. Patient wake up with inability to speak. He went to a doctor. He still could not sleep
but he could cough when asked. He gave you a picture of his larynx by a laryngoscope which looks
grossly normal. A. paralysis of vocal cords. B. infection. C. FUNCTIONAL APHONIA

112. COPD coughing greenish sputum. What is the organisms. A. staph aureus. B. s.
pneumo. C. mycoplasma. D. chlamydia. E. h influenza

113. Patient with bilateral infiltration in lower lobe. Which organism is suspected. A.
LEGIONELLA. B. klebsiella.

114. Old patient was coughing then he suddenly deelopped pneumothorax. Best mgt is. A.
right pneumonectomy. B. intubation. C. TUBE THORACOTOMY. D. lung pleurodisis

115. Patient with ards. He got tension pneumothorax. What is the probable cause. A.
SEVERE LNG INJURY. B. negative pressure. C. oxygen 100%

116. Patient has pharyngitis rather he developed high grade fever then cough then
bilateral pulmonary infiltration in CXR, WBC was normal and no shift to left. What is the organism.
A.staph aureus. B. S. PNEUMO. C. legionella. D. chlamydia

117. Asbestosis: A. bilateral fibrosis --- end result. B. pleural calcification--- the specific
sign.

118. Patient suffering from wheezing and cough after exercise, not on medication. What is
the prophylactic medication? A. INHALED B2 AGONIST. B. inhaled anticolinergic. C. oral theophylline

119. Old patient stopped smoking since 10 years, suffering from shortness of breath afer
exercise but no cough and there was a table. FEV1=71%. FVC=61%. FEV1/FVC= 95%. TLC=58%. What
is the diagnosis? A. restrictive lung disease. B. asthma. C. bronchitis. D. emphysema. E. OBSTRUCTIVE
WITH RESTRICTIVE LUNG DISEASE.

120. Patient with asthma on daily steroid inhaler and short acting b2 agonist. What
category. A. mild intermittent. B. mild persistent. C. MODERATE. D. severe
121. Young patient with mild intermittent asthma, attacks once or twice a week. What is
best for him as prophylaxis. A. INHALED SHORT ACTING B AGONIST. B. inhaled steroid

122. Young lady with emphysema. .A. ALPHA1 ANTI TRYPSIN DEFICIENCY

123. Patient lives near industries came with attack of SOB. The prophylaxis is A. B2
AGONIST. B. oral steroid. C. inhaled corticosteroid

124. Young patient with unremarkable medical history presented with SOB, wheeze, long
expiratory phase. Initial mgt. A. SHORT ACTING B AGONIST. B. ipratropium. C. steroids. D. diuretic

125. If there is a relationship between anatomy and disease, pneumonia will occur in. a.
right upper lobe. B right middle lobe. C. right lower lobe. D. left upper lobe. E. left lower lobe.

nephrology

Female presented with thirst and polyuria, all medical history is negative
and she is not known to have a medical issues, she gave history of
being diagnosed as Bipolar and on lithium but her Cr and BUN is normal.
What is the cause of her presentation?
Answer - Adverse effect of lithium
Female patient was presented by dysuria, Epithelia cells were seen on
urine analysis. What is the explanation in the case? (a) Renal Cause (b)
Contamination.
Answer - Renal cause
Adenosine dose should be reduced in which of the following cases?
 Chronic renal failure (correct answer)
 Patient on theophylline
Adult polycystic kidney disease is inherited as
 Autosomal Dominant (Correct answer)
 Autosomal recessive
 Autosomal linked
Best way to diagnose post streptococcus glomerulonephritis (spot
diagnosis)
 Low C3 (correct answer)
 RBC casts
Patient came with HTN, KUB shows small kidney, arteriography shows
renal artery stenosis. What is the next investigation?
 Renal biopsy
 Renal CT scan (Correct answer)
 Renal pyelography
Female patient did urine analysis, epithelia cells in urine, it comes from
 Vulva (correct answer)
 Cervix
 Urethra
 Ureter
Female with history of left flank pain radiating to the groin, symptoms of
UTI. What is the diagnosis?
 Appendicitis
 Diverticulitis
 Renal colic (Correct answer)
IVP study done for a male and showed a filling defect in the renal pelvis
non radio opaque u/s shows echogenic structure and hyper acoustic
shadow. The most likely diagnosis is
 Blood clot
 Tumor
 Uric acid stone (correct answer)
No question
 Ciprofloxacin (correct answer)
 Ceftriazone
 Erythromycin
 Trimethoprim
 Gentamicin
Patient complaining of left flank pain radiating to the grin, dysuria and
no fever. The diagnosis is
 Pyelonephritis
 Cystitis
 Renal calculi (correct answer)
A 3 week old baby presented with a scrotal mass that was transparent
and reducible the diagnosis is
 Hydrocele (correct answer)
 Inguinal hernia
 Epididymitis
A 29 years old man complaining of dysuria. He was diagnosed as a case
of acute prostatitis. Microscopic examination showed gram negative rods
which grow on agar yeast, the organism is
 Chlamydia
 Legionella (correct answer)
 Mycoplasma
Uncomplicated UTi treatment
 TMP – SMX for 3 days (Correct answer)
 Ciprofloxacin for 5 days
Patient with renal transplant, he developed rejection one week post
transplantation, what could be the initial presentation of rejection?
 Fever (correct answer)
 Increase urine output
 Hyper coagulability
 Anaemia
Patient with hematuria and diagnosed with bladder cancer, what’s the
likely causative agent?
 Schistosoma hematobium (correct answer)
Diabetic patient on insulin and metformin has renal impairment, what’s
your next step
 Stop metformin and add ace inhibitor
Patient has saddle nose deformity, complaining of SOB, hemoptysis and
hematuria, most likely diagnoses is
 Wagner granulomatosis
Most common manifestation of renal cell carcinoma is
 hematuria (correct answer)
 Palpable mass
 HTN
Patient came with metabolic acidosis with anion gap of 18, she took
drug overdose. What could it be.
 Salicylate
Patient with excessive water drinking and frequent urinating, FBS 68,
diagnosis up to now
 Normal blood sugar
 IFG (Correct answer)
 DM
Urine analysis showed epithelia cell diagnosis is
 Renal calculi (correct answer)
 Chlamydia Urethritis
Patient with DKA, the PH=7.2 HCO3 = 5 K= 3.5 the treatment
 Insulin 10U
 2L N/s
 2L NS with insulin infusion 0.1 u/kg/hr. (correct answer)
A 6 years old presented with cola colored urine with nephritic symptoms,
the first test you would like to do
 Renal function test
 Urine microscopic sedimentation (correct answer)
 Renal ultra sound
Young adult presented with painless penile ulcer, rolled edges, what
next to do
 CBC
 Dark field microscopy (correct answer)
 Culturing
Diabetic female, her24h urine protein is 150mg
 Start on ACEi
 Refer to nephrologist
 Do nothing, this is normal range (correct answer)
Patient with plank pain, fever, vomiting treatment is
 Hospitalization and intravenous antibiotic & fluid.
Elderly patient complaining of urination during night and describe when
he feel the bladder is full and need to wake up to urinate, he suddenly
urinate on the bed, this is
 Urgency incontinence
 Urge incontinence (correct answer)
 Stress incontinence
 Flow incontinence
The best test for renal stones
 CT without contrast
70 years old male patient with mild urinary dripping and hesitancy, your
diagnosis is mild BPH. What is your next step in management
 TURP
 Start on medication
 Open prostatectomy
Patient with dysuria, frequency and urgency but no flank pain, what is
the treatment?
 Ciprofloxacin PO once daily for 3-5 days (answer)
 Norfocin Po od 7 – 14 days
Man with sudden onset of scrotal pain also had history of vomiting, on
examination tender scrotum and there is tender 4cm mass over right
groin, what will you do pre renal failure
 Cast
 Urine 05m < 400
 Urine Na < 20mmol/L (correct answer)
 Decreased water excretion
 Hematuria
Patient with history of severe hypertension, normal creatinine, 4g
protein/24 hrs. Right kidney 16cm and left kidney 7cm with suggesting
of left renal artery stenosis. Next investigation
 Bilateral renal angiography (correct answer)
 Right percutaneous biopsy
 Left percutaneous biopsy
 Right open surgical biopsy
 Bilateral renal vein determination
All of them are renal complication of NSAIDS except
 Acute renal failure
 Tubular acidosis
 Interstitial nephritis
 Upper Gi bleeding (correct answer)
Acute glomerulonephritis, all are acceptable investigation except
 Complement
 ANA
 Blood culture
 Cystoscopy (correct answer)
20 years old female present with fever, loin pain and dysuria,
management include all of the following except
 Urinalysis and urine culture
 Blood culture
 IVU (IVP) (correct answer)
 Cotrimoxazole
Urine analysis will show all except
 Handling phosphate (correct answer)
 Specific gravity
 Concentrating capacity
 Protein in Urine
In acute renal failure, all is true except
 Phosphatemia
 Uremia
 Acid phosphate increases (correct answer)
 K+ increases
A 6 years old female from Jizan with hematuria, all the following
investigation are needed
 Hbs
 Cystoscopy (correct answer)
 Hb electrophoresis
 Urine analysis
 U/s of the abdomen to see any changes in the glomeruli
Patient has bilateral abdominal masses with hematuria, what is the most
likely diagnosis?
 Hypernephroma
 Polycystic kidney disease (correct answer)
Old patient bedridden with bacteremia organism is enterococcus fecalis,
what is the source of infection?
 UTi (correct answer)
A 56 years old, his CBC showed, HB=11, MCV 92, Reticulocyte = 0.25%,
the cause is
 Chronic renal failure (correct answer)
 Liver disease
 Sickle cell anaemia
 G6P dehydrogenase deficiency
30 years old with repeated UTis, which of the following is a way to
prevent her condition
 Drink a lot of fluid (correct)
 Do daily exercise
65 years old presented with acute hematuria and passage of clot and
left loin and scrotal pain, the diagnosis
 Prostitis
 Cystitis
 Testicular cancer
 Renal cancer (correct answer)
A 5 years child diagnosed as UTi, what is the best investigation to
exclude UTi complication
 Kidney USS (correct answer)
 CT
 MCUG
 IVU
Old patient complain of Urinary incontinence occur at morning and at
night, without feeling of urgency o desire of micturition, without
exposure to any stress, what is the diagnosis
 Urge incontinence
 Stress incontinence
 Overflow incontinence (correct answer)
Heavy smoker came to you asking about cancer, not lung cancer, that
smoking increase its risk.
 Colon
 Bladder (correct answer)
 Liver
The most common cause of secondary HTN is
 Renal artery stenosis (correct answer)
 Adrenal hyperplasia
 Phaeochromocytoma
 Cushing’s disease
The most common cause of chronic renal failure
 HTN
 DM (correct answer)
 Hypertensive renal disease
 Parenchymal renal disease
 Acute glomerulonephritis
Male patient present with prostatitis (prostatitis was not mentioned in
the question), culture showed gram negative rods, the drug of choice
 Consult surgeon (correct answer)
 Consult urologist
 Do sonogram
 Elective surgery
UTi FOR 14 days, most probably cause pyelonephritis
 0.05%
 0.5%
 5%
 50%
Man have long history of urethral structure present with tender right
testis and WBC in urine, so diagnosis is
 Epididymorchitis (correct answer)
 Testicular Torsion
 Varicoele
Non opaque renal pelvis filling defect seen with IVP, US reveals dense
echoes and acoustic shadowing, what is the most likely diagnosis
 Blood Clot
 Tumor
 Sloughed renal papilla
 Uric acid stone (correct answer)
 Crossing vessels
Young age male presented after RTA with inured membranous Urethra,
best initial treatment
 Suprapubic catheter (correct answer)
 Perireal repair
 Retropubic repair
 Trans abdominal repair
Epididymitis one is true
 The peak age between 12 – 18 years
 U/S is diagnostic
 Scrotal content within normal size
 Typical Iliac fossa pain
 None of the above (correct answer)
The most important diagnostic test for previous Q is
 Microscopic RBC (correct answer)
 Macroscopic RBC
 RBC cast
A 17 years old male presented to you with history of abdominal pain and
cramps in his legs, he vomited twice, his past medical history was
unremarkable, on examination he looks dehydrated with dry mucus
membrane.
His Investigation: Na=155mmol/L, K=5.6, Glucose 23.4mmol/L,
HCO3=13, best tool to diagnose this condition
 Plain x-ray
 Ultrasound
 Gastroscopy
 Urine analysis (dip stick) (correct answer)
Patient come with abdominal pain and tender abdomen with
hypernatremia and hyperkalemia and vomiting and diarrhea. What is the
next investigation?
 Urinalysis (correct answer)
BPH all true except
 Prostitis (correct answer)
 Nocturia
 Haematuria
 Urine retention
 Diminished size and strength of stream
Screening program for prostatic Ca, the following is true
 Tumor marker (like PSA) is not helpful
 PR examination is the only test to do
 Early defection does not improve overall survival
The most accurate to diagnose acute glomerulonephritis is
 RBC cast in urinalysis (correct answer)
 WBC cast in Urinalysis
 Creatinine level increase
 Shrunken kidney in US
 Low Hb but normal indices
A 75 years old man came to ER complaining or acute Urinary retention,
what will be your initial management
 Empty the urinary bladder by foley’s catheter and tell him to come
back to the clinic
Patient present with URTi, after 1 week the patient present to have
hematuria and edema, what is the most probably diagnosis
 1g A nephropathy
 Post streptococcus GN (correct answer)
Regarding Group A strep pharyngitis, what is true
 Early treatment decrease incidence of post strept GN
All the following cause hyponatremia except
 DKA
 Diabetes insipidus (correct answer)
 High vasopressin level
 Heart failure
The investigation of high sensitivity and specificity of Urolithiasis
 IVP
 X-ray abdomen after CT scan (correct answer)
 US
 MRI
 Nuclear scan
Female patient present with dysuria, urine analysis shows epithelia cast:
 Contaminated sample
 Chlamydia urethritis
 Kidney disease (correct answer)
 Cervical disease
Patient with PID, there is over abdominal tenderness, on pelvic exam
there is small mass in ____ ligament. What is the treatment?
 Colpotomy (correct answer)
 Laparotomy
 Laparoscopy
13 year old child with typical history of nephritic (present with an Urea,
coca color urine, edema, HTN). What is the next step to diagnose?
 Renal function Test
 Urine sediment microscope (correct answer)
 US
 Renal biopsy
Young male patient with dysuria, fever, leukocytosis PR indicate soft
boggy tender prostate, DX
 Acute prostitis (correct answer)
 Chronic Prostitis
 Prostatic CA
Combination of rapid correction of hypernatremia
 Brain edema
Most common cause of ESRD
 HTN
 DM (Correct answer)
Patient have DM and renal impairment, when he had DM nephropathy,
there is a curve for albumin
 25 years (correct answer)
The most likely cause of gross hematuria in a 35 year old man is
 Cystitis
 Ureteral calculi
 Renal carcinoma
 Bladder carcinoma (correct answer)
Concerning urinary calculi. Which one of the following is true?
 75% are calcium oxalate stones (correct answer)
BPH
 TRUSS is better than PSA
 No role in PSA
 PSA role (Correct answer)
 Biopsy
An 80 year old man presented with full aching loin pain and interrupted
voiding of urine.
BUN and Creative were increased, US revealed a bilateral
hydronephrosis, what is the most probable diagnosis
 BPH (correct answer)
Patient present with testicular pain, O/E; bags of worm, what is the
diagnosis?
 Varicocele
In Testicular Torsion, all of the following are true Except
 Very tender and progressive swelling
 More common in young males
 There is hematuria (correct answer)
 Treatment is surgical
 Has to be restored within 12 hours or the testis will infarct.
A 50 years old patient complaining of Episodes of Erectile dysfunction,
history of stress attacks and he is now in stress, what will you do?
 Follow relaxation strategy.
Premature ejaculation, all true except
 Most common sexual disorder in males
 Uncommon in young men (correct answer)
 Benefits form sexual therapy involving both partners
 It benefits form anxiety RX
Child with scrotal swelling, no fever, with a blue dot in the superior
posterior aspect of the scrotum
 Testicular appendix torsion
10 years old boy woke up at night with lower abdominal pain, important
area to check
 Kidney
 Lumbar
 Rectum
 Testis (correct answer)
Old age man, feel that the voiding is not complete and extreme of urine
not strong and by examination, there is moderate BPH and PSA =
1ng/ml. what will you do?
 Surgery
 Surgical prostatectomy
An opaque renal pelvis filing defect seen with IVP, US reveals tense
echoes, and acoustic shadowing, the most likely diagnosis is
 Tumor
Old man presented with tender and enlarged prostate and full bladder
investigation, what is the likely diagnosis
 Acute renal failure
 Bladder cancer
 BPH (Correct answer)
A patient with gross hematuria after blunt abdominal trauma, has a
normal appearing cystogram after the intravesical instillation of 400ml of
contrast, you should next order
 An intravenous pyelogram (correct answer)
Patient will to cystoscope suffer from left hypochondrial pain
 Refer to vascular surgery
 Refer to urologist (correct answer)
Old patient complaining of hematuria, on investigation, patient has
bladder calculi, most common causative organism
 Schistosoma
 CMV (correct answer)
Old man with urinary incontinence, palpable bladder after voiding,
urgency and sense of incomplete voiding, diagnosis is
 Urge incontinence
Child with painless hematuria, what initial investigation
 Repeat urinalysis (correct answer)
 Renal biopsy
 Culture
Young male with a 3 day of dysuria, anal pain, O/E per rectum boggy
mass
 Acute Prostatitis
Radiosensitive testicular cancer
 Yolk sac
 Seminoma
 Choriocarcinoma
A 60 years old male known to have (BPH) DRE shows soft prostate with
multiple nodularity and no hard masses, the patient request for
screening for prostatic cancer, what will you do?
 Sit with patient to discuss the cons and rods in PSA test (answer)
82 years old patient with acute urinary retentions, the management is
 Insert a foley’s catheter then send the patient home to come back
in the clinic
Epididymitis one is true
 None of the above
Common cause of male infertility
 Primary hypogonadism (correct answer)
 Secondary hypogonadism
 Ejaculation obstruction
BPH, all are true except
 Parotitis (correct answer)
GENERAL PRACTITIONER – PSYCHIATRY MCQs

1. In battered women which is true


A. Mostly they come from poor socioeconomic area
B. Usually they marry a second violent man
C. Mostly they come to the ER complaining from other symptoms (correct)
D. Mostly they think that the husband respond like this because they still have strong feeling
for them

2. Obsessive neurosis patients will have


A. Major depression (correct)
B. Lake of insight
C. schizophrenia

3. Before giving bipolar patient lithium you will do all of the following except
A. TFT
B. LFT (correct)
C. RFT
D. Pregnancy test

4. Antidepressants associated with hypertensive crisis treatment


A. SSRI
B. MOAIs (correct)
C. TCAs

5. Partner lost his wife by AMI 6 months ago, presented by loss of appetite, low mood, sense of
guilt, whats his diagnosis
A. Bereavement (correct)
B. Major depression episode

6. 22 years old complaining of insomnia & sleep disturbance, what is the treatment
SSRI

7. The initial management of insomnia


Good sleep hygiene

8. Chronic psychotic disorder managed by


Haloperidol

9. Side effect of diazepam

10. Generalize anxiety disorder best treatment

A. SSRI (correct)
B. TCA
C. MAOI

11. About antidepressant


A. Start single type even patient have severe depression (correct)
B. Start any one of them they all have the same effect
C. Stop the medication after 2 weeks if no improvement

12. Major depression management


A. Initial MONOTHERAPY even severe depression (correct)
B. Treatment should be change if no response during 2wk (AT LEAST 6 WEEKS)
C. Psychotherapy, medication, and electroconvulsive therapy

13. Major depression disorder treatment


Citalopram

14. Patient having major depression and taking medicine for it, after taking medicine she is
complaining of insomnia and irritable, which medications is she taking
A. SSRI (correct)
B. TCA
C. MAO
D. ECT

15. Why SSRI are the 1st line treatment of major depression?
A. Less expensive
B. Most tolerable and effective (correct)

16. Psychiatric patient with un compliance of drugs treatment


A. Depot haloperidol injection (correct)
B. Oral clonazepam

17. Patient with depression started on amitriptyline, he had headache or dizziness, vomiting, “I
am not sure what exactly was the symptoms”
Change to SSRI

18. Unfavorable prognosis for schizophrenia


A. Family history (correct)
B. Failed marriage
C. Adolescence age
D. Presence of psychosis

19. Which of the following indicates good prognosis in schizophrenia


A. Family history of schizophrenia
B. Gradual onset
C. Flat mood
D. Prominent affective symptoms (correct)
E. No precipitating factors

20. SSRI was prescribed to a patient with depression, the effect is suspected to be within
A. One day
B. Two weeks
C. Three to four weeks (correct)

21. Which of the following personality is characterized by inflexibility, perfectionism?


A. Narcissistic personality disorder
B. Borderline personality disorder
C. Obsessive compulsive personality disorder (correct)
D. Histrionic personality disorder

22. Best drug to treat depression in children and adolescent is


Fluoxetine (Prozac)

23. Alternative therapy for severe depression and resistance to anti-depressant medications are
A. SSRI
B. TCA
C. ECT (correct)

24. Patient had history of pancreatic cancer on chemotherapy then improved completely, came
to doctor concerned about recurrence of cancer and a history of many hospital visits. This
patient has
A. Malingering
B. Hypochondriasis (correct)
C. Factitous
D. Conversion

25. Patient came with symptoms of anxiety including palpitation, agitation, and worry. The first
best line for treatment is
A. SSRI (correct)
B. TCA
C. B-blocker
D. MAOI

26. Patient came with hallucination and illusion the medication that should be given is
A. Carbamazepine
B. Haloperidol (correct)

27. Recent study revealed that anti-psychotic medication cause the following complication
A. Weight gain (correct)
B. Alopecia
C. Cirrhosis

28. Female patient developed extreme fear from zoo, park, sporting events, the fear prevented
her from going out
A. Agoraphobia (correct)
B. Social phobia
C. Schizophrenia
29. Which psychiatric disease is treated with electroconvulsive therapy
A. Paranoia
B. Major depression (correct)

30. Patient turns to be erratic, for 4 months, he said that people in TV knows what people are
thinking about, in last 2 months he claim that he has special power that no one has, what is
the most likely diagnosis
A. Uni-polar ….depression
B. Bi-polar …Mania
C. Schizophrenia (correct)

31. In dementia, best drug to use


A. Haloperidol
B. Galantamine (correct)

32. Patient with mushroom toxicity will present with


A. Constipation
B. Hallucination (correct)
C. Anhidrosis

33. 12 years old boy is mocked at school because he is obese, ate a lot of pill to sleep and never
wake up again, best management is
A. Refer to mental professional (correct)
B. Tell him that most kid grow out before they grow up
C. Advice healthy food

34. Man walking in street and saying bad words to strangers, he is not aware of his condition,
what Is the description
A. Flight of idea
B. Insertion of idea
C. Loosening of association (correct)

35. A 60yr old patient with history of heart attack 6 weeks ago, complaining of not getting
enough sleep.
Psychiatric evaluation is unremarkable for depression or anxiety, what should be given to
this patient?
A. Amitriptyline
B. Buspirone
C. Buprionfe
D. Zolbidem (correct )
36. Young female with BMI 18, fine hair all over body, feeling of she is fat, doesn’t eat well with
excessive exercise….
A. Anorexia nervosa (correct)
B. Body dysmorphic disorder
C. Bulimia nervosa

37. Vertigo and tinnitus are caused by which of the following drug?
A. Amphotericin B (correct)
B. Penicillin reaction
C. INH

38. A 25 year old secondary school teacher says that every time she enters the class room she
starts sweating and having palpitations. She is fired to give wrong information and unparsed.
What is the diagnosis
A. Specific phobia
B. Social phobia (correct)

39. A patient is having a 2 year history of low interest in life, he doesn’t sleep well and can’t find
joy in life. What is the most likely diagnosis
A. Dysthymia (correct)
B. Major depressive disorder
C. Bipolar disorder

40. What is the mechanism of OCD drugs?


A. Increase availability of serotonin (correct)
B. Decrease production of serotonin
C. Increase production of serotoinin

41. Young female develops flushing face and tremors when she talks to anyone. What’s the
treatment?
Beta blocker (there is no SSRI in choices)

42. Which of the following antidepressant is not given in erectile dysfunction


A. Sertraline (correct)
B. Amitriptyline
C. Butriptyline

43. Patient complains of loss of association and circumstantionciality the defect in


Form

44. 44 years old mother of 3 presented with bouts of shortness of breath, fatigue, dizziness,
chest discomfort. She thinks about her job and children a lot. She is doing well at her job
A. Depression
B. Panic attack
C. Generalized anxiety disorder (correct)
D. Social phobia

45. Which of the following antipsychotic is associated with weight gain


A. Respiridone
B. Quitapine
C. Olanzapine (correct)
D. Ziprasidone

46. Best treatment for bulimia nervosa


Cognitive behavior therapy

47. 60 year old male presented with depressed mood, loss of interest, sleep disturbance after
death of his son 3 months back after a prolonged illness. What’s your diagnosis?
Bereavement

48. Adult male complained of inability to sleep as usual. Every night he should check that the
light is off, oven is off, and child sleep. This occur also at morning and every day he cannot
sleep if he didn’t do this. He knows this is abnormal behavior and feeling bad.. whats his
diagnosis
A. Generalized anxiety disorder
B. Depression
C. Obsessive compulsive disorder (correct)

49. Young male with depression on citalopram present with toxicity of unknown substance.
Investigation result showed metabolic acidosis with anion gap of 18. What is the cause
A. Citalopram
B. Aspirin (correct)
C. Paracetamol

50. Patient came with expressive talking and unable to concentrate in one topic. Diagnosis?
A. Flight of ideas (correct)
B. Insertion of ideas

51. Patient came to you complaining of hearing voices, later started to complain of thoughts
getting into his mind and can be taken out
A. Schizophrenia (correct)
B. Mood
C. Mania
D. Agoraphobia
52. Female had history of severe depression, many episodes, she got her remission for three
months with paroxetine (SSRI). Now she is pregnant. Advise her
A. Stop SSRIs because it cause fetal malformation
B. Stop SSRI because it cause premature labour
C. Continue and monitor her depression (correct)
D. Stop SSRIs

53. One of the anti-psychotics causes ECG changes, leukopenia, drooling


A. Respiredone
B. Clozapine (correct)
C. Amisulpride

54. Patient with 2 months insomnia, memory is intact, with symptoms of psychosis.
Management?
A. Lithium
B. Carbamezipine
C. Venlafaxine (correct)

55. What comes with bulimia?


Elevated liver enzymes

56. One of these antipsychotics is the least to cause tardive dyskinesia


A. Clozapine (correct)
B. Haloperidol
C. Risperidone

57. What is the best treatment of somatization?


A. Multiple appointment (correct)
B. Multiple telephone calling
C. Antidepressant
D. Send him to chronic pain clinic

58. Adult age complain of tight headache, mostly periorbital, and has stress at work
A. Tension headache (correct)
B. Migraine

59. Side effect of prolong use of 100% O2 therapy


A. Depression
B. Dizziness (correct)
C. Ocular toxicity
D. Seizures
60. A 40 year old man who becomes sweaty with palpitation before giving a speech in public,
otherwise he does very well at his job, he is having
A. Generalized anxiety disorder
B. Performance anxiety (correct)
C. Agoraphobia
D. Depression
61. A woman who lost her husband 2 weeks ago is unable to sleep at all. What will you give her
A. Fluoxetine
B. Diazepam (correct)
C. Haloperidol
D. Amitriptyline

62. Which of the following antipsychotic medication routes have rapid onset of action
A. Sublingual
B. Oral
C. IM (correct)
D. IV

63. Patient with severe depression and now he shows some improvement with therapy, the risk
of suicide now is
A. No risk
B. Become greater (correct)
C. Become lower
D. No change

64. A 70 year old female brought to your clinic by her daughter. The daughter said her mom’s
memory deteriorated in the last 2 years. She can cook for herself but sometimes leave the
oven on. She can dress herself but with difficulties. The daughter mentioned her mother’s
personality changed into a more aggressive person (patient has Alzheimer’s disease).
According to the history what is your appropriate management?
A. prescribe diazepam for the daughter and haloperidol for the mother
B. refer the mother into chronic illness institute
C. refer mother to geriatric clinic (correct)
D. immediate hospitalization

65. A man was intent as if he was listening to somebody, suddenly started nodding & muttering.
He is having
A. Hallucination (correct)
B. Delusion
C. Illusion
D. Ideas of reference
E. Depersonalization
66. A female patient present to you complaining of restlessness, irritability, and tachycardia.
Also she has excessive worries when her children go outside of the house. What’s your
diagnosis?
A. Panic disorder
B. Generalized anxiety disorder (correct)

67. Male patient who is otherwise healthy has depression for 4 months. He retired 6 months
ago. Examination unremarkable except for jaundice. What’s your diagnosis?
A. Major depressive disorder
B. Mood disorder due to medical illness
C. Adjustment disorder; depressed type (correct)

68. 43 year old female presented to ER with history of paralysis of both lower limbs and
paresthesia in both upper limbs since 2 hours ago, she was seen lying on stretcher & unable
to move her lower limbs (neurologist was called but he couldn’t relate her clinical findings to
any medical disease ). When history was taken, she was beaten by her husband. The most
likely diagnosis is
A. Complicated anxiety disorder
B. Somatization disorder
C. Conversion disorder (correct)
D. Psychogenic paralysis
E. Hypochondriasis

69. The best treatment for the previous case is


A. Benzodiazepines
B. Phenothiazine
C. MAOI
D. SSRI
E. Supportive psychotherapy (correct)

70. 28 year old lady, complaining of chest pain, breathlessness and feeling that she’ll die soon.
On examination just slight tachycardia otherwise unremarkable, what’s the most likely
diagnosis
Panic disorder

71. 65 year old lady came to your clinic with history of 5 days insomnia and crying (since her
husband died). The best treatment for her is:
A. Lorazepam (correct)
B. Fluoxetine
C. Chlorpromazine
D. Haloperidol
72. Lady on imipramine feels dizzy on standing, resolves after 10-15 minutes on sitting, decrease
on standing, most likely diagnosis
Orthostatic hypertension

73. What is the most appropriate treatment for the above patient
A. Antiemetic
B. Antihistamine
C. Change the antidepressant to SSRI (correct)
D. Thiazide diuretics
E. Audiometry

74. Generalized anxiety disorder best treatment


A. SSRI (correct)
B. TCA

75. Unilateral headache, throbbing, decrease in the dark


Migraine

76. Tyramine increases the side effects of


MAOI

77. Generalized anxiety treatment is


Buspirone or SSRI

78. A mother came with her son who is 7 years old, very active, never sitting in class, and with
poor concentration. Your management would be
A. Olanzapine
B. Amitriptyline (correct)
C. Aloxane

79. 80 year old living in nursing home for the last 3 months. His wife died 6 months ago and he
had a coronary artery disease in the last month. He is now forgetful especially of short term
memory and decrease eye contact and loss of interest. Diagnosis?
A. Alzheimer’s
B. Depression (correct)
C. Hypothyroidism

80. Hallucinations and paranoia


A. Schizophrenia (correct)
B. Mood
C. Mania
D. Phobia
81. Obsessive neurosis
A. Treatment is easy
B. Clomipramine doesn’t work
C. Mostly associated with severe depression (correct)
D. Can be cured spontaneously
82. Patient developed sudden loss of vision bilaterally while she was walking in the street,
followed by numbness, the subjective symptoms are different from objective symptoms, and
does not match anatomically. What’s your diagnosis?
Conversion syndrome

83. Treatment of mania that does not cause hepatotoxicity


Lithium

84. The best initial Treatment for depression is


A. SSRIs (correct)
B. Tricyclic depressants
C. MAOI
D. Beta blockers

85. 25 year old teacher have fear attack and worry before entering the class, what’s the initial
treatment?
A. SSRI (correct)
B. Tricyclic depressant
C. Beta blocker

86. Female with hair on different side of body and refuse intake of food and BMI<18 and feel as
body is fat. Diagnosis?
A. Anorexia nervosa (correct)
B. Bulimia nervosa
C. Body dysmorphic syndrome
D. Anxiety

87. Psychiatric patient on antipsychotic drug. Which drug mostly leads to impotence with
antipsychotic
A. Propranolol (correct)
B. NSAI
C. ACEI

88. Effect of fluoxetine starts after


A. 1-2 weeks (correct)
B. 3-4 weeks
89. What is the half-life of fluoxetine (Prozac)
A. 2 hours
B. 18 hours
C. 2 days
D. 6 days (correct)
E. 8 days

90. What is the half-life of sertraline


A. 2 hours
B. 18 hours
C. 2 days (correct)
D. 8 days

91. 29 year old teacher has recurrent attacks of intense fear before the beginning of her classes
in the secondary school, she said “it’s only a matter of time before I make mistakes.
Diagnosis?
A. Specific phobia
B. Social phobia (correct)
C. Mixed phobia
D. Panic attacks with agoraphobia
E. Panic without agoraphobia

92. The best treatment for the previous patient(….no answer!!!!!)


A. Alprazolam
B. Aropropanolol (propranolol?) (wrong)
C. Chlorpromazine
D. Clomipramine

93. Patient with schizophrenia, the best prognostic sign is


A. Gradual onset
B. Family history of schizophrenia
C. Age of patient
D. Coincidence of other
E. Psychological problems (correct)

94. Regarding antidepressant side effects, all of the following are true EXCEPT
A. Anticholinergic side effect tend to improve with time (wrong)
B. Sedation can be tolerated by prolonged use
C. Small doses should be started in elderly
D. Fluoxetine is safe drug to use in elderly

95. One of the following is secondary presenting complaint in patient with panic attack disorder
A. Dizziness
B. Epigastric pain
C. Tachycardia
D. Chest pain
E. Phobia (correct)

96. Treatment of GAD it should be SSRI


A. TCA
B. Benzodiazepine (correct)
C. Busrilone

97. Patient came with symptoms of anxiety including palpitation, agitation, and worry. The first
best line for treatment is
A. SSRI (correct)
B. TCA
C. B-blocker
D. MAOI

98. Electroconvulsive therapy indications are


A. Rapid
B. Highly effective (wrong)
C. Lifesaving

99. Patient came with symptoms of anxiety including palpitation, agitation and worry. The first
line for treatment is
A. SSRI (correct)
B. TCA
C. B-Blocker
D. MAOI

100. In the case of failure of anti-psychotic drugs use


A. Person centered psychotherapy (wrong)
B. Pect
C. Behavioral therapy

1. Psychiatry patient whom swallowed a small pin 5 hours ago, came to the hospital and
showed an xray which showed pins in the small intestine and no free air. What will be the
action
A. Admit and do a CT scan or MRI
B. Investigations only to CT and MRI
C. Give laxatives
D. Admit and do surgery to remove the pins (wrong)

2. You prescribed paroxetine to depressed man, u should inform him that


Drug starts working after 3-4 weeks

3. 70 years old with progressive dementia, no personality changes, neurological examination


was normal but there is visual deficit, Brain CT showed cortical atrophy and ventricular
dilatations
A. Multi micro infarct dementia
B. Alzheimer’s dementia (correct)
C. Parkinsonism dementia

4. 70 year old with progressive dementia, on brain microscopy amyloid plaques and
neurofibrillary tangles are clearly visible, also plaques are seen. Diagnosis?
A. Lewy dementia
B. Parkinsonism
C. Alzheimer (correct)

5. Patient complain from diplopia, nausea, vomiting, back pain


Somatization disorder

6. Young girl recently failed in math exam, came with paraesthesia


A. Hyperventilation syndrome (correct)
B. Generalized anxiety disorder

7. 20 year old lady thinks that she is fat although her height and weight are ok
A. Bulimia nervosa
B. Anorexia nervosa (correct)
C. Depression

8. Delusion
A. Perception of sensation in absence of an external stimulus
B. Misinterpretation of stimulus
C. False belief not in accordance of a person’s culture (correct)

9. He has gastric cancer, he went to 6 gastroenterologist did CT, barium enema and series of
investigations all were normal. What’s the diagnosis?
A. Hypochondriasis (correct)
B. Conversion
C. somatization

10. 27 year male with tonic clonic in ER. 20mg diazepam was given & convulsion did not stop.
What will u give?
A. Diazepam till total dose of 40mg (correct)
B. Phenytoin
C. Phenobarbitone
11. Characteristic feature of major depressive illness is
A. Late morning awakening
B. Hallucination and flight of ideas
C. High self esteem
D. Over eating (correct)
E. Decreased eye contact during conversation

12. Severe postpartum depression mostly associated with


A. Decrease socioeconomic class
B. Emotional separation between the patient & his mother
C. Past history of depression (correct)
D. 1st birth delivery
E. Poor weight gain during pregnancy

13. Anorexia nervosa, all true except (all options re correct)


A. Lethargy
B. Lanugo hair
C. Amenorrhea
D. Young female

14. A male presented with headache, tinnitus and nausea thinking that he has a brain tumor. He
had just secured a job in a prestigious company and he thinks that he might not meet its
standards. CNS exam, CT all normal. Diagnosis?
A. Generalized anxiety disorder
B. Hypochondriasis (correct)
C. Conversion reaction
D. Panic attack

15. 65 year old male with hypertension, congestive heart failure and peptic ulcer disease came
to your office for his regular blood pressure check. Although his blood pressure is now under
control, he complains of an inability to maintain an erection. He currently is taking
propranolol, verapamil, hydrochlorothiazide, and ranitidine. On examination his blood
pressure is 125/76mmHg. His pulse is 56 and regular. The rest of the cardiovascular
examination and the rest of the physical examination are normal. Which of the following
generally considered the MOST common cause of sexual dysfunction?
A. Pharmacological agents (correct)
B. Panic disorder
C. Generalized anxiety disorder
D. Major depressive disorder
E. Dysthymic disorder

16. 26 year old patient came to your office with recurrent episodes of binge eating
(approximately four times a week) after which she vomits to prevent weight gain. She says
that “she has no control” over these episodes and becomes depressed because of her
inability to control herself. These episodes have been occurring for the past 2 years. She also
admits using self-induced vomiting, laxatives and diuretics to lose weight. On examination,
the patient’s blood pressure is 110/70 mmHg and her pulse is 72 and regular. She is not in
apparent distress. Her physical examination is entirely normal. What is the MOST likely
diagnosis in this patient?
A. Borderline personality disorder
B. Anorexia nervosa
C. Bulimia nervosa (correct)
D. Masked depression
E. Generalized anxiety disorder

17. Hopelessness most likely predictor


A. Suicide (correct)
B. Impulse control problem

18. 23 years old female came to your office with a chief complain of having “a peculiar jaw”. She
tells you she has seen a number of plastic surgeons about this problem, but “everyone has
refused to do anything”. On examination, there is no protrusion that you can see, and it
appears to you that she has a completely normal jaw and face. Although the physical
examination is completely normal, she appears depressed. What’s the MOST likely diagnosis
in this patient?
A. Dysthymia
B. Major depressive disorder with somatic concerns
C. Somatization disorder
D. Body dysmorphic disorder (correct)
E. Hypochondriasis

19. Known risk factors for suicide include all of the following EXCEPT
A. Repeated attempts at self-injury
B. Male sex
C. Symptoms of depression with guilt
D. Drug and alcohol dependence
E. If the doctor asked the patient about suicide (correct)

20. Hypochondriasis, all true EXCEPT


A. More common in medical students
B. Less common in male (correct) -no sex predominance
C. More common in lower social class
D. Defined as morbid preoccupation of one’s body or health

21. Family behavior toward schizophrenic patient affect prognosis adversely


A. Double binding
B. Over emotion behavior
C. Schismatic parents
D. Projective identification (wrong)

22. The investigation to confirm Alzheimer’s


A. CT of the brain (correct)
B. EEG
C. Neurological examination
D. Lab investigations

23. Antidepressant in patient with somatization disorder and depression


A. Elderly need lower dose
B. Potential side effect shouldn’t be discussed
C. Fluoxetine safe in elderly
D. Effectiveness assessed after few weeks (correct)
E. Need monitoring of antidepressant level

24. How to treat social phobia


Propranolol

25. Female patient manager since short time, become depressed, she said she can’t manage the
conflicts that happen at work between the employees. Diagnosis?
A. Depression
B. Generalized anxiety disorder
C. Adjustment disorder (correct)

26. Patient present 2-3 days before menses with depressed mood that disappear by 2-3 days
after the beginning of menses. Diagnosis?
Premenstrual dysphoric disorder if severe symptoms (or premenstrual syndrome)

27. Female patient complain of thirst & drinking a lot of water, & frequent urination, she has a
history of diagnosed bipolar since 2 weeks, started on lithium
A. Psychogenic polydipsia
B. Central diabetes insipidus
C. Nephrogenic diabetes insipidus (correct)

28. Man changed his job, he must talk in front of 50 persons in his new job, he feels he cannot
do this and he send his friend to do that instead of him. How can you help him?
A. Propranolol
B. Biofeedback (correct)

29. Patient looks to his right most of the time when talking to doctor, when asked why is that,
he said that his mother is there but in fact no one is there, after asking the patient family
they said that the mother died when he was a child, what is the diagnosis?
A. Visual hallucination (correct)
B. Auditory hallucination
C. Psychosis

30. 50 years old patient complaining of episodes of erectile dysfunction, history of ?stress
attacks and now he is stressed. What will u do?
A. Follow relaxation strategy (correct)
B. Viagra
C. Ask for investigations including testosterone

31. Patient has fear, shortness of breath, sweating when he is in automobile


A. Specific phobia (correct)
B. Panic disorder
C. Generalized anxiety disorder
D. Post-traumatic stress disorder

32. Treatment of hallucination and delusion


Antipsychotic

33. The best way to treat binge induced nervosa


A. Interpersonal psychotherapy
B. Cognitive behavior therapy (correct)
C. Pharmacotherapy

34. The drug used in maintenance phase of bioplaris (?bipolar)


A. Lithium (correct)
B. Na valproate

35. The antidepressant used for secondary depression that cause sexual dysfunction
A. Sertraline
B. Imipramine
C. Levofluxine

N.B: Questions in bold had no correct answers on the marking scheme. The bold
option in each question indicates the correct answer. Also some of the questions
had just one option (maybe the correct option)

G.P. CARDIOLOGY MCQs


1. Old Patient Presented with abdominal pain, back pain, pulsatile abdomen. What is
the next step to confirm diagnosis?
a) Abdominal USS
b) Abdominal CT
c) Abdominal MRI
2. How to diagnose DVT:
a) Contrast Venography
b) Duplex US
3. Drug that will delay need of Surgery in AR:
a) Digoxin
b) Verapamil
c) Nifedipine
d) Enalapril
4. Secondary Prevention is:
a) Detection of Asymptomatic DM Patient
5. Anticoagulation prescribe for
a) One Month
b) 6 months
c) 6 weeks
d) One Year
6. Patient with Left Bundle Branch Block will go for Dental Procedure, regarding
endocarditis prophylaxis:
a) No need
b) Before Procedure
c) After the Procedure
7. When to give Aspirin and Clopidogrel?
a) Patient with history of previous MI
b) Acute MI
c) History of Previous Ischaemic Stroke
d) History of Peripheral Artery Disease
e) After Cardiac capt
8. In Patients with hypertension and diabetes, which antihypertensive would you
want to add first?
a) B-blockers
b) ACE Inhibitor
c) ?Alpha-Blocker
d) Calcium Channel Blocker
9. ECG findings of Acute Percarditis?
a) ST Segment elevation in all leads
b) Low-Voltage, diffuse ST-segment elevation
10. 59year old presented with new onset supraventricular tachycardia with
palpitation, no hx of SOB or chest pain. Chest examination normal, O2 saturation in
room air = 98%, no peripheral edema. Others normal. The best initial investigation:
a) ECG stress test
b) Pulmonary arteriography
c) CT scan
d) TSH
11. The Mechanism of action of Aspirin:
a) Inhibit Cycloxygenase
b) Inhibit Phospholipase A2
c) Inhibit Phospholipid D
12. A known case of Chronic Atrial Fibrillation on the Warfarin 5mg came for follow
up. You find INR 7 but no signs of bleeding. Your advice is:
a) Decrease dose to 2.5mg
b) Stop the dose and repeat INR next day
c) Stop Warfarin
d) Continue same and repeat INR
13. Patient is a known case of CAD. The best exercise:
a) Isotonic exercise
b) Isometric exercise
c) Anaerobic Exercise
d) Yoga
14. A known case of treated Hodgkin Lymphoma (mediastinal mass) with
radiotherapy. Not on regular follow up, presented with gradual painless difficulty in
swallowing and SOB. There is facial swelling and redness. Dx:
a) SVC Obstruction
b) IVC obstruction
c) Thoracic aortic aneurysm
d) Abdominal aortic aneurysm
15. Complication of sleep apnea is:
a) CHF
16. Which is not found in Coarctation of the aorta:
a) Upper limb hypertension
b) Diastolic murmur heard all over the Precordium
c) Skeletal deformity on CXR
17. Which of the following medication if taken need to take the patient immediately
to the hospital:
a) Penicillin
b) Diphenhydramine
c) OCPs
d) Quinine or Quinidine
18. True about alpha Blocker:
a) Causes hypertension
b) Worsens Benign prostatic hyperplasia
c) Cause tachycardia
19. Which of the following drugs increases the survival in a patient with heart failure:
a) B-Blocker
b) ACE Inhibitors
c) Digoxin
d) Nitrites
20. Elderly Patient presented with SOB, rales in auscultation, high JVP, +2 Lower limb
edema. The main Pathophysiology is:
a) Left ventricular dilatation
b) Right ventricular dilatation
c) Aortic regurgitation
d) Tricuspid regurgitation
21. 60year old presented with recurrent venous thrombosis including Superior
venous thrombosis. This patient most likely has:
a) SLE
b) Nephrotic syndrome
c) Blood Group O
d) Antiphospholipid Syndrome
22. IV drug abuser was presented by fever, arthralgia and conjunctival haemorrhage.
What is the diagnosis?
a) Bacterial endocarditis
23. Which of the following is the commonest complication of patients with chronic
atrial fibrillation?
a) Sudden death
b) Cerebrovascular accidents due to multiple atrial thrombi
24. Which of the following is the recommended diet to prevent IHD?
a) Decrease the intake of meat and dairy
b) Decrease the meat and bread
c) Increase the intake of fruits and vegetables
Arterial injury is characterised by:
a) Dark in color and steady
b) Dark in color and spurting
c) Bright red and steady
d) Bright red and spurting
Patient has fatigue while walking last night. He is on Atorvastatin for 8 months,
Ciprofloxacin, Diltiazem and alphaco. The cause of this fatigue is
a) Diltiazem and Atorvastatin
b) Atorvastatin and Ciprofloxacin
c) Atorvastatin and Alphaco
Obese lady with essential hypertension, lab work showed high Na and K. What is the
reason for her Hypertension?
a) Obesity
b) High Na Intake
c) High K Intake
All the following are risk factors for heart disease except:
a) High HDL
b) Male
c) Obesity
True about systolic hypertension
a) Could be caused by mitral regurgitation
b) More serious than diastolic hypertension
c) Systolic>140 and diastolic >90
Patient with continuous murmur
a) PDA
b) Coarctation of the Aorta
Patient has High BP on multiple visits so he was diagnosed with hypertension. What
is the Pathophysiology?
a) Increased Peripheral Vascular resistance
b) Increased salt and water retention
Prophylaxis of Arrythmia Post MI:
a) Quinidine
b) Quinine
c) Lidocaine
d) Procainamide
Best single way to reduce high blood pressure is:
a) Smoke cessation
b) Decrease lipid level
c) Reduce weight
Drug of choice for Supraventricular tachycardia is:
a) Adenosine
Which of the following Pulse Characters goes with the disease?
a) Collapsing Pulse & severe anaemia
b) Pulsus Alternans and Premature Ventricular Complex
c) Slow rising pulse and mitral stenosis
d) Pulsus Bisferiens and mitral regurgitation
e) Pulsus paradoxus and aortic stenosis
An Old Patient presents with history dizziness and falling down 1 day ago
accompanied by history of epigastric discomfort. He has very high tachycardia
around 130-140 and BP 100/60. What is the diagnosis?
a) Peptic Ulcer
b) GERD
c) Leaking aortic aneurysm
Patient with Orthostatic hypertension. What is the mechanism?
a) Decreased Intravascular Volume
b) Decreased Intracellular Volume
c) Decreased Interstitial Volume
Which of the following anti-HTNsives is contraindicated for an uncontrolled DM
patient?
a) Hydrochlorothiazide
b) Losartan
c) Hydralazine
d) Spironolactone
69year old non diabetic with mild HTN and no history of Coronary Heart disease, the
best drug in treatment is:
a) Thiazide
b) ACEI
c) ARB
d) CCB
Which of the following decrease mortality after MI?
a) Metoprolol
b) Nitroglycerine
c) Thiazide
d) Morphine
Cause of sudden death in athlete is:
a) HOCM
Male Patient with HTN on Meds, well controlled. He is using Garlic water and is
convinced that it is the reason for BP ctrl. What would U do as his Physician?
a) Tell him to CT using it
b) To stop the medication and continue using it
c) Tell him that he is ignorant
d) To stop using garlic water
Patient with rheumatic fever after untreated Strep infection after many years
presented with mitral regurg. The cause of massive regurg is dilatation of
a) Right atrium
b) Right ventricle
c) Left atrium
d) Left ventricle
Regarding MI all true except
a) Unstable angina, longer duration of pain and can occur even at rest
b) Stable angina, shorter duration and occur with excretion
c) There should be Q wave in MI
d) Even if there is very painful unstable angina the cardiac enzymes will be
normal
Asystole in adult
a) Adrenalin
b) Atropine
After doing CPR on child and showing asystole:
a) Atropine
b) Adrenaline
c) Lidocaine
Classic Scenario of stroke on diabetic and HTnsive Patient. What is the
pathophysiology?
a) Atherosclerosis
b) Aneurysm
Middle-aged patient with an acyanotic congenital heart disease. Xray showed
ventricle enlargement and pulmonary HTN
a) VSD
b) ASD
c) Truncus arteriosus
d) Pulmonary Stenosis
Middle-aged acyanotic male with CXR showing increased lung marking and enlarged
pulm artery shadow. Most likely diagnosis?
a) VSD
b) Aorta Coarctation
c) ASD
d) Truncus Arteriosus
Most common cause of secondary hypertension in female adolescent is
a) Cushing Syndrome
b) Hyperthyroidism
c) Renal Disease
d) Essential HTN
e) PCOS
Most common cause of intracerebral haemorrhage:
a) Ruptured aneurysm
b) Hypertension
c) Trauma
Cause of syncope in aortic stenosis
a) Systemic hypotension
Medical Student had RTA systolic pressure is 70mmHg, what will you do next in Mgt:
a) IV fluid therapy
b) ECG
c) Abdominal U/S
25yr old female with bradycardia and palpitation. Normal ECG except HR=130 and
apical pulse = 210. Past Hx of Full ttt Ovarian teratoma so your advice is
a) Struma ovarii should be considered
b) Vagal stimulate should be done
c) Refer to Cardiology
55yr old with dyspnoea, PND and past Hx of Mitral Valve disease. Diagnosis?
a) Left sided HF
b) Right sided HF
c) Pneumothorax
d) PE
The first sign of left sided HF?
a) Orthopnea
b) Dyspnea on exertion
c) Pedal Edema
d) PND
e) Chest pain
Middle-aged male is involved in RTA. RR=30/min, Muffled Heart Sounds and elevated
JVP. BP is 80/40 and a bruise over the sternum. What is the diagnosis?
a) Pericardiac Tamponade
b) Pneumothorax
c) Pulmonary Contusion
d) Hemothorax
Oral anticoagulants:
a) Can be given to pregnant women during first trimester
b) Can be reversed within 6 hours
c) Are enhanced by barbiturates
d) Cannot cross Blood-brain barrier
e) None of the above
The following are features of rheumatic HD except:
a) Restless involuntary abnormal movement
b) Rashes of trunk and extremities
c) Short P-R interval on ECG
d) Migratory arthritis
Premature Ventricular Contracture (PVC), all are true except:
a) Use of antiarrhythmic post MI improves Prognosis - this is not totally true as
class 1 increase mortality
b) Use of antiarrhythmic type 1 increases mortality
One of the ff is NOT useful in patient with atrial fibrillation and stroke:
a) Aspirin and AF
b) Warfarin and AF
c) Valvular HD can lead to CVA in young patients
d) AF in elderly is predisposing factor
Shoulder pain most commonly due to:
a) Infraspinatus muscle injury
b) Referred pain due to cardiac ischaemia
c) In acute cholecystitis
ECG stress test is indicated in the following except:
a) Routine (yearly) test in asymptomatic patients
b) In high risk jobs
c) 40year old patient before starting exercise program
Most serious symptom of CO poisoning:
a) Hypotension
b) Arrhythmia
c) Cyanosis
d) Seizure
35year old male has SOB, orthopnea, PND, nocturia and lower limb edema. What’s
the most common cause of this condition in this patient:
a) Valvular HD
b) UTI
c) Coronary Artery Disease
d) Chronic HTN
5days after MI, the patient developed SOB and crackles in both lungs. Most likely
cause is:
a) Pulmonary Embolism
b) Acute Mitral Regurgitation
70year old male came with hx of leg pain after walking, improved after resting, he
noticed loss of hair in the shaft of his leg and become shiny;
a) Chronic limb Ischaemia
b) DVT
Patient comes to the ER with weak rapid pulse, what is your next step?
a) Give him 2breaths
b) Do CPR (2 breaths/30 Compressions)
c) Wait until resuscitation group comes
DM with controlled blood sugar and BP of 138/89. What will be your next step?
a) Nothing
b) Add ACEI
ECG shows ST elevation in leadsnV1, 2, 3, 4 & reciprocal changes in leads aVF&2,
what is the diagnosis?
a) Lateral MI
b) Anterior MI
c) Posterior MI
Which of the following is considered as HMG-CoA reductase inhibitor?
a) Simvastatin
b) Fibrate
DVT for a lady, best mgt?
a) Bed rest, warfarin and heparin
50year old diagnosed with hypertension. He is used to drinking one glass of wine
every day, also gets high Na and K intake. BMI is 30kg/m. What is the strongest risk
factor for having HTN in this patient?
a) Wine
b) High Na Intake
c) High K Intake
d) BMI=30
Patient wants to do dental procedure. He was diagnosed to have mitral valve
prolapse clinically by a Cardiologist. He had never done Echo before, what is
appropriate action?
a) Do Echo
b) No need for Prophylaxis
c) Give Ampicillin
d) Give Amoxicillin-Clavulanic
Old Patient with HTN & BPH. Rx is:
a) B-Blocker
b) Phentolamine
c) Zedosin
Sign of severe Hypokalemia is:
a) P-wave absence
b) Peak T-wave
c) Wide QRS Complex
d) Seizure
Patient with AMI and Multiple PVC, what is your treatment for this arrhythmia?
a) Amiadrone (Amiodarone)
b) No Rx
Causes of secondary Hyperlipidemia, all except:
a) HTN
b) Nephrotic Syndrome
c) Hypothyroidism
d) Obesity
70year old lady on …, feels dizzy on standing, resolves after 10-15minutes on sitting,
decrease on standing, decrease on standing, most likely she is having:
a) Orthostatic hypotension
The effectiveness of ventilation during CPR is measured by:
a) Chest Rise
b) Pulse Oximeter
c) Pulse Acceleration
Cardiac syncope:
a) Gradual onset
b) Fast recovery
c) Neurological sequence after
Young patient with HTN came complaining of high blood pressure and red, tender,
swollen big left toe, tender swollen foot and tender whole left leg. Diagnosis:
a) Cellulitis
b) Vasculitis
c) Gout Arthritis
Patient with hypertrophic subaortic stenosis referred from dentist before doing
dental procedure. What is true?
a) 50% risk of endocarditis up to my knowledge
b) 12% risk of endocarditis
c) No need for prophylaxis
d) Post Procedure antibiotic is enough
Female, narrow QRS, contraindication of adenosine:
a) LHF
b) Mitral
c) Renal
31year old, autopsy shows bulky vegetations on aortic and mitral valves. What is the
diagnosis?
a) Infective endocarditis
b) Rh endocarditis
Patient on Digoxin started to visualize bright lights and other signs of visual
disturbances. What caused this?
a) Digoxin toxicity
How does the heart get more blood?
a) Increasing blood pressure
b) Increasing Heart rate
c) Increasing SV
The best way of Rx patient with BP of 130-139/80-85 is:
a) Weight reduction and physical activity
b) Exercise alone
Family Hx of CAD eaten fruit 4 veget 4 bread 8 meat 3 dairy 4 what to do:
a) Decrease meat and dairy
Premature ventricular contraction is due to:
a) Decrease O2 requirement by the heart
b) Decrease blood supply to the heart
c) Decrease O2 delivery to the heart
Male patient who is a known case of hypercholesterolemia, BMI: 31, investigations
show high total Cholesterol, high LDL & high TG. Of these, which is the most
important risk factor for developing coronary artery disease?
a) Elevated LDL
b) Elevated HDL
c) Low HDL
d) Elevated Cholesterol
e) Elevated TG level
Patient was brought by his son. He was pulseless and ECG showed Ventricular
tachycardia, BP=80/, what is your action?
a) 3 set shock
b) One D/C shock
c) Amiodarone
d) CPR
One of the ff is a xtic of syncope (vasovagal attack):
a) Rapid recovery
b) Abrupt onset
c) When turn neck to side
d) Bradycardia
e) Neurological deficit
Which of the ff indicate inferior wall MI (inferior chest leads) in ECG?
a) II, III, AVF
b) V1, V2, V3
c) V2, V3, V4
d) I, V6
e) I, aVL, V1
Patient who is a known case of posterior MI who presented with syncope.
Examination showed canon (a) wave with tachycardia, unreadable BP & wide QRS
Cplxes on ECG. Diagnosis:
a) Atrioventricular re-entrant nodal tachycardia
b) Ventricular tachycardia
c) Pre-existing AV Block
d) Anterograde AV Block
e) Bundle Branch Block
Warfarin is given to all except:
a) Young male with Atrial Fibrillation (AF) and mitral stenosis
b) Male with AF and COM
c) Male with AF & Prosthetic heart valve
d) Elderly male with normal heart
Angina with a decreased ST 1-2 cm < 5min. What is the diagnosis?
a) Ischemia-heart block
15year old male complaining of joint pain and fever for 1 week, difficulty swallowing,
liver 1cm below costal and pan-systolic murmur
a) RHD
b) Infective Endocarditis
What will increase heart blood flow when there is increased load on the heart?
a) Dilatation of the Coronary
b) Constriction of the aorta
c) Increased HR
d) Increased Venous return
Pregnant with History of DVT 4 years back, what will U give her?
a) Aspirin
b) Clopidogrel
c) LMW Heparin
Patient comes with precordial pain, ECG ST segment elevation, patient given aspirin
and nitrate, but no relief of pain. What next step will U do?
a) Give IV Morphine
Most common cause of chronic hypertension:
a) DM
b) HTN
c) Interstitial Renal Disease
All are true about the best position in hearing murmurs EXCEPT:
a) Supine: Venous hum
b) Sitting: AR
c) Sitting: Pericardial Rub
d) Supine: Innocent Ouflow Obstruction
e) Left lateral in: MS
The most risk of anti-HTNsive drugs on elderly patients?
a) Hypotension
b) Hypokalemia
c) CNS side effect
About Ventricular fibrillation:
a) Can only be Rx with Synchronized Defibrillation
b) Waves are similar in shape, size and pattern
c) Course VFs indicates new VF and can be treated with ??
60Year Old Male presented with Hx of 2hours chest pain, ECG showed ST elevation
on V1-V4 with multiple PVC and Ventricular tachycardia. Mgt is:
a) Digoxin
b) Lidocaine
c) Plavix & Morphine
d) Amiodarone
Diastolic blowing murmur best (to) heard in the left sternal border increasing with
squatting
a) AS
b) AR
c) MS
d) MR
e) MVP
Female patient with moderate AS has syncope in the gym while she was doing
exercise. If the syncope was due to AS, what is the cause?
a) Systemic Hypotension
b) Cardiac arrhythmias
Which of the ff is the least likely to cause Infective Endocarditis?
a) ASD
b) VSD
c) Tetralogy of Fallot
d) PDA
Patient Presented in ER with low BP, Distended Jugular veins, muffled heart sounds
and bruises over the sternal area. What is the diagnosis?
a) Cardiac tamponade
35year old woman presented with exertional dyspnea. Precordial exam revealed
loud S1 and rumbling mid-diastolic murmur at apex. Possible complications of this
condition can be all of the ff EXCEPT:
a) Atrial Fibrillation
b) Systemic embolization
c) Left Venricular Failure
d) Pulmonary Edema
e) Pulmonary HTN
S3 occurs in all of the ff EXCEPT:
a) Tricuspid regurgitation
b) Young athlete
c) LV Failure
d) Mitral stenosis
Rx of chronic atrial fibrillation include all EXCEPT:
a) Cardioversion
b) Digoxin
c) Warfarin
Rx of Unstable angina include all EXCEPT:
a) Heparin
b) Nitroglycerin
c) B-Blocker
d) Aspirin
e) All those answers are true
The ff are features of Rheumatic Fever except:
a) Restless, involuntary abnormal movement
b) Subcutaneous nodules
c) Rashes over the trunk and extremities
d) Short PR interval on ECG
e) Migratory arthritis
The cause of death in Ludwig angina?
a) Dysrhythmia
b) Asphyxia
c) Pneumonia
d) Wall Rupture
Nitroglycerine causes all of the ff except:
a) Increase coronary blood flow
b) Methemoglobinemia
c) Venous pooling of blood
d) Efficient for 5min if taken sublingual
e) Lowers arterial blood pressure
In atrial fibrillation and stroke, all are true except:
a) Aspirin can be given in AF for prevention of stroke
b) Warfarin can be given in AF for prevention of stroke
c) Non-Valvular AF can cause stroke
Sinus tachycardia and atrial flutter, how to differentiate?
a) Temporal aa massage
b) Carotid aa massage
c) Adenosine IV
Young patient came to the ER with dyspnea and productive tinged blood frothy
sputum. He is a known case of Rheumatic HD, AF and his cheeks has dusky rash.
Diagnosis?
a) Mitral stenosis
b) CHF
c) Endocarditis
Patient with sudden cardiac arrest, ECG showed no electrical activity with oscillation
of QRS with different shapes. Underlying process is
a) Atrial dysfunction
b) Ventricular dysfunction
c) Toxic ingestion
d) Metabolic
Best treatment for female with migraine and HTN is
a) Propanolol
20 yr old patient with palpitations. ECG shows narrow QRS Cplxes and pulse is
300bpm. What is true?
a) Amiodarone should be included in the mgt
How coronary aa disease causes MI?
a) Narrowing of the blood vessel
CCBs such as Nifedipine, Verapamil and Diltiazem are extremely useful in all the ff
except:
a) Prinzmetal angina pectoralis
b) HTN
c) Atrial tachycardia
d) Ventricular tachycardia
e) Effort angina Pectoralis
Old man with stable angina. All the ff are correct except:
a) Angina will last less han 10 mins
b) Occur on exertion
c) No enzymes will be elevated
d) Will be associated with loss of consciousness
70yr old male brought to the ER with sudden onset pain in left lower limb. The pain
was severe with numbness. Had acute MI 2 weeks previously and was discharged
24hrs prior to his presentation. Left leg was cold and pale. Right leg was normal.
Most likely diagnosis:
a) Acute aa thrombosis
b) Acute aa embolus
c) DVT
d) Ruptured disc at L4-5 with radiating pain
e) Dissecting thoraco-abdominal
Coarctation of the aorta commonly associated with which syndrome:
a) Down
b) Turner
c) Patau
d) Edward
e) Holt-Orain
What is your next step before an operation to a child with continuous murmur in his
right sternal area?
a) Postpone and reevaluate the patient again
Each of the ff murmur will be elicited by change of position except:
a) Innocent murmur by sitting
Patient post MI with hemiparesis and drowsy. First thing to do:
a) Heparin
A known case of Coronary artery disease with symptoms of it to diagnose that
patient has MI or not, by first ECG and cardiac enzyme:
a) Exercise stress test
b) Coronary angiography
c) Exercise
Patient with carotid aa obstruction by 80%. Rx is by
a) Carotid endarterectomy
b) Surgical bypass
Old male comes with CHF & Pulmonary oedema. Best Initial therapy:
a) Digoxin
b) Furosemide
c) Dobutamine
HYPerkalemia characterised by all of the ff except:
a) Nausea & Vomiting
b) Peaked T-waves
c) Widened QRS cplx
d) Positive Chvostek Sign
e) Cardiac arrest in diastole
10yr old had an episode of rheumatic fever without any defect to the heart. Patient
need to take antibiotic prophylaxis for
a) 5months
b) 6years
c) 15years
Antibiotic prophylaxis for endocarditis is:
a) 2g amoxicillin one hr before procedure
b) 1g amoxicillin after procedure
c) 2g clindamycin 1h before procedure
d) 1g clindamycin after procedure
Patient with hypercholesterolemia should avoid
a) Organ meat
b) Avocado
c) Chicken
Difference btw stable and unstable angina:
a) Necrosis of heart muscle
b) Appears to be independent of activity; pathophysiology of atherosclerosis
Drug contradiction for Hypertrophic cardiomyopathy:
a) Digoxin
b) B-Blocker
c) Alpha Blocker
Fick method in determining Cardiac output:
a) BP
b) O2 saturation in the blood
Which enzyme for a man who has had MI:
a) CPK
b) ALP
c) AST
d) Amylase
Which medication decreases mortality in CHF?
a) Furosemide
b) Digoxin
c) ACEI
Murmur of mitral stenosis is:
a) Holo-systolic
b) Mid-systolic
c) Mid-diastolic rumbling murmur
What is correct about Unstable angina:
a) Same drug that is used in stable angina
b) Should be treated seriously as it might lead to MI
Patient with hx of AF+MI. Best prevention for stroke is:
a) Warfarin
b) Surgery
c) Shunt
Which of these is the most common condition associated with endocarditis?
a) VSD
b) ASD
c) PDA
d) TOF
Patient on Lisinopril complaining of cough. What’s the drug with the same action
without the same side effect?
a) Losartan
RBBB:
a) Long S in lead 1 and V6 & long R in V1
b) Long S in lead V1 and long R in V6
Decreases mortality in CHF:
a) B-blocker
b) Verapamil
c) Nitrates
d) Digoxin
Stable angina for 2 years, came C/o palpitation; Holtis monitor showed 1.2mm ST
depression for 1 to 2 minutes in 5-10 minutes. Your Dx?
a) Myocardial ischaemia
b) Sinus erythema
c) Normal variant
Presented to the ER with substernal chest pain, 3 months he ago had complete
physical exam and was normal; ECG normal, only high LDL for which he started low
fat diet and medication for. What is the factor the doctor will take into consideration
as a risk factor?
a) Previous normal ECG
b) Previous normal physical exam
c) Previous LDL level
d) Current LDL level
e) Current symptom
Carpenter 72 years old (lost one family member to heart attack) came to you to do
some investigations. He is well and fit. He Denied any hx of chest pain or SOB. o/e,
everything is normal except mid systolic ejection murmur at left sternal area without
radiation to the carotid. What is your diagnosis?
a) Aortic stenosis
b) Aortic sclerosis
c) Flow murmur
d) Hypertrophic subaortic stenosis
Patient came with chest pain, radiates to the Jaw, increases with exercise, decreases
with rest. What is the diagnosis?
a) Unstable angina
b) Stable angina
c) Prinzmetal angina
Patient with sudden SOB had posterior inferior MI. What’s the cause?
a) Pulmonary embolism
b) Acute MR
c) Acute AS
d) Arrhythmia
Increases survival rate in Heart failure
a) Enalapril
b) Isosordil
c) Furosemide
d) Spironolactone
Causes of Bundle Branch Block
a) Aortic stenosis (causes LBBB)
b) Pulm Stenosis (RBBB)
c) Mitral
d) Cardiomyopathy (LBBB)
Patient with risk factor for developing infective endocarditis will undergo a urology
surgery. He is sensitive for penicillin. What would U give him?
a) IV Vancomycin + IV Gentamicin
b) Oral tetracycline
c) No need to give
Patient had rheumatic episode in the past. He developed mitral stenosis with orifice
less than … (severe stenosis). This will lead to
a) Left Atrial hypertrophy & Dilatation
b) Left atrial dilatation and decreased pulmonary wedge pressure
c) Right atrial hypertrophy and decreased pulmonary wedge pressure
d) Right atrial hypertrophy and chamber constriction
Elderly patient presented with SOB, rales in auscultation, Orthopnea, PND, exertional
dyspnoea. The main pathophysiology?
a) Left Ventricular dilatation
b) Right ventricular dilatation
c) Aortic regurgitation
d) Tricuspid regurgitation
Patient with BP of 180/140, you want to lower the Diastolic (which is true):
a) 110-100 in 12 hours
b) 110-100 in 1-2 days
c) 90-80 in 12hrs
d) 90-80 in 1-2days
Unstable angina:
a) Least Grade II and new onset less than 2 months ago
b) Usually there is evidence of Myocardial ischaemia
c) Same Rx as stable angina
d) Discharge when chest pain subsides
Patient Post MI 5 weeks, complaining of chest pain, fever and arthralgia:
a) Dressler’s syndrome
b) Meigs syndrome
c) Costochondritis
d) MI
e) PE
Patient with chest pain. Xray revealed pleural effusion, high protein and high LDL:
a) TB
b) CHF
c) Hypothyroidism
d) Hypoproteinemia
Drugs used in systolic dysfunction heart failure:
a) Nifedipine
b) Diltiazem
c) ACEI
d) B-Blocker
Elderly patient, known case of AF came with abdominal pain and bloody stool. What
is the diagnosis?
a) Ischemic mesentery
Patient having chest pain radiating to the back, decrease blood pressure in left arm
and absent left femoral pulse with left sided pleural effusion on CXR, LEFT
VENTRICULAR hypertrophy on ECG, most proper investigation is:
a) Aortic angiogram
b) Amylase level
c) CBC
d) Echo
60yr old patient has only HTN. Best drug to start with:
a) ACEI
b) ARB
c) Diuretics
d) B-blocker
e) Alpha Blocker
Patient is a known case of Coronary Artery Disease. Presents with symptoms of it,
to diagnose that patient has MI or not, first ECG and cardiac enzymes then:
a) Exercise stress test
b) Coronary angiography
c) Exercise
Patient after 2months POST MI cannot sleep. What to give him?
a) Zolpidem
b) Diazepam
Obese, HTN, Cardiac patient with hyperlipidemia, sedentary lifestyle and unhealthy
food. What are the 3 most correctable risk factors?
a) HTN, Obesity, Low HDL
b) High TAG, Unhealthy food, sedentary life
c) High Cholesterol, unhealthy food, sedentary life
d) High Cholesterol, HTN, Obesity
15years old with palpitation and fatigue. Investigations showed right ventricular
hypertrophy, right ventricular overload and right branch block. What is the
diagnosis?
a) ASD
b) VSD
c) Coarctation of the aorta
HTNsive patient on diuretics developed painful big toe. What kind of drug?
a) Hydrochlorothiazide
b) Furosemide
What best explains Coronary Artery Disease?
a) No atherosclerosis
b) Fatty deposition with widening of aa
c) Atherosclerosis with widening of aa
Old patient she has MI and complicated with ventricular tachycardia then from that
time receive Buspirone. She came with fatigue, normotensive and pulse was 65.
What investigation must be done?
a) Thyroid function
b) Liver and thyroid
Patient has AF, risks:
a) CVA
b) MI
Case of Pericarditis:
a) Pain in chest increase with movement
b) Best investigation is ECG
c) Best investigation is Cardiac enzymes
Patient complains of MI, on treatment; after 5 days, he has shortness of breath +
crepitations on both lungs.
a) Pulmonary embolism
b) Pneumonia
c) MR
d) AR
High pitch diastolic murmur:
a) MS
b) MR
c) MVP
Came to the ER with AF, BP of 80/60. What is the mgt?
a) Synchronized CD
b) Digoxin
Long Scenario of MI, what is the inappropriate mgt?
a) IV Ca++ Channel Blocker
b) Nitrate
c) IV Morphine
d) B-Blocker
Patient presented with chest pain of 2 hours with anterolateral lead showing ST
elevation. Providing PCI in hospital mgt:
a) Streptokinase, nitroglycerine, ASA & B-blocker
b) Nitroglycerine, ASA, Heparin, B-Blocker
c) Alteplase, Nitroglycerine, heparin, B-Blocker
Which of the ff is a MINOR criterion for rheumatic fever?
a) Arthritis
b) Erythema marginatum
c) Chorea
d) Fever
A teacher with aortic stenosis fainted while in class. What is the cause?
a) Cardiac syncope
b) Hypotension
c) Neurogenic syncope
A case of CHF, loved to eat outdoor 2-3 times weekly. Your advice to him:
a) Eat without any salt
b) Eat 4gm of salt
c) Low fat, high protein
Picture of JVP graph. Patient had low volume pulse, low resting BP, no murmur and
pedal edema.
a) Constrictive pericarditis
b) Tricuspid regurg
c) Tricuspid stenosis
d) Pulm HTN
46year old male came to the ER with abdominal pain (not severe). He has
hyperlipidemia, smoking, HTN. Not following his medications very well. Vitals are
stable. o/e tall obese patient, mid line abdominal tenderness. Dx:
a) Marfan syndrome
b) Aortic aneurysm
Old patient with tachycardia, pulse 150, otherwise normal
a) TSH
b) Stress ECG
One non-pharmacological is the most appropriate in HTN:
a) Weight loss
b) Decrease alcohol
c) Stop smoking
Female patient known case of rheumatic heart disease, diastolic murmur, complains
of aphasia and hemiplegia. What will You do to find the aetiology of this stroke?
a) MR angiography
b) Non-Contrast CT
c) Echo
d) ECG
e) Carotid Doppler
Normal child, he wants to walk. He had a brother who died after walking. Which of
the following must be excluded before walking?
a) PDA
b) VSD
c) Hypertrophic COM
A component of TOF:
a) ASD
b) VSD
c) LVH
d) Aortic stenosis
e) Tricuspid stenosis
Patient with anterior MI + premature ventricular ectopics that indicate pulmonary
edema. Gave Digoxin + Diuretics + Afterload reducer. What else would U add?
a) Amiodarone
b) Propranolol
Patient with rheumatic valvular disease, mitral orifice is 1cm. What is the action to
compensate that?
a) Dilatation in the atrium with chamber hypertrophy
b) Dilatation in the ventricle with chamber hypertrophy
Very long scenario about mitral stenosis, the surface area of the valve was 0.7cm2.
What is the Rx?
a) Medical Rx
b) Percutaneous mitral valvuloplasty
c) Mitral valve replacement
All can cause secondary hyperlipidemia except:
a) Hypothyroidism
b) Alcoholism
c) Nephrotic syndrome
d) Hypertension
Prolongs QT interval:
a) Chlorpromazine
b) Clozapine
c) Haloperidol
d) Ziprasidone
How can Group A Beta Streptococci cause Rheumatic HD?
a) When they cause tonsillitis/pharyngitis
b) Via bloodstream
c) Through skin infection
d) Invasion of the myocardium
Pansystolic machinery murmur at left sternal border:
a) Aortic stenosis
b) Mitral stenosis
c) PDA
d) MR
Miscellaneous questions

1. Which of the following is treatment for giardiasis?

A. Praziquantel

B. Mebedazole

C. Metronidazole ( correct answer)

D. Albendazole.

2. Patient with epilepsy came with left shoulder pain, on examination flattened contour of the
shoulder, and fixed adduction with internal rotation, what is the diagnosis?

A. Inferior dislocation

B. Subacromal posterior dislocation ( C)

C. Subglenoid anterior dislocation

D. Subclavicle anterior dislocation

E. Subclavicle anterior dislocation

3. 12 years old female brought by her mother to ER after ingestion of unknown number of
Paracetamol tablets. Clinically she is stable. Blood Paracetamol levels suggests toxicity. The most
appropriate treatment
A. N- acetylcestine

4. All the following are side effects of fur as oxide except:

A. Hyperkalemia (C)

B. Hypoglycaemia

C. Bronchospasm

D. Hemolytic anaemia

E. Pre-renal azotemia

5. Patient with right arm tenderness with red streak line, the axillary lymph node is palpable :

A. Cellulitis

B. Carcinoma

C. Lymphangitis

6. Patient with central line became septic, what organisms

A. GBS

B. Neisseria

C. Pseudomonas

D. E.coli

7. Best way to prevent Entamoeba histolytica is

A. Boiling

8. Prevention of Lyme disease, what is the best advice to parents?

A. Insect and tick removal.

9. Prevention of Kyme disease:

A. Treat early disease with doxycycline, prevent with tick bite avoidance

10. Parents asking about Lyme disease for their children, practitioner is most correct to tell them (
for prevention)

A. Kill vector (C)

B. Clothes of natural fibers

C. Antibacterial soap

11. Drug of choice for a schistosomiasis is:

A. Praziquantel as single oral dose annually (C)

B. Oxaminiquine
C. Arthemeter

12. In flame burn, the most common cause of immediate death :

A. Hypovolemic shock

B. Septic shock

C. Anaemia and hypoalbumin

D. Smoke inhalation (C)

13. Patient present with submandibular swelling with eating, relieved after eating, Dx:

A. Submandibular gland stone

14. Long scenario of restless leg syndrome( he didn't mention Dx in scenario) , 85 old male many
times awake from his sleep because of leg pain, this pain relieved by just moving his foot, but it
recurs, best management :

A. Colazpin

B. Haloperidol

C. Lorazepam

D. One drug from dopamine agonist group, forgot it's name, it's the right answer.

15. Best drug for Von Will errand disease is:

A. Fresh frozen plasma

B. Cryoprecipitate

C. Steroids

16. Which of th following is a feature of iron deficiency anaemia?

A. Low MCH

17. Patient just received organ transplantation, what is the sign of acute rejection?

A. Fever (c)

B. Hypotension

18. Sodium amount in Normal Saline (0.9% NaCl):

A. 75 mmol

B. 90 mmol

C. 154 mmol ( c)

D. 200 mmol

19. Young drug user, asymptomatic. What to investigate?

A. HIV, HIV, St. Viredans

20. Anaemia of chronic disease


A. Decrease iron and increase TIBC in iron deficiency

B. Increase iron and Increase TIBC

C. Increase iron and decrease TIBC( C)

21. Man with polycythemia Vera cam with bruising. What causes decrease blood flow?

A. Hyperviscosity ( C)

B. Hypoxia

C. Hyperviscosity

22. Patient with polycythemia Vera, the cause of bleeding in this patient is

A. Increase viscosity

B. Low platelets

23. What is the major thing that can tell you that patient have polycythemia Vera rather than
secondary polycythemia :

A. Hepatomegaly

B. Splenomegaly (C)

C. Venous engorgement

D. Hypertension

24. Group A Haemolytic streptococcus, causes rheumatic fever when :

A. Invade blood stream

B. Invade myocardium

C. After tonsillitis and pharyngitis (C)

D. Skin infection

25. Man came with bruising an increase time of bleeding with factor 8 deficiency:

A. Haemophilia A

B. Von willebrand disease

26. An old man 65 years with HB= 9, you will

A. Assess iron levels

B. Assess LDH

C. Arrange for endoscopy (C)

27. In Aspirin overdose :

A. Liver enzyme will peak within 3- 4 hr

B. First signs include peripheral neuropathy and loss of reflexes


C. 150mg/kg of Aspirin will not result in aspirin toxicity (C)

28. Man who is having severe vomiting and diarrhoea and now developed leg cramps after receiving
3 liters of dextrose, he is having

A. Hypokalemia (C)

B. Hyponatremia

C. Hyperkalemia

D. Hypetnatremia

29. Man who received blood transfusion back in 1975 developed jaundice most likely has:

A. Hepatitis a

B. Hepatitis C

C. Hepatitis D

D. Hepatitis E

30. Best method to prevent plague is :

A. Hand wash

B. Kill rodent (C)

C. Spray pesticide

D. Give prophylactic AB

31. Ibuprofen is contraindicated if patient has

A. Peptic ulcer (C)

B. Seizures

C. RA

32. Patient come to the ER with constricted pupil and respiratory compromise, you will suspect

A. Opiates like morphine (C)

B. Cocaine

C. Ecstasy

33. The best to give for DVT patients initially which is cost effective :

A. Low molecular weight heparin ie Enoch paring (C)

B. Unfractionated heparin

C. Heparin

D. Warfarin

34. Management of somatization:


A. Multiple phone call

B. Multiple clinic appointments (C)

C. Refer to pain clinic

D. Antidepressant

35. A lot of bacteria produce toxins which are harmful. Which one of the following is useful?

A. Botulism (C)

B. Tetanus

C. Diphtheria

D. Staph. Aureus

36. Organphosphate poisoning, what is the antidote.

A. Atropine (c)

B. Physostigmine

C. Neostigmine

D. Pilocarpine

E. Endrophonium

37. Patient using haloperidol, developed rigidity( dystonia) treatment :

A. Antihistamine and anticholinergic

38. High risk factor in CLL

A. Age(C)

B. Smoking

C. History of breast Ca

D. History o radiation

39. 60 year old male was referred to you after stabilisation, investigation wa Hb 8.5 g/l, Hot 64%,
Rbc 7.8, Wbc 15.3 and platelet 570, Diagnosis :

A. Iron deficiency anaemia

B. Hemoglobinopathy

C. CLL(C)

D. 2ry polycythemia

40. 24 years old patient. Came for check up after a promiscuous relation 1 month ago, he was
clinically unremarkable, VDRL:1/128. He was allergic to penicillin, other line of management is:

A. Ampicillin

B. Amoxicillin
C. Trimethoprim

D. Doxycycline (C)

41. Cellulitis in children, most common cause;

A. Group A Streptococcus

B. Staphylococcus aureus(c)

42. Patient with Hodgkin's lymphoma and Reed Stenberg cell in pathology and there is eosinophil,
lymphocyte in blood so pathological classification is:

A. Mixed cellularity subtype

B. Nodular sclerosis subtype of Hodgkin's lymphoma (C)

43. In IV cannula and fluid:

A. Site of entry of cannula is a common site of infection

44. Therapeutic range of INR ( in presence of anticoagulant)

A. 2.5-3.5

B. 2.0- 3.0. But normal range in absence of anticoagulant is 0.8-1.2 (C)

45. Patient had arthritis in two large joints and pansystolic murmur 'carditis'. Hx of URTI, the most
important next

A. ESR

B. ASO titre (C)

C. Blood culture

46. Patient with gunshot and part of his bowel spilled out and you decide to give him antibiotic for
Bacteriod fragilis,so what will you give?

A. Amoxicillin

B. Clindamycin(C)

C. Erythromycin

D. Doxycycline

E. Gentamicin

47. Treatment of peritonitis the organism is Bacteroid fragilis

A. Clindamycin

B. Metronidazole

C. Carbapenem(C)

48. Patient with high output fistula, for which TPN wa ordered, after 2 hours of the central venous
catherization, the patient became comatose and unresponsive. What is the most likely cause.
A. Septic shock

B. Electrolyte imbalance (C)

C. Delayed response of blood mismatch

D. Hypoglycaemia

E. Hypernatremia

49. What is the most sensitive indicator for factitious fever?

A. Pulse rate

50. All the following tests are necessary to be done before initiating lithium except?

A. Liver function tests.

51. Healthy patient with family history of DM type 2, the most factors that increase chance of DM
are:

A. HTN an Obesity (C)

B. Smoking and Obesity

C. Pregnancy and HTN

D. Pregnancy and Smoking

52. In diabetic retinopathy, most related factors:

A. HTN and Obesity (C)

B. HTN and smoking

C. Smoking and Obesity

53. Patient with blood group A had blood transfusion group B, the best statement that describe the
result is

A. Type IV hypersensitivity

B. Inflammatory reaction

C. Type 2 hypersensitivity (C)

54. Besides IV fluids, what is the most important drug to the given in anaphylaxis?

A. Epinephrine

B. Steroids

55. Management of anaphylactic shock, all of the following except:

A. IVF

B. 100% O2

C. Corticosteroids (C)
56. Patient developed lightheadness and SOB after bee sting. You should treat him with the
following :

A. Epinephrine injection, antihistamine, a IV fluid(C)

B. Antihistamine

57. A child had bee bite presented after 18 hour with left arm erythema and itching, what to do?

58. In a patient with anaphylactic shock, all are correct treatments except:

A. Epinephrine

B. Hydralazine (C)

C. Adrenaline

D. Aminophylline

59. Most common symptoms of soft tissue sarcoma:

A. Paralysis

B. On growing mass(C)

C. Pain

60. All following are criteria of chronic fatigue syndrome except

A. More than 6 month, muscle pain, and joint pain

B. Persistent, idiopathic, headache

C. Not relieved by rest + poor cognition

61. Regarding chronic fatigue syndrome, which is true?

A. Antibiotics may reduce the symptoms

B. Antidepressants may reduce the symptoms (c)

C. Rest may reduce th symptoms

62. Burn patient is treated with silver sulfadiazine, the toxicity of this drug can cause:

A. Leucocytosis

B. Neutropenia(C)

C. Electrolyte disbalance

D. Hypokalemia

63. Patient complaining of hypotension and bradycardia. Electrolytes show ( gibberish) . So the cause
is:

A. Hyponatremia (c)

B. Hyperkalemia

C. Hyperchloremia
D. Uremia.

64. The most common complication of mumps in adults :

A. Labyrinthitis ( 0.005% of cases)

B. Orchitis(30% of cases) (C)

C. Meningitis (10% of cases)

D. Encephalitis (less than 1 % of cases)

65. Adolescent female counseling on fast food. What you should give her?

A. Calcium and folic acid (C)

B. Vitamin C and folic acid

C. Zinc and folic acid

D. Zinc and vitamin C

66. 17 years old boy admit to involve in recurrent illegal drug injection, what is the screening test to
do?

A. HIV

B. Hepatitis B

C. Hepatitis C(C)

67. Patient alcohol drinker complains of headache, dilated pupil, hyperactivity, agitation. He had
history of alcohol withdrawal last week so treatment is ;

A. Diazepam( Valium) (C)

B. Naxtrol

C. Haloperidol

68. Patient present with high blood pressure ( systolic 200), tachycardia, mydriasis " dilated pupils",
sweating. What is toxicity?

A. Anticholinergic

B. Sympathomimetic drug (C)

C. Tricyclic antidepressant

D. Organophosphorus compunds

69. Patient with gonorrhoea infection. What else you want to check for?

A. Chlamydia trachomatis

70. Patient known case of SCA, the doctor planning to give him pneumococcal vaccine, which one is
true?

A. Patient need antibiotic when there is history of contact even with vaccine
71. Long scenario for patient care to ER after RTA, splenic rupture was clear, accurate sentences
describe long term management :

A. We give pneumococcal vaccine for high risk people

B. We should give ABs prophylaxis if there is history of contact even with vaccination against
pneumococcus(C)

C. Pneumococcal vaccine should not be given at same time with MMR

72. Sickling patient after acute attack, discharge on

A. Penicillin (C)

B. Iron

C. Vitamin

73. Man present with painless ulcer in his penis with indurate base and everted edge so diagnosis is:

A. Syphilis ( painless) (C)

B. Gonorrhoea

C. Chancroid ( painful)

D. HSV

74. Clonidine( A2 - agonist) decrease the effect of :

A. Benzotropin

B. Levodopa(C)

C. Rubstin

D. Amitriptyline

75. Which of the following drug cause hypertensive crisis?

A. Clonidine

76. Clonidine( A2 - agonist) decrease the effect of :

A. Benzotropin

B. Levodopa(C)

C. Rubstin

D. Amitriptyline

77. Which of the following is not transmitted by mosquitoes?

A. Rift Valley fever

B. Yellow fever

C. Relapsing fever (C)

D. Filariasis
E. Dengue fever

78. Male patient gave a history of left knee swelling and pain 5 days, two days back he had right
wrist swelling and redness. He had recently traveled to India. On examination, there was tenderness
and limitation of movement. 50 cc of fluid was aspirated from the knee. Gram stain showed gram
negative diplococcoi. What is the most likely organism?

A. Brucella militans

B. Neisseria gonorrhoea (C)

C. Staph aureus

D. Strep pneumoniae

E. Strep pyogenes

79. 12 years old girl with malaise, fatigue, sore throat and fever. On examination, there were
petechial rash on the palate, large tonsils with follicles, cervical lymphadenopathy and
hepatosplenomegaly. All are complications except:

A. Aplastic anemia

B. Encephalitis

C. Transverse myelitis

D.splenic rupture

E. Chronic active hepatitis (C)

80. Patient has EBV, during abdomen exam, because pale with tender LUQ:

A. IVF

B. Urgent C (C)

C. Rush him to OR

81. Treatment of EBV ( in scenario there patient with tonsillar exudates, lymphadenopathy,
splenomegaly)

A. Oral acyclovir

B. Oral antibiotic

C. IM or IV acyclovir

D. Supportive TTT( C)

E. Observation

82. 20 years old man involved in RTA brought to ER by his friends. In examination, found to be
conscious but drowsy. HR 120/min, BP 80/40. The most urgent initial management measure is:

A. CT brain

B. X- Ray cervical spine


C. Rapid infusion of crystalloids (C)

D. ECG to exclude haemopericardium

E. US abdomen

83. Normal daily caloric intake is:

A. 0.3 kcal/kg

B. 1.3 kcal /kg

C. 2.0kcal/kg

D. 3.5kcal/kg

E. 35kcal/kg (c)

84. The following can be used as prophylaxis for Malaria in chloroquine resistant areas except:

A. Mefloquine

B. Doxycycline

C. Chloroquine with proguanil

D. Pyrimethamine

E. Days one (C)

85. Malaria case, beside antibiotic how to prevent?

A. Kill the vector

86. Patient with malaria in outbreak, what is the common way to prevent?

A. Vector eradication and avoid mosquito bites (C)

B. Kill the vector and spray your clothes

C. Avoid and spray ( gibberish)

87. Giemsa stained blood film:

A. Malaria

88. Patient with history of fever, peripheral blood film +ve for Malaria:

A. Banana shaped erythrocyte is seen in P. Vivax

B. Mostly due to P. falciparum (C)

C. Treated immediately by primaquine 10mg for 3 days.

D. Response to Rx will take 72 hr to appear

89. Regarding protective measures of malaria,all true except :

A. Infection occur more in day than (C)

B. Using insect repellant is useful


C. Because no antimalarial is 100% effective, avoiding exposure to mosquitoes in endemic areas is
essential

D. Female anopheles mosquito feeds primarily from dusk until dawn, travelers can reduce their risk
of malaria by limiting evening outdoor activities

E. Using permethrin- treated clothing in conjunction with applying a topical DEET repellant to
exposed skin gives nearly 100% protection

F. Sleep in an air- conditioned or well- screened room under mosquito nets

90. Malaria in a child

A. Crescent shaped gametocyte of Vivax is diagnostic in the stool

B. The immediate TTT primaquine for 3 d

C. 72 hours treatment of malaria is sufficient

D. The most common cause is falciparum (c)

91. The most important factor in the development of spinal headaches after spinal anaesthesia is :

A. The level of the anaesthesia

B. The gauge of the needle used (C)

C. The closing pressure after the injection of tetracaine

D. The occurrence in the elderly

E. The selection of male patients

92. Which of the following would most likely indicate a hemolytic transfusion reaction in an
anaesthetised patient ?

A. Shaking chills and muscle spasm

B. Fever and oliguria

C. Hyperpyrexia and hypotension

D. Tachycardia and cyanosis

E. Bleeding and hypotension (C)

93. In a gram negative bacterial septicaemia :

A. Pseudomonas is the most common organism involved

B. Many of the adverse changes can be accounted for by endotoxin (C)

C. The cardiac index is low

D. Central venous pressure is high

E. Endotoxin is mainly a long-chain peptide

94. In septic shock :


A. The mortality rate is 10 to 20%

B. Gram- negative organisms are involved exclusively

C. The majority of patients are elderly(C)

D. The most common source of infection is alimentary tract.

E. Two or more organisms are responsible in the majority of cases.

95. 40 years old white male is transferred to your institution in septic shock less than 24 hours after
onset of symptoms of a non- specific illness. He underwent a splenectomy for trauma 5 years ago.
Antibiotic coverage must be direct against:

A. Streptococcus, group A

B. Klebsiella pneumoniae

C. Staphylococcus aureus.

D. Escherichia coli

E. Streptococcus pneumoniae (C)

96. Splenectomy does not have a role in the management of patients with haemolytic anemia due
to:

A. Spherocytosis

B. Elliptocytosis

C. Pyruvate kinase deficiency

D. Glucose -6- phosphate dehydrogenase deficiency (C)

E. Sickle cell anaemia

97. 23 years old white female is diagnosed as having chronic ITP. Which of the following will best
predict a favorable remission after splenectomy.

A. Presence of antiplatelet antibodies

B. Increased home marrow megakaryocytes

C. Absence of splenomegaly

D. Platelet count of 17000/mm3 on corticosteroids (C)

E. Complement on platelet surfaces

98. HSV type 1 infection of the oral cavity, all true except :

A. Is the commonest viral infection in the oral cavity

B. Can give gingivostomatitis

C. In primary infect, there is systemic involvement (C)

D. May present with tonsillitis without oral lesion


99. All true about cephalosporin use, except :

A. The most common side effect is allergy ( C)

B. There is a skin test for cephalosporin sensitivity

100. Gingivitis most likely cause :

A. HSV

101. All of the following drugs contraindicated in G6PD deficiency, except:

A. Aspirin(C)

B. Nitrofurantoin

C. Chloroquine

D. Sulphonamide

E. Gentamycin

102. All the following are side effects of thiazide diuretics except :

A. Has diabetogenic effect

B. Cause hypocalcemia(C)

C. Cause hypomagnesimia

D. Flat curve response

E. Cause hypokalemia

F. It causes hypercalcemia

103. Which of the following combination is safe?

A. Alcohol and metronidazole (C)

B. Digoxin and amiodarone

C. Warfarin and propranolol

D. Furosemide and gentamicin

104. All of the following cause photosensitivity except :

A. Lithium (C)

B. Propranolol

C. Tetracycline

D. Chlorpromazine

E. Chloropropamide

105. Hb electrophoresis done for a patient shows HbA1= 58%, HbS= 35%, HbA2=2 %, HbF=5%, Dx:

A. Thalassemia minor
B. Thalassemia major

C. Sickle cell trait (C)

D. Sickle cell anaemia

E. Sickle cell thal

106. All of the following are signs of allergy to local anesthesia, except :

A. Laryngeal spasm.

B. Urticaria

C. Low BP

D. Bronchospasm

107. Regarding urticaria, all true except :

A. May be due to drug ingestion

B. Not always caused by immune response

C. Could be a part of anaphylactic shock

D. Always due to deposition of immune complexes (C)

108. Where should we stop the OCP " Oral contraceptive pills"

A. In varicose veins

109. What is the osmolarity of NaCl?

A. 155 mmol

110. 55 years old male patient presented for check up, physical examination is normal, lab
investigation microcytic hypochromic anaemia, Hb=9, what is the most likely cause to exclude?

A. Lymphoma

B. Gastroenterology malignancy (C)

111. Side effect of steroid all except :

A. Pelvic muscle myopathy.

112. Patient who is a smoker, the least cancer he is predisposed to:

A. Urinary bladder cancer (high risk in smoker)

B. Colon cancer (C)

C. Lung cancer

D. Esophageal cancer

113. Patient gave history or malaise, fatigue and gave history of decrease meat in her diet. Hb is 9
and hypochromic microcytic anaemia, what will you give her?

A. Trial of iron therapy as she has iron deficiency anaemia (C)


B. Iron and multivitamin

114. 25y male presented with scrotal swelling notice before 1 day, no pain, tenderness or Urinary
symptoms. What is the management?

A. Referral to do US and consultation to surgery (C)

B. Referral to do biopsy.

115. Human bite to the brand, greatest risk of infection in which position :

A. Dependent

B. Clenched fist injury ( infection rate is higher than other types (C)

C. Finger extended

D. Extended thumb

E. Extended fingers

116. Cat bite predispose to skin infection by which organisms:

A. Staph

B. Strept

C. Pasteurella multocida (C)

117. Human bite :

A. Cleanse and debride as usual

B. Tetanus prophylaxis as indicated (c)

C. Antibiotic prophylaxis, Augmentin

118. A boy who was bitten by his brother and received tetanus shot 6 months ago and his laceration
was 1 cm and you cleaned his wound, next you will :

A. Give Augmentin (C)

B. Suture the wound

C. Give tetanus shot

D. Send home with close observation and return in 48 hours.

119. Most common cause of hand infection :

A. Trauma (C)

B. Immunocompromised

120. Diagnosis of thalassemia minor :

A. HbA2 and HbF by electrophoresis (C)

B. Microcytosis

121. One of the following combination of drugs should be avoided :


A. Cephaloridine and Paracetamol

B. Penicillin and probenecid

C. Digoxin and levodopa

D. Sulphamethomazole and trimethoprim

E. Tetracycline and aluminium hydroxide (C)

122. Aluminium salt will decrease the absorption of:

A. Tetracycline (C)

B. Penicillin

123. Aluminium hydroxide and magnesium hydroxide inhibits the intestinal absorption of which
drug?

A. Tetracycline (C)

B. Folic acid

125. What is the ration of ventilation to chest compression in a one person CPR?

A. 2 ventilation and 15 compression at rate of 80-100/ min (C)

B. 1 ventilation and 15 compression at rate of 80-100/ min

C. 2 ventilation and 7 compression at rate of 80-100/ min

D. 1 ventilation and 7 compression at rate of 80-100/ min

E. 3 ventilation and 15 compression at rate of 80-100/ min

126. When lactic acid accumulates, the body respond by :

A. Decrease production of bicarbonate

B. Excrete CO2 from the lungs (C)

C. Excrete chloride from the kidneys

D. Metabolise lactic acid in the liver

127. German measles ( Rubella)

A. Arthralgia

B. Arthritis (C)

128. Rubella infection, one is true :

A. Incubation period is 3-5 days

B. Oral ulcer

C. Arthritis

D. Does not cause heart complication for the fetue


129. Critical count of platelets which lead to spontaneous bleeding is :

A. 20000(C)

B. 50000

C. 75000

D. 100000

E. 200000

130. 43 years old man is brought to the emergency department after a motor vehicle accident
involving a head-on collision. He mentioned that he is having headache and dizziness. During his
overnight admission for observation, he developed polyuria and his serum sodium level rises to
151Meq/L. All of the following tests are indicated except :

A. Overnight dehydration test

B. Measurement of response to desmopressin

C. MRS scan of the head

D. Measurement of morning cortisol level (C)

E. Measurement of plasma and urine osmolality.

131. 26 years old man presented with headache and fatigue. Investigations revealed : Hb 8 g/ do,
MCV 85fl, reticulocyte 10%. All the following investigations are useful except:

A. Coombs test

B. Sickling test

C. Serum bilirubin

D. Serum iron (C)

E. Hb electrophoresis

132. Serum ferritin reflects

A. Total iron stores (C)

B. Serum iron

C. Bone marrow iron.

D. None of the above

133. To differentiate between low iron level from iron deficiency anaemia and anaemia of chronic
disease is:

A. Ferritin(C)

B. TIBC

C. Serum iron

D. Serum Transferrin
134. 33 years old Saudi man from Eastern province came to you for routine pre- employment
physical exam. He has always been healthy and his examination is normal. Lab: HTC 35%, MCV 63fl,
WBC 6800/ul, retics:4000/ul(0.7%), platelet :27000/ul his stool:-ve for occult blood. The most direct
way to confirm suspected diagnosis

A. Peripheral smear (C)

B. Measure HbA2 level

C. G6PG screening

D. Measure iron, TIBC, ferritin level

E. Bone marrow stain for iron

135. 15 years old Saudi boy presented to ER with fever, akin rash and shock. He was resuscitated and
admitted to isolation ward with strong suspicion of meningococcal meningitis. LP confirmed the
diagnosis. One of th following statements is true :

A. Patient should be isolated in negative pressure room(C)

B. Prophylaxis treatment should be given to all staff and patient who were in the ER when the
patient was there

C. Ciprofloxacin 500mg once is an acceptable chemotherapy

D. Meningococci are transmitted by contact only

E. Meningococci are resistant to penicillin

136. Most common source of bacterial infection in IV cannula is;

A. Contamination of fluids during manufacturing

B. Contamination of fluids during insertion of the cannula

C. Contamination during injection of medication

D. Seeding from remote site due to intermittent bacteraemia

E. Contamination at site of entry through skin (C)

137. 68 years old businessman diagnosed to have hepatocellular carcinoma. One is true regarding
disclosure ( informing patient)

A. Patient should be told immediately after confirming the diagnosis regardless of his wishes

B. Only patient's family should be informed

C. 50% survival rate should be calculated according to literature and discuss with patient

D. Social worker should be responsible to tell the patient

E. Patient morale and understanding should be studied before telling him (C)

138. In brucellosis, all the following are true except :

A. Brucella abortus cause more severe form than B. melitansis in children (C)
B. Human to human is rarely documented

C. Human can be infected through inhalation

D. Brucella species are small, non motile gram (gibberish) coccobacilli

139. Which of the following is the appropriate method to prevent brucellosis?.

A. Killing vectors.

B. Prophylactic antibiotics

C. Pasteurisation of the milk (C)

140. In brucellosis, all is true except

A. Back pain

B. Hepatomegaly

C. Splenomegaly

D. Lymphadenopathy(C)

E. Gastroenteritis

141. Common symptoms of Hodgkin's lymphoma not seen in Non Hodgkin's lymphoma :

A. Night sweat

B. Superior vena cavalry syndrome

C. COS involvement

D. Intussusceptions

E. Bone pain(C)

142. Boy presented with painless neck mass, history of 5 weeks of fatigue, generalised pruritus and
mild cough :

A. Hodgkin's lymphoma (C)

B. Lyme

C. Infectious mononucleosis

143. Blood pressure, all the following are true except :

A. If 2/3 of cuff false high BP

B. Internal cuff must cover 80% of arm

C. Follow circadian vary late night high BP(C)

D. High BP 3 standard deviation away from normal

E. You have to have more than one reading to Dx high BP

144. Blood PH:


A. High after diarrhoea

B. Low after vomiting

C. More on right atrium than Lt atrium

D. Lower in the right atrium than left ventricle (C)

E. Lower in renal vein than renal artery

145. Increased bleeding time is seen in all of the following except :

A. Haemophilia (c)

B. Scurvy

C. Von- willebrand disease

146. Fecal leukocytes come with all except :

A. Shigellosis

B. Clindamycin induced colitis (C)

C. Idiopathic ulcerative colitis

147. All of the following cause secondary HTN except :.

A. Pheochromocytoma

B. Addison disease (C)

C. Hyperaldosteronism (Conn disease)

D. Renal disease

E. Pregnancy

148. All can be used for the treatment of acute gout except :

A. Allopurinol (c)

B. Penicillamine

C. Gold salt

D. Paracetamol

E. Indomethacin

149. Patient on chemotherapy presented with fever, all should be done except

A. Blood culture

B. Urine culture

C. Aspirin is effective (C)

D. Broad spectrum antibiotics

150. Complications of systemic hypertension are all except


A. Intracerebellar haemorrhage

B. Renal artery stenosis (C)

151. All are true except :

A. Iron supplement is not essential in all breast fed infant (C)

B. Normal pregnancy do not aways end in normal deliveries

C. All TB regimens should have INH

D. One or more essential amino acids are deficient in most vegetables

E. Protein of low biological value present in cereals and legumes

152. All cause recent loss of weight except :

A. AIDS.

B. Cancer

C. Nephritic syndrome (C)

D. Kwashiorkor

153. Patient suspected of having brain abscess, what is the most important question in the history?

A. Frontal sinusitis (C)

B. Ear discharge

C. Head injury

D. Bronchiectasis

154. All of the following is extrapyramidal symptoms except:

A. Dyskinesia

B. Akathesia

C. Bradykinesia

D. Clonic-tonic convulsion (C)

155. Migraine case( How to confirm the diagnosis)

A. MRI

B. Careful history and examination (C)

156. 17 years old girl presented with unilateral headache, nausea, exacerbated by movement and
aggravated by light, what is the diagnosis?

A. Migraine ( photophobia, vomiting) (C)

B. Cluster

157. Adult with unilateral headache, pulsatile increase with activity and light :
A. Migraine

158. Old patient with progressive weakness of hand grip, dysphagia

A. Myasthenia gravis

159. Gullian-Barrie syndrome is closely associated with which of the following :

A. Descending paralysis start from Upper limb

B. Normal CSF

C. Ascending paralysis start from the lower limb(C)

D. Need ECG

160. Patient with CV came after 6h, give him

A. Aspirin

B. t-PA

C. Clopidrogrel

D. Heparin

161. Most common cause of CVA. Mostly embolic source.

A. AF(C)

B. VSD

162. An old man undergoing brain surgery and in aspirin. He needs prior to surgery:

A. Vitamin K parenterally

B. Vit. K orally.

C. Delay surgery for 2 days

D. Delay surgery for 2 weeks(C)

163. Depressed patient ha ingested big quantity of Aspirin 6 hours ago, came to ER complaining of
nausea, vomiting, increase respiration, investigation showed highly elevated level of ASA, what is
your action?

A. Urine acidity

B. Charcoal

C. Haemodialysis

D. Alkalinisation of the urine (C)

164. Positive meningococcal TB

A. Rifampicin 7 days (C)

B. 3- single dose IM Ceftriaxone

165. Patient discharge with meningococcal meningitis and now asymptomatic, what is the next step?
A. Rifampicin

B. Ceftriaxone (C)

C. No vaccine

166. Child was sick 5 days ago, culture taken showed positive for meningococcal. Patient now at
home and asymptomatic, your action will be

A. Rifampicin

B. IM Ceftriaxone (C)

167. Old female with recurrent fracture, vitamin D insufficiency and smoker. Which exogenous
factor the greatest exogenous side effects on the osteoporosis?

A. Old age

B. Smoking

C. Vitamin D insufficiency (C)

D. Recurrent fracture

168. Miscellaneous

A. Avoid Obesity

B. Vitamin D daily (c)

C. Weight bearing exercise

169. Which is true about backache with osteoporosis?

A. Normal x-ray vertebrae exclude the diagnosis

B. Steroids is beneficial TTT

C. Vitamin D deficiency is the cause (_C)

170. Old lady with recent osteoporosis ask about drug to prevent lumbar fracture:

A. Vit D

B. Bisfosphonate

C. Exercise

171. What is the most common non- traumatic fracture caused by osteoporosis?.

A. Colle's fracture

B. Femoral fracture

C. Vertrbral compression fracture (C)

172. Adolescent female with eating disorder and osteoporosis. What is the treatment?

A. Weight gain(C)

B. Vit D
C. Bisfosphonates

173. 70 years old male with osteoporosis, the T score of Bone densitometry would be :

A -3.5 (C)

B. Less than -2.5

C. -1

D. -2

174. Old male, back pain, examination is normal, gave him steroid, came back with vesicle from back
to abdomen :

A. VZV

175. Patient present with mid face pain, erythematous lesions an vesicles on periorbital and
forehead , the pain is at nose, nose is erythematous. What isn't he diagnosis?

A. Roseola.

B. HSV

C. Herpes zoster (C)

176. All the following cause hyponatremia except :

A. DKA

B. Diabetes insipudus( causes hypernatremia due to huge loss of water in the form of diluted urine)
(C)

C. High vasopressin level

D. Heart failure

177. Anti- inflammatory drug can cause all except

A. Acute renal failure

B. Tubular necrosis

C. Hypokalemia (C)

D. Interstitial nephritis

178. All in hypokalemia except:

A. Hyperosmolar coma

B. Phenytoin toxicity (C)

C. Muscle paralysis

179. Which of the following could be seen in patient with bulimia?

A. Hypokalemia (C)

B. Metabolic acidosis
180. One of the following condition does not cause hypokalemia

A. Metabolic alkakosis

B. Furosemide

C. Hyperaldosteronism

D. Acute tubular necrosis (C)

E. Diarrhoea

181. Best economical NSAID is

A. Indomethacin

B. Brucella (C)

182. The drug with the least side effects for the treatment of SLE is :

A. NSAIDS (C)

B. Methotrexate

C. Corticosteroids

D. Hydroxychloroquine

183. All the following cause hyponatremia except :

A. DKA

B. Diabetes insipudus (hypernatremia) (C)

C. High vasopressin

D. Heart failure

184. Cherry red skin found in :

A. Polycythemia

B. CO poisoning

185. Duration of drug in rheumatoid fever is:

A. 6 years.

B. 15 years

C. Primary prevention lasts for 10 days and 2ry prevention lasts for 5 years or 10 years depending on
presence of carditis (C)

186. Not true about hypokalemia :

A. ST changes

B. Happened in hyperosmolar, non- ketotic

C. PR changes
D. All true (C)

187. Earlier sign of puberty in male is:

A. Appearance of Pubic hair

B. Increase testicular size (C)

C. Increase penis size

D. Increase prostate size

188. Patient with 2ry syphilis receive 2nd of penicillin became hypotensive

A. Stop penicillin

189. Male patient with hemarthrosis. What is the most likely diagnosis?

A. Thrombocytopenia

B. Factor 8 deficiency (C)

190. Female patient had carpopedal spasm after measuring her BP. This is caused by:

A. Hypocalcemia

191. Patient with microcytic anemia without megaloblast. What's the most likely diagnosis?

A. Folic acid

B. Vit. B12 deficiency

C. Alcoholism

192. Which of the following method is rapid and best for complete gastric evacuation?

A. Gastric lavage (C)

B. Manual induce vomiting

C. Syrup

D. Active charcoal

193. Patient with severe vomiting and diarrhoea in ER when he stand he feels dizzy. Supine BP
120/80 on sitting 80/40. When asking him he answers with loss of sensorium, what does he likely
have?

A. Insulin (gibberish)

B. Dehydration (gibberish) (C)

194. Patient with a scenario going with liver cirrhosis with ascites , diet instructions:

A. High carbs, low protein (C)

195. In active increase transaminase which of the following drugs contraindicated :

A. Rinatidine

B. Infidipine
C. Vastatin (C)

196. 40 years old patient known to have Crohn' s disease, came with fevers, hip and back pain, blood
positive brown stool. On examination, soft abdomen, normal bowel sounds, normal range of motion
of hip. What has the best radiological diagnosis?

A. Abdominal US

B. Abdominal CT(C)

C. Hip CT

D. IV venogram

E. Kidney US

197. Patient wi chronic heartburn, treated with antacids, no improvement what is the next action:

A. Another antacids

B. H2 blockers

C. PPIs(C)

D. Prokinetic agents

198. Symptom of reflux esophagitis

A. Minor the risk of MI

B. Not affected by alkali

C. Increase by standing

D. Can be distinguished between it and duodenal ulcer. (C)

199. Patient with diffuse abdominal pain, diminished bowel sounds, x- ray showed dilated loop
specially the transverse, what's the diagnosis?

A. Acute pancreatitis(C)

B. Acute cholecystitis

C. Bacterial enteritis

200. Celiac disease patient, all should be avoided except :

A. Wheat

B. Oat

C. Rice(C)

201. Which drug increase incidence of reflux esophagitis :

A. Theophylline (C)

B. Amoxicillin

C. Metoclopramide
D. Ranitidine

E. Lansoprazole

202. Young patient complaining of artery diarrhoea, abdominal pain, with a previous history of
mucus diarrhoea

A. Crohn's

B. UC

C. IBS

203. Young female complaining of severe diary, weight loss, vomiting, abdominal pain, has been
diagnosed to have Crohn's disease

A. Female more affected

B. Non- caseating granules as (c)

C. Diabetic.

D. Unknown

204. Which of the following is true about headache ;

A. Increase ICP at last of day

B. Normal CT may exclude SAH(C)

C. Amarousis fugax never come with temporal arteritis

D. Neurological sign may exclude migrant

205. Young man predict that he is going to have a seizure, then h became rigid for 15 sec then
developed generalised tonic clinic convulsion for 45 sec, your initial ER action in future attacks will
be :

A. Insert airway device (C)

B. Apply physical splint or protection

206. Patient with disc prolapse will have:

A. Loss of ankle jerk (C)

B. Fasciculation of posterior calf muscles

C. Loss of dorsiflexion compartment of the foot

D. Loss of the sensation of the groin and anterior aspect of the thigh.

207. Patient after trauma to the knee present with knee swelling of bloody content, the probable
mechanism is:

A. Platelet deficiency

B. Clotting factor deficiency (C)

C. Platelet dysfunction
D. Blood vessels dysfunction

208. Blast cell

A. AML

B. ALL (C)

C. CML

D. CLL

209. Uric acid in body, how the body removed by

A. Increase metabolism of Uric acid in the liver

B. Increase excretion of Uric acid in urine (C)

C. Excretion of uric acid by lung

210. What is the more prognostic factor for chronic granulocytic leukaemia

A. Stage (C)

B. Bone marrow involvement

C. Age at discovery

211. Elderly patient known case of IHD, you give him PRBC,but after that he suffered from fever with
temperature of 38.5, what will you do?

A. Decrease rate of transfusion

B. Stop transfusion and treat patient with acetaminophen only (C)

C. Stop transfusion and treat patient with mannitol and acetaminophen

212. Patient came with pitting edema grade 1, where will the fluid accumulate?

A. Arteriole

B. Venule

C. Interstitial (C)

D. Capillary

213. What is the pathophysiology of infection in DM why they develop infection?

A. Decrease phagocytosis

B. Decrease immunity (C)

C. Help in bacteria overgrowth

214. Case about patient with papules in the genital area with central umbilication, history of
unprotected sex 'molluscum contagiosum', what is the treatment?

A. Acyclovir

215. Doctor do breath by mask, but nothing happens. What will you do?
A. Continue one breath every second

B. Put him in recovery position.

C. Intubation (C)

D. Do nothing till whole medical team

216. DKA

A. Starvation cause increase of amino acids and fatty acids which utilise by the body

B. Ketone body which excreted in urine

C. Decrease in insulin lead to fatty acid and Ketone body (C)

217. 70 years old patient, came with investigations showed osteolytic lesion in skull, monoclonal
spike, rouleaux formation :

A. Multiple myeloma

218. Cause hypertensive crisis :

A. Enalapril

B. Lorsatan

C. Hydralazine. (C)

219. Which one of the drugs causes hypertensive crisis when it is not stopped gradually?

A. Diltiazim

B. Clonidine (c)

C. Beta blocker

220. Hypertensive patient with liver cirrhosis, Lower limb edema and ascites, what to use?

A. Thiazide better K- sparing diuretic (C)

B. Hydralazine

221. Patient with hepatomegaly, Kayser- Fleischer rings, what is the treatment?

A. Penicillamine ( Wilson disease)

222. Patient work in hot weather, came with claims cold skin, hypotensive, tachycardia

A. Heat stroke

B. Heat exhaustion (C)

223. Elderly patient known case of HTN and BP, which one of the following drug is potentially
recommended for such case :

A. Atenolol

B. Terazocin

C. Losartan
224. In cachetic patient the body utilize the proteins of the muscles.

A. To provide amino acids for protein Synthesis

225. Patient walking for relatively long time on ice when she was on vacation. Her feet is pale with
marked decrease in pain sensation but the pulse is palpable over dorsalis pedis. What is rh
appropriate thing to do:

A. Immediate heat with warm air

B. Put her feet in warm water (C)

227. Man traveled to some country. There is endemicity of onchocerciasis, he stays there for 1 wk.
His ability to get the disease is.

A. High

B. Sever

C. Minimum

D. Non existent

228. Patient with severe hypothyroidism and hyponatremia (Na=108), high TSH and not responding
to painful stimulus, how would you treat him?

A. Intubatev, give 3% Na then treat hypothyroidism status(C)

B. Treat hypothyroidism and monitor serum Na level every 6 hours

C. Thyroid and fluid replace only

D. Thyroid and fluid and 3% Na

E. Give 3% Na, hydrocortisone and treat hypothyroidism

228. Patient with HTN presented with edema, azotemia, GFR:44 (not sure about -5) what is the
cause of her kidney disease?

A. Bilateral renal artery stenosis (c)

B. Diabetic nephropathy

C. Reflux.

D. Renal tubular acidosis

229. 100% O2 given for prolonged periods can cause all except

A. Retrosternal pain

B. Seizures

C. Depression

D. Ocular toxicity
OBSTETRICS AND GYNAECOLOGY
1. Female patient with DM well controlled and she wants to get pregnant and she asked you
about the risk of congenital abnormality, to avoid this, diabetes control should start in

a. Before pregnancy(correct)
b. 1st trimester
c. 2nd trimester
d. 3rd trimester

2. Pregnant lady, she wants to do a screening test, she insists thst she doesn’t want any
invasive procedure, what will you do?

a. U/S(correct)
b. Amniocentesis

3. What is the risk of GDM on her life later?

a. DM type1
b. DM type2(correct)
c. Impaired fasting glucose

4. Clomiphene Citrate;

a. Induce Ovulation

5. Pregnant lady with cardiac disease, presented in labour. You will do all except;

a. Epidural anasthesia
b. C/S(correct)
c. Diuretic
d. Digitalis
e. Oxygen
6. Asymptomatic woman with trichomoniasis

a. Treat if symptomatic
b. Treat if she is pregnant
c. Treat her anyway(correct)

7. A pregnant woman, multigravida, 38weeks gestation, presents with glucosuria.


Gestational diabetes was confirmed by glucose tolerance. What is the next step?
a. Repeat glucose tolerance test
b. C/S
c. Diet adjustment(correct)
d. Start sliding scale insulin
e. Start oral hypoglycemic medication

8. Pregnant lady in her 30weeks gestation, diagnosed as having swine flu. She has high grade
fever and cough for 4days and her RR=25/min. What will you do?

a. Give her Tamiflu 75mg BID for 5days(corrrect)


b. Refer her to er for admission
c. Refer her to an OBGY Doctor

9. A 27years old pregnant lady, 19weeks gestation, smoker, presented with PV bleeding,
followed by painless delivery. She was told nothing was wrong with her or her baby. The
diagnosis is;

a. Cervical Incompetence(correct)
b. Fetal chromosomal anomaly
c. Molar pregnancy

10. The commonest symptom in the presentation of abruptio placenta is;

a. Vaginal bleeding 80%(correct)


b. Abdominal pain
c. Abnormal mass
d. Irregular uterine contractions 35%
e. Hypogastric tenderness

11. Pregnant lady, 8weeks gestation, came with history of bleeding for the last 12 hours with
lower abdominal pain and she passed tissue. O/E; the internal Os was 1cm dilated. The
diagnosis is;

a. Complete abortion
b. Incomplete abortion(correct)
c. Missed abortion
d. Molar pregnancy
e. Threatened abortion

12. Young primigravida, 35weeks gestation had BP of 140/90, headache, proteinuria and
lower limb edema. What is the best management?

a. Oral labetolol
b. Diuretics
c. Low sodium diet
d. Admission and observation of fetal-maternal condition(correct)
13. A 30year old lady in the 3rd trimester of her pregnancy developed a sudden massive
swelling of the left lower extremity extending from inguinal ligament to the ankle. The
most appropriate sequence of workup and treatment;
a. Venogram, bed rest, heparin
b. Impedance plethysmography, bed rest, heparin(correct)
c. Impedance plethysmography, bed rest, vena caval filter
d. Impedance plethymosgraphy, bed resr, heparin, warfarin
e. Clinical evaluation, bed rest, warfarin

14. A 55years old lady on HRT is complaining of spotting on day21 of the cycle. What will you
do?

a. Pap smear
b. Endometrial sampling(correct)
c. Stop HRT
d. Add progesterone
15. A young female patient,office worker,presented with itchingin the vagina associatedwith
greenish-yellow vaginal discharge. Examination revealed red spots on the cervix. The
diagnosis is;

a. Trichomoniasis(correct)
b. Candidiasis
c. Gonorrhea
d. Gardnerella vaginalis

16. A female patient presented with oligomenorrhea, she had 3periods in the last year. She
also had acne and hirsutism. Her body wgt was 60kg. PV examination was normal. The
diagnosis is?

a. Polycystic ovary disease(correct)


b. Hyperprolactinemia
c. Adrenal tumour
d. Hypothyroidism
e. Premature ovarian failure

17. Uterovaginal prolapse;

a. Increased heaviness in erect position(correct)


b. More in Blacks
c. A common cause of infertility

18. A couple is trying to have a baby for the last 6monthsof unprotected sex. They wanted to
know the possible cause of their infertilty. What will you do?

a. Wait and see if less than 35y(correct)


b. Send to fertility clinic if more than 35y
c. Semen analysis
d. Pelvic exam
e. Body temperature chart

19. A 34years old lady presented with pelvic pain and menorrhagia. There is history of
infertility, on examinations,the uterus was normalsize and retroverted, she had multiple
small tender nodules palpeable in the uterosacral ligament. The most likely diagnosis is;

a. Fibroid
b. Endometriosis(correct)
c. Adenomyosis
d. PID

20. 50years old woman(postmenopausal) who is taking estrogen OCP every month and stops
at the 21st day of the cycle. She presented with vaginal bleeding in the form of spotting 2-
3days after stopping the estrogen OCP(A case of postmenopausal bleeding). The best
management is;

a. Pap smear
b. Endometrial sampling [biopsy](correct)
c. Stop estrogen
d. Continue estrogen
e. Add progesterone

21. OCP;

a. Changes the cervival mucus(correct)


b. Increases post menstrual tension
c. Have a failure rate of 3%

22. The best indication of labour progression is;

a. Dilation
b. Degree of pain
c. Fetal heart rate
d. Decent
e. Dilation and decent

23. OCP;

a. Decreases the risk of ovarian cancer(correct)


b. Inrease the risk of breast cancer

24. Average length of menstrual cycle


a. 22days
b. 25days
c. 28days(correct)
d. 35days

25. About antepartum hemorrhage;

a. Rarely due to hypofibrinogenemia


b. Maternal mortality more than fetal mortality(correct)
c. PV exam is always indicated

26. Old patient ,known case of hypothyroidism on thyroxin, presented with many symptoms,
labs all normal(TSH,T3,T4) except low calcium, high phosphate. What is the diagnosis?

a. Primary hyperthyroidism
b. Secondary hyperthyroidism
c. Secondary hypoparathyroidism(correct)
d. Uncontrolled hypothyroidism

27. Pregnant lady came to antenatal clinic foe routine checkup, her glucose tolerance test was
high glucose,diagnosed as DM. Management;
a. Nutritional advice(correct)
b. Insulin
c. OHA
d. Repeat GGT

28. Pregnant lady with negative antibodies for rubella and measles. What will you give her?

a. MMR
b. Antibodies
c. Terminate pregnancy
d. Do nothing(correct)

29. 20year old pregnant lady,exposed to rubella virus since 3days, never was vaccinated
against rubella mumps or measles. Whatis the best thing to do?

a. Give IG
b. Vaccine
c. Do nothing(correct)
d. Terminate the pregnancy

30. Pregnant lady, 7cm dilated cervix, had induction of labour with oxytocinand artificial
rupture of membrane. Hypertensive and the baby is Brady. What wiil you do?

a. Magnesium sulfate(correct)
b. Give dose of oxytocin

31. Treatment of chlamydia with pregnancy;

a. Azithromycin
b. Erythromycin(correct)

32. The most common cause of postpartum hemorrhage is?

a. Uterine atony(correct)
b. Coagulation
c. Retained placenta

33. Girl with amenorrhea for many months. BMI is 20 and is stable over ladt 5years. Diagnosis;

a. Eating disorder
b. Pituitary adenoma(correct)

34. Regarding GDM;

a. Screening for GDM at 24-28weeks(correct)


b. Diet control is always succesful treatment
c. Screening at 8weeks

35. 48years old with irregular menses presented with fatigue and mensruation for 3months
with increased pigmentation around the vaginal area without other symptoms. Your nexy
step would be;

a. Reassure the patient


b. Do a pregnancy test(correct)
c. Do U/S

36. Marker of ovarian cancer;

a. CA125

37. Total vaginal hysterectomy with posterior and anterior repair. The patient complains that
urine is coming out through vagina. What is your diagnosis?

a. Uterovaginal fistula(correct)
b. Vesicovaginal fistula(continuous urinary incontinence)
c. Urethrovaginal fistula
d. Cystisis
38. Irregular menses 2months and sometimes nothing, lasts for seven days and uses 10-
15pads on heavy days;

a. Polymenorrhea(cycles with intervals of 21days or fewer)


b. Menorrhagia
c. Menometrorrhagia(correct)
d. Oligomenorrhea

39. Couple came for reversible contraception, the wife previous DVT. What will you advice?
a. Tubal ligation
b. IVD(correct)

40. Lactating lady who didn’t take MMR;

a. Take the vaccine and stop feeding for 72hours


b. It is harmful for the baby
c. She can take vaccine(correct)

41. Signs of androgen excess and ovarian mass

a. Seroli-leydig cell tumor

42. Female on antibiotics has white cottage chessy vaginal discharge

a. Candida

43. Girl with hirsutism,deep voice,receeding hair line;

a. Androgen excess

44. Pregnant in the 3rd trimester, came with painless bleeding

a. Placenta previa

45. Trichomonas vaginalis associated with STD;


a. DM
b. Pregnancy
c. Greenish frothy discharge(correct)

46. Pregnant lady 16weeks, U/S shows snowstorm appearance;

a. Complete hydatiform mole

47. Patient came with whitish discharge from the nipple. Her investigation show pituitary
adenoma. Which hormone is responsible for this?
a. Prolactin

48. Young girl came with history of full term uterine demise and now she is in 34weeks. What
will you do?

a. C/S in 38weeks
b. Wait for spontaneous delivey
c. Induce labor at 36weeks, not more than 4weeks from diagnosis(correct)

49. Lady pregnant, in her 3rd trimester, came with bright red gush of blood, no abdominal pain
or uterine tenderness;

a. Placenta previa

50. Patient complain of tension headache, was on acetaminophen but no improvement. She
noticed that the headache improved when she was pregnant;

a. Triptan trial medication(for cluster and migraine,inhibit dilation of cranial


vessels)[correct]
b. Let her quit her job
c. Drug induced amenorrhea

51. Cause of polyhydraminois;

a. Renal agenesis
b. Duodenal athresia(correct)
c. Mother with diabetes insipidus
d. Postmortem pregnancy

52. The most accurate diagnostic investigation for ectopic pregnancy;

a. Culdocentesis
b. Pelvic U/S
c. Endometrial biopsy
d. Serial B-HCG
e. Laparoscopy(correct)

53. A 14year old female with 6month history of lower mid abdominal pain, pain is collickily,
radiates to the back and upper thigh,begins with onset of menses and lasts for 2-4days.
She misses several days of school during the last 2months. Physical examination of
abdomen and pelvis normal, normal secondary sex development. What is the likely
diagnosis?

a. Primary dysmenorrhea(correct)
b. Secondary dysmenorrhea
54. Nulligravida at 8weeks GA, follow up for genetic screening,she refuse the invasive
procedure but she agree for once screening. What is the appropriate action now?

a. Do U/S
b. 1st screening U/S + Maternal blood(correct)
c. 2nd screening
d. 3rd screening
e. Amniocentesis

55. Which type of contraceptive is contraindicative in action?

a. OCPs(correct)
b. Mini pills
c. IUD
d. Condom
e. Depo-provera

56. Best medication to be given for GDM(gestational) is;

a. Insulin(correct)
b. Metformin

57. A vaccination for pregnant lady with DT;

a. Give vaccine and delivery within 24hours


b. Contraindicated in pregnancy
c. Not contraindicated in pregnancy(correct)

58. Contraceptive pill that contain estrogen increases risk of;

a. Breast Cancer(correct)
b. Ovary cancer
c. Cervical cancer

59. The best stimulus for breastmilk secretion is;


a. Estrogen
b. Breast feeding

60. Pregnant diagnosed with UTI. The safest antibiotic is;


a. Ciproflaxin
b. Ampicillin(correct)
c. Tetracycline
61. Full time wide pelvis lady, on delivery station +2, vertex,CTG showed late deceleration.
What is the most appropriate management?
a. C/S(correct)
b. Suction
c. Forceps Delivery
d. Spontaneous delivery

62. Endometriosis best diagnosed by;


a. U/S
b. Laparoscopy(correct)
c. Laparotomy

63. 41weeks pregnant lady. Last biophysical profile showed oligohydroamnios. She has no
complaints except mild HTN. What is the appropriate management?

a. Wait
b. Induce labour post 42weeks
c. Induce labour(correct)
d. Do biophysical profile twice weekly

64. Young female with whitish grey vaginal discharge KOH Test. Smell fish-like. What is the
diagnosis?

a. Gonorrhea
b. Bacterial vaginosis(correct)
c. Trichomonas vaginalis

65. Complain of painless, odorless and colorless vaginal discharge that appear after
intercourse, so ttt;

a. Give her antibiotic


b. Douche after intercourse(correct)
c. Cervical cancer should be considered
d. May be due to chronic salpigitis

66. Obstructed labour, which is true?

a. Common in primi
b. Excessive caput and moulding are common signs
c. Most common occipitio-ant
d. Can not be expected before labour(correct)

67. Primary dysmenorrhea;

a. Periods painful since birth


b. Pain starts a few days before flow
c. NSAID helps(correct)

68. After delivery, start breastfeeding;

a. As soon as possible(according to WHO: within half an hour)[correct]


b. 8hours
c. 24hours
d. 36hours
e. 48hours

69. A 28years old lady with 7 weeks history of amenorrhea has lower abdominal pain, home
pregnancy test was positive, comes with light bleeding. Next step;
a. Check progesterone
b. HCG(correct)
c. Placenta lactogen
d. Estrogen
e. Prolactin

70. Pregnant on 36th week, came with 7cm cervical width at 0station. During birth, CTG shows
late deceleration. Managemant is;
a. Give oxytocin
b. Oxygen and change mother position(correct)sssssss
c. Give Mg sulfate

71. Patient has history of cervical incompetence, pregnant at eight weeks. What is the
management?

a. Do cervical cerclage at 14-16weeks

72. Patient has amenorrhea for 6weeks,presented with abdominal pain. O/E: there is fluid on
Douglas pouch and clot blood;

a. Rupture ectopic pregnancy

73. Patient has a white vaginal discharge and itching, what does the patient have?

a. DM

74. Pregnant lady, the thyroid function test shows (high TBG&T4) and upper normal T3. This is
due to;

a. Pregnancy
75. Female patient came with generalized abdominal pain, by examination you found
suprapupic tenderness,by PV examination,there is tenderness in moving cervix and tender
. adnexia. Diagnosis;

a. Pelvic inflammatory disease

76. Treatment of patient with yellowish vaginal discharge and itchy by swab and culture. It is
Trichomonas vaginalis. Which of the following is correct?

a. Start treatment with metranidazole(correct)


b. Start treatment with clindamycin
c. No need to treat husband
d. Vaginal swab culture after 2weeks

77. 50years old giving history of(postmenopausal symptoms), hot flushes. Best drug to reduce
these symptoms is;

a. Estrogen only
b. Progesterone only
c. Combined pills(estrogen & progesterone)
d. Venlafaxine or clonidine or HRT, if not combined pills(correct)

78. Pregnant lady giving history of increased body wgt about 3kg from the last visit and lower
limb edema to confirm that she had pre-eclampsia. What will you do?

a. Measure her BP

79. Female patient came with severe vaginal bleeding. What is the appropriate initial
management?

a. Oxygen, IV fluid, ABC, if not stop,progesterone and estrogen, and last one is blood
transfusion

80. Pregnant lady G1P0 at 13weeks. She looks anxious but she is happy about her pregnancy.
Her BP is 142/96. She does exercise 4-5times in a week. She denies that she has any
previous medical problem. What is diagnosis?
a. Pre-eclampsia
b. Pregnancy induced hypertension
c. Chronic hypertension(correct)
81. Pregnant woman at 28weeks,she sits with child, this child develop chickenpox, she comes
to you asking for advice. You found that she is seronegative for( varicella) antibody. What
will be your management?
a. Give her (VZIG) varicella zoster immunoglobulin(correct)
b. Give her acyclovir
c. Give her varicella vaccine
d. Wait until symptoms appear in her

82. During rape, rupture of hymen is at;

a. 6 ‘0’ clock

83. Pregnant with 32weeks, no any abnormality, asking what the outcome should be to this
patient;

a. Induction at 36weeks

84. Comes with lower abdominal tenderness wiyh no sign of infection and HCG normal

a. Ovarian cyst torsion

85. Pregnant , 34weeks, with abdominal pain radiating to back. O/E; transverselie, back down
& PV revealed open cervix 3cm and plugging of bag. Management?

a. C/S(correct)
b. Tocolytics

86. Couples asking for emergency contraception.

a. Levonorgestrel 1.5mg

87. Pregnant, 36weeks, presented with agitation, BP=88/60, fetal distress. Diagnosis;

a. Pulmonary embolism
b. Amniotic fluid embolism(correct)

88. Patient with salpingitis and there is swelling in pelvis in posterior fornix and it is fluctuant.
Management;

a. Colpotomy
b. Laparoscopy
c. Continues oral therapy

89. Salpingitis and PID on penicillin but not improving. What is the most likely organism?

a. Chlamydia
b. Neisseria gonorrhae(correct)
c. Syphilis
d. HSV
90. Chronic use of estrogen association;
a. Increases the risks of breast and cervical cancer and reduced uterus and ovary
91. Regarding injectable progesterone;

a. Can cause skin problems


b. Asociated with irregular bleeding and weight gain(correct)
c. Decrease in bone mineral density(reversible)
d. Delayed fertility after discontinuation(one shot can last 10months)

92. Lady, 28years old G3+3, complaining of infertility

a. Uterine fibroid

93. 29years old lady, B-CHG160, complaining of vomiting and abdominal pain. Which is more
accurate to diagnosis?

a. BCHG Serial
b. Pelvic U/S (correct)
c. Laparoscopy

94. Breech presentation at 34weeks;


a. Do ECV now
b. Do ECV at 36weeks(correct)
c. C/S

95. Female lady after delivery started to develop pelvic pain, fever, vaginal dischargeand
negative Leich’r test. What is your diagniosis;(I DON’T KNOW WHAT THAT TEST IS)

a. Vaginal yeast
b. PID(correct)
c. Bacterial vaginosis

96. 34weeks with antepartum hemorrhage, she was conscious but fighting. What is the most
likely cause?

a. Post-coital bleeding

97. Pox virus vaccine in lactating lady;

a. Give the vaccine

98. Increased frequency of menses is called;

a. Menmetror
b. Polymenor(correct)
c. Hypermenor
99. Classical case of candida infection ‘’itching, white dischargefrom vagina’’. The treatment is;

a. Miconazole(correct)
b. Amoxicillin

100. Postpartum lady developed blues, beside psycotherapy;

a. Encourage family to support patient

101. Pregnant lady with cystitis,one of the following drugs contraindicated in her case;

a. Amoxicillin
b. Ceftriaxone
c. Fluoroquiolone(correct)

102. All of the following drugs contraindicated in breast feeding except;

a. Tetracycline
b. Chlorophenicol
c. Erythromycin(correct)

103. In initial evaluation of couples for infertility;

a. Temperature
b. Semen analysis(correct)
c. Refer to reproductive clinic

104. Common cause of male infertility;

a. Primary hypogonadism(correct)
b. Secondary hypogonadism
c. Ejaculation obstruction

105. Conditions not associated with increased alpheto protein;

a. Breech presentation(correct)
b. Down syndrome
c. Gastroschisis

106. Pregnant, never did checkup before, her baby born with hepatosplenomegaly and
jaundice;

a. Rubella
b. CMV(correct)
c. HSV
d. Toxoplasmosis

107. Female patient around 35years old, history of thromboembolic disease. What type
of reversible contraceptive can she use?

a. OCP
b. Mini pills
c. IUCD(correct)

108. Female with vaginal infection(chlamydia) not pregnant. Best treatment;

a. Doxycycline
b. Azithromysin
c. Metronidazole

109. Patient G3P3, all her deliveries were normal except after the second one, she did
D&C. Labs all normal except; high FSH, high LH, low estrogen. Diagnosis;

a. Ovaraian failure(correct)
b. Asherman syndrome

110. Pregnant, 6days in C/S, staining in her throbs from abdomen


a. Fascial dehiscence

111. Female with vaginal bleeding, abdominal pain. What is the first investigation?
a. U/S(correct)
b. Vaginal examination
112. 16 weeks pregnant, complaining of polydipsia and polyuria less than 126mg fasting
6.8;

a. Impaired DM

113. Breech presentation at 34weeks . treatment option;

a. External cephalic
b. Internal cephalic
c. Wait(correct)
d. Induction

114. Postpartum hemorrhage happens more commonly with

a. Multiple pregnancies(correct)
b. Anemia
c. Preterm delivery
d. Antithrombin III deficiency

115. What is the most complication after hysterectomy;

a. Ureteral injury
b. Pulmonary embolism
c. Hemorrhage(correct)

116. 34years old with HIV, pap smear negative. About cervical cancer screening;

a. After 3months if negative repeat after 6months


b. After 6months if negative repeat anually(correct)
c. After 1year if negative repeat anually

117. Post D&C, the most common site of perforation is the;

a. Fundus(correct)
b. Anterior wall of the corpus
c. Posterior wall of the corpus
d. Lateral wall of the corpus
e. Cervix

118. 16weeks of gestation presented with (++)glucosuria, FBS4.4, 1hoursPB=8,


2hours=7.2

a. Renal glucosuria(correct)
b. GDM
c. KM Syndrome

119. Primi at 36weeks of gestation with pre-eclampsia, BP is high with ankle edema.
The best to be done is;

a. Diuretics
b. Low salt diet
c. Labetolol
d. Immediate delivery
e. Maternal-Fetal monitoring with continuous hospitalization(correct)

120. Most common site for ectopic pregnancy

a. Fallopian tube

121. Most common site of gonococcus infection in females

a. Cervix
b. Posterior formix
c. Urethra(correct)

122. A 34weeks GA lady presented with vaginal bleeding of an amount more of that of
her normal cycle. O/E; uterine contracts every 4mins, bulged membrane, the cervix is 3cm
dilated, fetus is in a high transverse lie and the placenta is on the posterior fundus. U/S
showed transluscency behind the placenta and CTG showed FHR of 170. The best line of
management is;

a. C/S immediately(correct)
b. Give oxytocin
c. Do rupture of membrane
d. Amniocentesis

123. It is contraindication to stop preterm delivery in the following condition

a. Amniochoronitis
b. Placenta abruption
c. Pre-eclampsia
d. A and B (correct)

124. Before you start instrumental delivery. It is important to check if there is;

a. Face presentation
b. CPD
c. Breech presentation
d. Cord prolapse(correct)

125. In occipito-posterior malpositioning of the fetal head, all of these are true except

a. 10% of all vertex delivery


b. It causes significant delay of labour duration compared to the anterior presentation
c. Android pelvis is predisposing factor
d. Flexion of the head helps the rotation to the anterior position(correct)

126. 25years old female patient who is with secondary amenorrhea, her prolactin level
is 400ng/ml. The probability to have pituitary prolactin secreting adenoma is;

a. <25
b. 25-49
c. 50-74(correct)
d. 75-85
e. >85

127. Which is not compatible with head engagement?


a. Vertex at zero station
b. Crowning of the head
c. 3/5 head felt in the abdomen(correct)
d. BPD at ischial spines

128. Female with recently inserted IUCD coming with watery brownish vaginal
discharge and abdominal pain. What is the most likely diagnosis?

a. Uterine rupture
b. Ovarian torsion
c. Bacterial vaginosis(correct)
d. Ectopic pregnancy

129. What is an absolute contraindication of OCP?

a. History of previous DVT(correct)


b. Ovarian cancer
c. Breast cancer

130. OCP is prove to;

a. Decrease ovarian cancer(correct)


b. Decrease endometrial cancer
c. Increase breast cancer
d. Increase risk of ectopic pregnancy

131. Regarding weight gain in pregnancy. What is true?

a. Pregnant women should consume an average calorie of 300-500/day(correct)


b. Regardless of her BMI or body wgt, she should gain from 1.5pound 3lb which
represents the baby’s growth

132. First sign of magnesium sulfate toxicity is;

a. Loss of deep tendon reflex

133. Regarding postpartum psychosis;

a. Recurrences are common in subsequent pregnancies(correct)


b. It often progresses to frank schizophrenia
c. It has good prognosis
d. It has insiduous onset
e. It usually develops around the third week postpartum
134. A case of a patient with thin cervix and little amount of cervical mucus, how would
you treat her?

a. Estrogen injections

135. A pregnant female develops lesion on the vulva and vagina and she was diagnosed
as genital herpes. What should be included in her future health care?

a. C/S should be done if the lesions did not disappear before 2weeks of delivery
b. Oral acyclovir to treat herpes(correct)
c. Termination of pregnancy because of the risk of fetal malformations
d. Avoidance of sexual intercourse for 1month after the healing of the lesions

136. Female patient on the 3rd postpartum. She says to the physician that she
frequently visualizes snakes crawling to her baby’s bed. She knows that it is impossible but
she cannot remove the idea from her head. She says she wakes up around 50times at night
to check her baby. This problem prevents her from getting good sleep and it started to
affect her marriage. What is this problem she is experiencing?

a. An obsession
b. An hallucination
c. A postpartum psychosis
d. A delusion

137. Regarding postpartum depression, what is the most appropriate intervention to


reduce the symptoms?

a. Include family in the therapy(correct)


b. Isolation therapy
c. Add very low doses of imipiramine
d. Encourage breast feeding

138. Pregnant lady delivered anencephaly still born. Occurrence of neural tube defect
in next pregnancy

a. 8%
b. 2%(correct)
c. 10%
d. 20%

139. Young pregnant(primigravida), 32weeks of gestation came to you. C/O; lower


limbs swelling for 2weeks duration, she went to another hospital and she was
prescribed(thiazide & loop diuretic)... O/E; BP=120/70, mild edema, urine dipstic: negative
and otherwise normal. The best action is;
a. Continue thiazide and stop loop diuretic
b. Continue loop diuretic and stop thiazide
c. Stop both(correct)
d. Continue both and add potassium sparing diuretic
e. Continue both and add potassium supplement

140. 38years old female came to you and her pap smear report was unsatisfactoryfor
evaluation. The best action is

a. Consider it normal and D/C the patient


b. Repeat it immediately
c. Repeat it as soon as possible
d. Repeat it after 6 months if considered low risk
e. Repeat it after 1year if no risk(correct)

141. Placenta previa all true except;

a. Painless vaginal bleeding


b. Tone increase of uterus(correct)
c. Lower segmental abnormality
d. Early 3rd trimester

142. 8weeks primigravida came to you with nausea and vomiting. Choose the
statement that guides you to hyperemmesis gravidarm

a. Ketonia(correct)
b. ECG evidence of hypokalemia
c. Metabolic acidosis
d. Elevated liver enzyme
e. Jaundice

143. Pregnant woman G4P3+1, 10weeks GA came to you with IUCD inserted and the
string is out from O.S. What is the most important measure?

a. Leave the IUCD and give A,B


b. Leave the IUCD and send to OBGY to remove(correct)
c. Leave IUCD
d. Do laparoscopy to see if there is ectopic pregnancy
e. Reassure the patient

144. Pregnancy test positive after;


a. One day post-coital
b. 10 days after lose menstrual cycle
c. 1week after lose menstrual cycle
d. 1week before the expected menstruation(correct)
145. 20 year old lady came to ER with history of right severe lower abdominal pain with
history of amenorrhea for about 6weeks. The most serious diagnosis of your differential
diagnosis could reach by;

a. CBC
b. ESR
c. U/S of pelvis(correct)
d. Plain X-ray
e. Vaginal swab for C/S

146. Deep laceration in the anterior aspect of the wrist, causing injury to the median
nerve. The result is

a. Vaginitis(correct)
b. Cystitis
c. CA of vagina
d. Urethritis

147. Treatment of bacterial vaginatis;

a. Ampicillin
b. Tetracycline
c. Metronidazole(correct)
d. Erythromycin

148. 35years G4P2+1, 1year history of irregular heavy bleeding. O/E; WNL. The most
diagnosis is;

a. Early menopause
b. Nervous uterus
c. Dysmenorrhea
d. DUB(correct)
e. Endometriosis

149. All are true about prenancy EXCEPT;

a. Ovarian site at 20%(correct)


b. Cause of death in 1st trimester
c. Doubling time of B-HCG
d. Can be diagnosed by laparoscopy
e. Empty uterus + HCG before 12weeks is diagnostic

150. Concerning obstructed labor, one is true;

a. Common in primigravida(correct)
b. Common in occipito-anterior position
c. Caput succedaneum and excessive molding are usual signd
d. Easily diagnosed before onset of labour
e. Oxytocin is used to induce labor

151. Hyperprolactinemia associated with all of these except;

a. Pregnancy
b. Acromegaly
c. OCP(correct)
d. Hypothyroidism

152. Pregnant teacher in her 20weeks of regnancy reported 2 of her students


developed meningitis. Prophylactic treatment;

a. Observe for signs of meningitis


b. Meningitis polysaccharide vaccine
c. Ciprofloxacine500mg OP once contraindicated
d. Ceftriaxone250mg IM or IV once
e. Rifampcin600mg BID for 2 days(correct)

153. All of these are causes of intrauterine growth restriction(IUGR) except;

a. Toxoplasmosis
b. CMV
c. Rubella
d. HSV II(correct)
e. Syphilis

154. What is most common cause of death in 1st trimester?

a. Ectopic pregnancy

155. In obstructed labour;

a. Moulding is prominent

156. Pregnant lady, 28weeks with chlamydia infection

a. Azithromycin
b. Erythromycin in pregnant, best treat partners if amox(correct)
c. Doxcycline

157. Patient before menstruation by 2-3days presented with depressed mood that
disappear by 2-3days after thre beginning of menstruation. Diagnosis?
a. Premenstrual syndrome

158. Female patient with itching in the vagina associated with the vaginal discharge.
pH=5, no trichomoniasis infection, pseudohyphae by culture. Diagnosis

a. Physiological discharge
b. Candida infection(correct)

159. Primigravida with whitish discharge, the microscopic finding showed


pseudohyphae. The treatment is;

a. Meconazole cream applied locally(correct)


b. Tetracycline
c. Metronidazole
d. Cephtriaxone

160. Female with monilial vaginal discharge. The treatment is ;

a. Meconazole cream for 7day(correct)


b. Fluconazole orally for one day
c. Metronidazole orally for 7days

161. Female patient presented with thick discharge color, no itching, vaginal
examination by spectrum normal, pH=4. What is the diagnosis?

a. Physiological discharge

162. Pregnant woman presented with a mass in her mouth, bleeding when brusging her
teeth, by examination mass is 3x2cm. What is the diagnosis?

a. Aphthous ulcer
b. Cancer
c. Granuloma(corret)

163. Young lady with pelvic pain and menorrhagia, examination showed mass. What is
the diagnosis?

a. Uterine fibroid(correct)
b. Adenpmyosis
c. Endometriosis

164. Postpartum hemorrhage management;


a. Oxytocin infusion(correct)
b. Mesoprostol
165. 38weeks pregnant lady with placenta previa, marginal with mild bleeding; cervix
2cm. How to manage;

a. C/S
b. Spontaneous delivery(correct)
c. Forceps delivery
d. Do amniotomy

166. Female patient with hiatialhernia. Which is correct;

a. It becomes more severe in pregnancy

167. Which heart condition is tolerable during pregnancy;

a. Eisenmenger syndrome
b. Aortic stenosis
c. Severe mitral regurgitation
d. Dilated cardiomyopathy with EF20%
e. Mitral stenosis and mital area is 1cm(or mm)

168. Cervicitis+ strawberry+ mucopurelnt yellow discharge cervix eroded+ friable. What
is diagnosis?

a. Trachimonus vaginitis(correct)
b. Chlamydia
c. Neissera gonorrhea

169. Female young with few tear vessicles on rose base and painful on valve;

a. Syphilis
b. HSV(correct)
c. Chancroid

170. Womean, 52years complain of loss of libido, dry vagina, loss of


concentration,weight gain since 10months or days, affecting marital state. What will you
give her;

a. Estrogen
b. Progesterone
c. Fluxatine

171. Female takes OCPs, comes with skin changes on the face;

a. Lupus lipura
b. Melasma(correct)
172. The most dangerous condition in menopause is;

a. Ovarian cancer
b. Endometrial cancer
c. Osteoporosis(correct)

173. Pregnant lady underwent U/S which showed anteriolateral placenta. Vaginal exam
, the examiner’s finger can’t reach the placenta;

a. Low lying placenta


b. Placenta previa totalis
c. Placenta previa marginalis
d. Placenta previa partialis
e. Normal placenta(correct)

174. 20years old, sexually active. She suffers from pain during intercourse. Urine
analysis was gram negative diplococci intracellular. Diagnosis;

a. Gonococcal sexual transmitted disease.

175. Most lethal infection for a pregnant woman;

a. Toxoplasmosis(correct)
b. HIV
c. Rubella
d. Measles

176. 32years old, have 2 children, did a pap smear that showed atypical squamous.
What is the next step?

a. Cone biopsy
b. Direct biopsy
c. Colscopy(correct)

177. Patient complain of dysmenorrhea+amenorrhea. Diagnosis;

a. Endometriosis(correct)
b. Endometritis
c. Polyp

178. Which of the following oral contraceptive drugs cause hyperkalemia?

a. Normethadone
b. Ethinyl estradiol(correct)
c. Seradiole

179. Early pregnant came to your clinic. Which is most bebeficial to do?

a. CBC
b. Urine pregnancy test
c. U/S(correct)
d. MRI
e. Blood grouping and Rh

180. 42years old pregnant lady came to you in the 2nd trimester asking to do screening
to detect down syndrome. What is the best method.

a. Triple screening
b. Amniocentesis(correct)
c. Cord blood sample
d. Choroinic villous sample.

181. Question about spontaneous abortion

a. 30-40% of pregnancies end with miscarriage


b. Most of the them happen in the 2nd trimester
c. Cervical assessment must be done(correct)

182. About fetal alcohol syndrome;

a. Placenta inhiobit the passage of alcohol


b. Will cause fetal retardation and facial features and other syndrome(correct)
c. It’s safe to drink wine and hard something once a week

183. Secondary amenorrhea;

a. Due to gonadal agenesis


b. Sheehan’s syndrome(correct)
c. It is always pathological

184. Patient with herpes in vagina. What is true?

a. Pap smear every 3years


b. C/S delivery if infected 2weeks before delivery(correct)

185. Female with atypical squamous cells of undetermined significance(ASCUS) on pap


smear, started 30day ttt with estrogen and told her to come back after 1week, and still
positive again on ;pap smear. What is next?
a. Vaginal biopsy
b. Endometrial biopsy(correct)
c. Syphilis serology

186. The most common antecedent cause of ectopic pregnancy is;

a. Salpingitis(correct)
b. Congenitally anomalous tube
c. Tubal surgery
d. Previous sterilization

187. Pregnant woman has fibroid. Whis is true?


a. Presented with sevee anemia
b. Likely to regress after pregnancy(correct)
c. Surgery immediately
d. Presented with antepartum hemorrhage

188. Pregnant with uterine fibroid, has no symptoms only abdominal pain. US showed
live fetus. What is the appropriate action to do?

a. Myomectomy
b. Hysterectomy
c. Pain management(correct)
d. Pregnancy termination

189. Female with dysuria, urgency and small amount of urine passed. She recieved
several courses of AB over the last months but no improvement. All investigations done,
urine and culture with CBC are normal. You should consider;

a. Interstitisl cystitis
b. DM(correct)
c. Cervical erosion
d. Candida albican

190. Normal puerperium;

a. It lasts for up to 4weeks


b. The uterus can’t be felt after the 1st week
c. Lochia stays red for 4weeks
d. Epidural analgesia cause urinary retention(correct)

191. The drug which is sed in seizures of eclamptic origin(pre-eclampsia);


a. Mg sulfate(correct)
b. Diazepam
c. Phenytoin
d. Phenobarbital
192. Female pregnant, previously she had DVT, you will now give her?
a. Warfarin
b. Heparin(correct)
c. Aspirin
d. Enoxaparin

193. Pregnant with HIV, the most accurate statement regarding risk of transmission of
HIV to baby is?

a. Likely transmitted through placenta


b. Through blood cord
c. Hand contamination
d. Breast feeding(correct)

194. Pregnant, on iron supplementation throughout her pregnancy for her anemia, now
she comes complaining of weakness and easy fatigability. Her hemoglobin7, MCV 60.
What is the diagnosis?

a. Iron deficiency anemia(correct)


b. Hypothyroidism
c. Vitamin B12 deficiency
d. Beta thalassemia

195. Pregnant, developed sudden left leg swelling. Best management is ;

a. Duplex
b. Rest
c. Heparin(correct)

196. Female complains of hypotension after she had C/S. What is the management?

a. IV Heparin and do Ctscan for PE(correct)


b. Broad spectrum antibiotic

197. 14years old female complains of irregular bleeding. Examination is normal sexual
character, normal vagina. What will you tell her?

a. If pregnancy test and blood is normal, this is not a physical illness(c0rrect)


b. Take FSH,LH test

198. Lady came with severe bleeding, she is nulligravida. HB is 10. By exam there is
blood on vagina. Management will be;

a. High dose of oral combined oral contraceptive pills(correct)


199. Chronic uses of estrogen association

a. Pulmonary embolism

200. Female presented with vaginal discharge, itching and on microscope showed
mycoleus cells and spores, this medical condition is most likely to be associated with ;

a. TB
b. Diabetes(correct)
c. Rheumatoid Arthritis

201. Trichomoniasis classically have;

a. Clue cells
b. Greenish frothy discharge(correct)

202. Trichomoniasis;

a. Associated with cytological abnormalities on pap smear


b. Associated with pregnancy and DM
c. Is a sexually transmitted parasite which cause pruitic discharge(correct)
d. May cause overt warts
e. Is diagnosed on a wet smear which reveals clue cells

203. Pregnant woman has allergy against sulfa, penicillin and another drug. Which is
safe for her ?

a. Nitrofuratoin(correct)
b. Cemitidine
c. Ciprofloxacin
d. Trimethoxazole

204. Female with –ve pap smear. You would advice to repeat pap every;

a. 6months
b. 12months(correct)
c. 18months
d. No repeat

205. Old female with itching of vulva, by examination there is pale and thin vagina, no
discharge, what is the management?
a. Estrogen cream(correct)
b. Corticosteroid cream
c. Fluconazole
206. Multipara lady with 1year history of stress incontinence. Your treatment;

a. Bulking of floor (correct)


b. Koplet exercise

207. Which of these increases during pregnancy?


a. Tidal volume
b. Functional residue
c. Total lung capacity
d. Residual volume
e. Dead long space

208. Post-delivery of a preterm baby, 25weeks with poor Apgar score. Good vital sign,
pH, pO2, pCO2. What is the first good step?

a. Send for photo therapy


b. Prenatal feeding
c. Give full oxygen(correct)

209. Preterm baby, 28weeks, resuscitated in delivery room, vitally stable, pH=7.35,
pCO2=42, pO2=63, then shifted to NICU. Wht will you do?

a. IV vancomycin
b. Phototherapy
c. IV bicarbonate infusion
d. Start entral feeding
e. Start glucose infusion(correct)

210. Old woman, she did a pap smear which was –ve then after 7years, she did another
pap smear which shows aquamous metaplasia undifferentiated. What is your next step?

a. Repeat a pap smear after 1 year


b. HPV testing
c. Colopscopy(correct)

211. Female patient complained of cyclic pelvic pain, which increases by defecation for
the last six months. Examination was normal with retroverted uterus. What is your
diagnosis?
a. Adenomyosis
b. Endometriosis(correct)
c. Polycystic ovary
d. Fibroid
211. In pregnancy;
a. Cardiac output will decrease
b. Cardiac output will increase more than non-pregnant(correct)

212. Female patient giving history of menorrhagia since last 3months, her HB=8. What
is the first action?

a. Endometrial biopsy
b. Hospitalization for blood transfusion(correct)

213. Old lady giving history of (postmenstrual symptoms), hot flushes. Best drug to
reduce these symptoms is;

a. Estrogen only
b. Progesterone only
c. Combined pills(correct)

214. 62years old female complaining of pruritis of pupic are, with bloody discharge. She
uses many treatment but no improvement,then she developed pea-shaped mass in
her labia. What will come to your mind as diagnosis?

a. Bartholin’s cyst(correct)
b. Bartholin gland carcinoma
c. Bartholin gland basses

215. Patient has a cyst on labia;

a. Bartholin’s cyst

216. The most common causes of prococious puberty

a. Idiopathic(correct)
b. Functional ovary cyst
c. Ovary tumor
d. Brain tumor
e. Adenoma

217. Lactating woman, 10days after delivery developed fever, malaise, chills tender left
breast with hotness and small nodule in upper outer quadrant with axillary LN.
Leukocytic count was 14x10/L. Diagnosis;

a. Inflammatory breast cancer


b. Breast abscess
c. Fibrocystic disease
218. Not used in the prevention of pre-eclampsia with protein uria and LL edema;
a. Admission and bed rest
b. Diuretics(correct)
c. Non-strss rest
d. Regular sonogram of baby

219. Woman complains of non fluctuated tender cyst for the vulva, came with pain in
the coitus and walking. Diagnosed Bartholin’s cyst. What is the treatment?

a. Incision and drainage(correct)


b. Refer to the surgery for excission (after you reassure her)
c. Reassure the patient
d. Give AB

220. Pregnant patient want to take varicella vaccine. What will you tell her?

a. That is a live vaccine(correct)


b. It’s okay to take it

221. Geststiom in NICU900g wgt... otherwise normal. What is to do?

a. Give her milk orally


b. Glucose infusion(correct)
c. Blood spectrum antibiotics

222. Pregnant lady has history of 2months GA, investigations: increased B-HCG, no fetal
parts ipn U/S. What is your diagnosis?

a. Trophoblastic disease

223. Pregnant lady in 3rd trimester, DM on insulin, patient complies to medication but
has hyperglycemic attacks, the common complication on fetus is;

a. Hypoglycemia(correct)
b. Hyperglycemia
c. Hypocalemia
d. Hyponatriemia

224. Pregnant lady 16 weeks GA. On U/S, fetus is small for age, P/E uterus size
12weeks. What is the diagnosis?

a. Choroinic carcinoma
b. Hydatiform mole
c. Tumor of placenta
225. Methylergonevine is in;

a. Maternal HTN

226. Not correct during management of labor;

a. Intensity of uterine contractions can be monitored manually


b. Maternal vital signs can vary relative to uterine contractions
c. Food and oral fluid should be witheld during active labor
d. Advisable to administer enema upon admission(correct)
e. IVF should be administered uopn admision

227. What requirements must be fufilled before instrumental delivery can be


performed?

a. Trained operator
b. Legitimate indication
c. Cervix dilated (correct)

228. In a vesicular mole;

a. B-HCG is lower than normal


b. Fundal height is lower than normal
c. Fetal heart can be detected
d. Ovarian cystt is a common association(correct)
e. Hypothyroid symptoms may occur

229. Which of these tests is mandatory for all pregnant women?

a. HIV
b. Hepatitis B
c. VRDL(veneral disease research laboratory)
d. All of them(correct)

230. Which of these suggests enormous ovarian cyst or more than ascities?

a. Fluid wave
b. Decrease bowel motion
c. Shifting dullness
d. Tympanic central, dullness lateral
e. Dullness central, tympanic lateral(correct)

231. All of these is true about IUGR EXCEPT;

a. Asymetric IUGR is usually due to congenital anomalies(correct)


b. IUGR babies are more prone to meconium aspiration and asphyxia
c. Inaccurate dating can caude misdiagnosed IUGR

232. Pernatal asphyxia could be all except;

a. Abruptio placenta
b. Hyper emisis gravidium(correct)
c. Pre-eclampsia

233. Healthy 28years old lady P1+0 presented with 6months amenorrhea. What is the
most likely cause?

a. Pregnancy(the most common cause of secondary amenorrhea is


pregnancy){correct}
b. Turner syndrome(cannot be because they have ovarian dysgenesis at fertility)

234. Action of contraceptive pills;

a. Inhibition of estrogen and then ovulation


b. Inhibition of prolactin
c. Inhibition of protozoa nby change in cervical mucosa
d. Inhibition of midcycle gonadotropins then ovulation(correct)
e. Inhibition of implantation of the embryo

235. 20year old married old lady presented with history of left lower abdominal pain
and amenorrhea for 6 weeks. The most appropriate investigation to role out serious
diagnosis is?

a. CBC
b. ESR
c. Pelvic U/S(correct)
d. Abdominal XR
e. Vaginal swab for culture and sensitivity

236. Anti-D Immunoglobulin, not given to a pregnant lady is;

a. 25-28weeks
b. Anti-D Ab titer of 1:8(correct)
c. After amniocentesis
d. After antepartum hemorrhage
e. After chorion villi biopsy

237. Blockage of first stage labour pain;

a. Block of the lumbosacral plexus afferent(correct)


b. Block of the lumbosacral plexus efferent
c. Block of the pudendal nerve
d. Block of sacral plexus

238. Premenstrual tension;

a. More in the 1st half of menses


b. 60% associated with edema
c. Associated with eating salty food(correct)
d. Menorrhagia

239. If a pregnant woman is eating well balanced diet, one of the following should be
supplied;

a. Ca2+
b. Phosphate
c. Vit. C
d. None of the above(correct)

240. Dysparenuria caused by all the following except;

a. Cervicitis(correct)
b. Endometriosis
c. Lack of lubricant
d. Vaginitis

241. All of the following are normal flora and should not be treated except;

a. Trichomonus(correct)
b. Candida
c. E. Coli
d. Fragmented bacteria

242. All the following should be avoided in pregnancy except;

a. Na+ valproate
b. Glibenclamide
c. Keflex(correct)
d. Septrin
e. Warfarin

243. Cord prolapse occurs in all except

a. Premature rupture of membranes


b. Preterm delivery
c. Oligohydramnios(correct)
d. Head high pelvis

244. Signs and symptoms of normal pregnancy except;

a. Hyperemesis(correct)
b. Hegar sign
c. Chadwick’s sign
d. Amenorrhea

245. In twins, all are true except;

a. Dizygote
b. In dizygote more twin-twin transfusion(correct)
c. Physical changes double time than single form
d. U/S can show twins

246. Breech presentation, all are true but;

a. Breech after 36weeks about 22%(correct)


b. Known to cause intercranial hemorrhage
c. Known with prematurity

247. In lactation, all are true except;

a. Sucking stimulates prolactin


b. Sucking cause release of oxytocin
c. Milk release decrease by overhydration(correct)

248. In Patients with post partum hemorrhage amd infertility, all can be found except;

a. Ballooning of sella turcica(correct)


b. Decreased Na
c. Hypoglycemia
d. Decreased T4
e. Decreased iodine uptake

249. Placenta previa, all true except;

a. Malpresentation
b. Head not engaged
c. Shock out of proportion of bleeding(correct)
d. Painless bleeding

250. Pelvic inflammatory disease, all true except;


a. Infertility
b. Endometriosis(correct)
c. Dyspareunia
d. Can betreated surgically

251. Recurrent abortion;

a. Genetic abnormality ‘’most common’’


b. Uterine abnormality
c. Thyroid dysfunction
d. Increased prolactin ‘’except’’

252. DIC occur in all except;

a. Abruptio placenta
b. Fetal death
c. DM(correct)
d. Pre-eclampsia

253. Pregnancy induced HTN, all are true except;

a. Ankle edema
b. Polyuria(correct)
c. Exaggerated reflex
d. RUQ pain

254. Pyelonephritis in pregnancy. All are true except;

a. Gentamycin is drug of choice(correct)


b. Abruptio placenta
c. E.coli common organism
d. Should be treated even for asymptomatic

255. Infertility. All true except;

a. Male factor present 24%


b. Normal semen analysis is >20,000,000
c. Idiopathic infertility is 27%
d. High prolactin could be a cause

256. The following drugs can be used safely during pregnancy

a. Septrin
b. Cephalexin
c. Tetracycline
d. Aminoglycoside
e. Cotrimoxazol

257. Primary amenorrhea due to;

a. Failure of canalization of mullarian duct


b. Kallmann syndrome
c. Agenesis
d. All of the above(correct)
e. None of the above

258. The following are risk factors of puerperal except;

a. Endometriosis(correct)
b. Cervical laceration
c. Anemia
d. Hemorrhage
e. Retained placenta

259. Pregnant 16year old, which of the following is the last likely to be a complication
of her pregnancy?

a. Anemia
b. Pelvic complication
c. Toxemia
d. Low birth wgt infant
e. Infant mortality(correct)

260. Indication of Hepatitis during pregnancy is?

a. WBC
b. Alkaline phosphatase(correct)
c. SGOT
d. BUN

261. BUN;

a. Need full investigation if persists more than 6months


b. Pregnancy should be considered
c. Prolonged use of contraceptive pill will increase risk of post pill amenorrhea
d. More common in women who had irregular periods(correct)

262. One drug cannot cross the placenta


a. Heparin(correct)
b. Warfarin
c. Aspirin

263. Vaginal trichomoniasis. All true except;

a. More in diabetic(correct)
b. Protozoal infection
c. Diagnosed by microscopic exam of vaginal smear

264. Toxemia in pregnancy. All are true except;

a. More in primigravida than multigravida


b. More in multiple pregnancy
c. Can progress rapidly to toxemia

265. Pre-eclempsia;

a. Commoner in multipara than primigravida


b. Mostly in diabetic
c. Headache and blurred vision(correct)
d. Progress very fast to eclempsia

266. Infertility due to endometriosis;

a. Progesterone
b. Danazole(correct)
c. Radiotherapy

267. Patient with history of prolonged heavy bleeding 2hours post partum; you will
give?

a. Ringers lactate
b. NS
c. NS+packed erythrocytes(PRBC) {CORRECT}

268. Alpha-fetoprotein increases in all except;

a. Myelomeningocele
b. Spina bifida
c. Encephalitis
d. Breech presentation(correct)
269. 25years old pregnant, presented with fever and sore throat(in flu season) then
she developed non-productive cough and dyspnea, she was extremely hypoxic. What
is the most likely diagnosis?

a. Pseudomonas pneumonia
b. Staph pneumonia(correct)
c. Strept pharyngitis
d. Viral pneumonia

270. 14years old girl complaining of painless vaginal bleeding for 2-4days every 3weeks
to 2months ranging from spotting to 2packs per day. She had secondary sexual
characteristics 1year ago and had her menstruation since 6months. O/E; she has
normal sexual characteristics, normal pelvic exam. Appropriate action;

a. OCP can be used(correct)


b. You should as for FSH and prolactin level

271. Pregnant with vaginal bleeding 2-3hours at 36weeks GA, has 3NVD. Important to
ask;

a. Smoking
b. Intercourse(correct)

272. Patient presented with PV bleeding. How can you differentiate between abruptio
placenta and spontaneous abortion

a. Oss discharge
b. Pain
c. Gush of blood(correct)

273. What is true regarding transdermal estrogen and OCP?

a. Transdermal needs less compliance in comparison with OCP(correct)


b. Transdermal caused more DVT
c. Transdermsl is less effective in contraception in compsrison to OCP

274. It should be 5% in the recurrence rate and 20% in monozygomatic twin

a. 8%(correct)
b. 2%
c. 10%
d. 20%

275. Woman with history of multiple intercourse had ulcer in cervix 1st line
investigation;
a. Pap smear(correct)
b. Cervical biopsy
c. Vaginal douch and follow up after 4weeks

276. Surveillance of patient on HRT includes all except;

a. BP
b. Breast examination
c. Glucose tolerance test(correct)
d. Pelvic exam
e. Endometrial sampling in the presence of abnormal bleeding

277. Age of menopause is predominantly determined by

a. Age of menerche
b. Number of ovulation
c. Body mass index
d. Socioeconomic ststus
e. Genetics(correct)

278. Pregnant lady with no fetal movement; platelets 7500. What is the diagnosis?

a. Autoimmune pregnancy

279. Lady with post-coital spotting, dysuria;

a. Chlamydia

280. 16 years old lady presented with a chief complaint of never having had a menstral
period.she had never had a pelvic exam. Physical exam reveals the following:
BP=110/70, Pulse=72, Wgt=60kg and Ht=172. The patient appears her tated age.
Axillary and pubic hair is scant. Breast are tanner stage IV. External genitalia are
normal female. A mass is palpable within the inguinal canal. Pelvic exam reveals an
absent cervix with the vagina ending in a blind pouch. The uterus and ovaries are
difficult to delienate. What is the most likely diagnosis?

a. Hypothalamic amenorrhea
b. Prolactin secreting adenoma
c. Polycystic ovarian syndrome
d. Turner syndrome
e. Androgen insensitivity syndrome(correct)

281. Confirmation of your diagnosis would be most readily obtained by ordering ;


a. Diagnostic laparoscopy
b. Pelvic ultrasound
c. Pelvic CT
d. Karyotype(correct)
e. MRI of pituitary

282. Karyotype is performed on the patient’s peripheral blood lymphocytes. The


karyotype is most likely?

a. 46 XX
b. 45 XX
c. 46 XY(correct)
d. 46 XX
e. 47 XXY

283. The hormone profile in this patient would include all of these except;

a. Elevated LH
b. Elevated estradiol for a male
c. Normal to elevated FSH
d. Normal to slightly elevated testosterone for a male
e. Normal testosterone for a female(correct)

284. The inguinal mass most likely represents

a. Uterus
b. Ovary with arteric follicles
c. Testis
d. Testes with hyperplastic leydig cells and no evidence of
spermatogenesis(correct)
e. Herniated sac containing a peritoneal content

285. The most long term treatment would be;

a. Total hysterectomy
b. Estrogen replacement therapy(correct)
c. Androgen replacement therapy
d. Oophorectomy

286. Without surgery, this patient is at risk of developing

a. Gonadoblastoma
b. Dysgerminoma(correct)
c. Neither
287. All of the following are true of this patient except;

a. H-Y antigen present


b. These patients are always sterile
c. Antimullarian hormone is present
d. Normal levela of dihydrotestosterone
e. Clitromegaly may develop later in life

288. All of these result from combined estrogen-progestin replacement therapy except;

a. Decrease in the risk of osteoporosis


b. Relief of vasomotor symptoms
c. Relief of dysparenuria
d. Increases the risk if coronary artery disease(correct)
e. Decreases the risk of coronary artery disease

289. All of these are known to increase yje risk of ostoporosis in the postmenopausal
women except;

a. Early menopause
b. Cigarette smoking
c. Low calcium intake
d. Sedentary lifestyle
e. Black race(correct)

290. Definitive therapy for hydaitidiform mole is most commonly;

a. Evacuation(correct)
b. Evacuation followed by methotrexate therapy
c. Abdominal hysterectomy
d. Evacuation followed by hysterectomy
e. Radiation

291. Evacuation of hydaitidaform mole may be complicated by;

a. Hemorrhage necessitating transfusion


b. Acute respiratory
c. Disease
d. Both(correct)
e. Neither

292. All are characteristics changes seen in menopase except;

a. Decreased body fat(correct)


b. Decreased skin thickness
c. Increased facial hair
d. Decreased collagen content in the endopelvic fascia

293. Following evacuation of a molar pregnancy, B-HCG titers will fall to undetectable
levels in about 90% of patient within

a. 2weeks
b. 4weeks
c. 8weeks
d. 1oweeks
e. 12-16weeks(correct)

294. Diagnosis of hydaitidiform mole can be made accurately on the basis of;

a. Elevated B-HCG
b. Pelvic U/S (correct)
c. Pelvic exam
d. Chest radiograph
e. Absence of fetal heart stone in a 16weeks size uterus

295. After the B-HCG titre becomes undetectable, the patient treated for hydaitidiform
mole should be followed with monthly titres for a period of

a. 3m0nths
b. 6months
c. 1 year(correct)

296. 25year ols G3P1 presented to the ER with lower abdominal crampy pain 6weeks
from her last normal period. She had significant bleeding but no passage of tissue.
What is the most likely diagnosis?

a. Incomplete abortion
b. Complete abortion
c. Missed abortion
d. Threatened abortion
e. Ectopic pregnancy(correct)

297. The most important step in this patient’s evaluation should be;

a. Sonography(correct)
b. Physical examination
c. CBC
d. Quantitative B-HCG
e. Detailed menstrual history
298. Transvaginal ultrasonography would moost likely reveal
a. Fetal heart motion
b. An intact gestational sac
c. A discrete yolk sac
d. A thickened endometrium with no gestational sac (correct)
e. Fetal heart motion in the adnexae

299. Ectopic pregnancy can be ruled out with a high degree of certainty if;

a. The patient has no adnexal tenderness


b. B-HCG level is less than 6000
c. The uterus measures 6weekssize on bimanual exam
d. An intrauterine gestational sac is observed(correct)
e. Tissue is observed in cervical os

300. Physical exam reveals the uterus to be about 6weeks size. Vaginal bleeding is scant
with no discernible tissue in the cervical os. There are no palpable adnexal masses.
The uterus is mildly tender. Ultrsonographic exams does not reveal gestational sac.
Which of these should be recommended?

a. Dilation and curettage


b. Culdocentesis
c. Observation dollowed by serial B-HCG determination (correct)
d. Diagnostic laparoscopy
e. Laparotomy

301. The most common presenting syndrome of ectopic pregnancy is;

a. Profuse vaginal bleeding


b. Abdominal pain(correct)
c. Syncope
d. Dysparenunia
e. Decreased pregnancy associated symptoms

302. If the above patient presented at 8weeks gestation and pelvic revealed unilateral
adnexal tenderness without discernible mass. Consideration should be given for;

a. Observation
b. Culdocentensis
c. Laparascopy(correct)
d. Dilation and curettage
e. Laparotomy

303. A majority of ectopic pregnancies occur in the;


a. Ampullary tube(correct)
b. Ovary
c. Isthmic tube
d. Cervix
e. Fimbriated (distal) tube

304. If the above described patient has had a previous term pregnancy prior to her
current ectopic pregnancy, her chances of subsequent intrauterine pregnancy would
be about;

a. 80%(correct)
b. 60%
c. 40%
d. 20%
e. <10%
305. A serum progesterone value <5ng/ml can exclude the diagnosis of a viable
pregnancy with a certainty of

a. 20%
b. 40%
c. 60%
d. 80%
e. 100%(correct)

306. In normal pregnancy, the value of B-HCG doubles every

a. 2 days(correct)
b. 4days
c. 8 days
d. 10days
e. 14days

307. Management of possible ruptured ectopic pregnancy would include all except;

a. Exploratory laparotomy
b. Diagnostic laparoscopy followed by observation
c. Partial salpingectomy
d. Total salpingectomy
e. Observation followed by methotrexate(correct)

308. Syndrome seen in preeclamptic women called HELLP syndrome is characterized by


all of the following except;

a. Elevation of liver enzymes


b. Low platelet count
c. Hemolysis
d. Prolongation of the prothrombin time(correct)

309. The most common prodromal sign or symptom in patient with eclampsia is;

a. RUQ abdominal pain


b. Edema
c. Headache(correct)
d. Visual disturbance
e. Severe hypertension

310. If a woman with pre-eclampsia is not treated prophylactically to prevent


eclempsia, her risk of seizure is approximately;

a. 1/10
b. 1/25
c. 1/75
d. 1/200(correct)
e. 1/500

311. The most consistent finding in patient with eclampsia is;

a. Hyper reflexia
b. 4+ proteinuria
c. Generalized edema
d. DBP>110mmHg
e. Convulsions(correct)

312. Appropriate responses to an initial eclamptic seizure include all of these except;

a. Attempt to abolish the seizureby administering 1M diazepam(correct)


b. Maintain adequate oxygen
c. Administer MgSO4 by either IM or IV route
d. Prevent maternal injury
e. Monitor the fetal heart rate

313. Eclampsia occuringprior to 20weeks gestation is most commonly seen in women


with;

a. History of chronoc hypertension


b. Multiple gestations
c. Gestational trophoblastic disease(mplar pregnancy)[correct]
d. History of seizure disorder
e. History of renal disease
314. Likely contributory mechanism of the anticonvulsant action of MgSO4 include all
these except

a. Neuronal calcium-channel blockade


b. Peripheral neuromuscular blockade
c. Reversal of cerebral arterial vasoconstrictions
d. Inhibition of platelets aggregation(correct)
e. Release of endothelial prostacyclin

315. Drugs that should be avoided during pregnancy include all of the following except

a. Cotrimox
b. Cephaeline(correct)
c. Na+ valproate
d. Doxicycline
e. Glibenclamide

316. All of these antihypertensive medications are considered safe for short term use in
pregnancy except;

a. Captopril(correct)
b. Methyldopa
c. Hydralazine
d. Nifedipine
e. Labetalol

317. Which is true about gonococcal infection?

a. Less common in females with IUCD


b. Causes permanent tubal blocking(correct)
c. No need for laparoscopy for further investigation

318. The reason to treat severe c hronic hypertension in pregnancy is to decrease the

a. Incidence of IUGR
b. Incidence of placental abruption
c. Incidence of pre-eclampsia
d. Risk of maternal complications e.g stroke(correct)

319. Which of these does not cross the placenta

a. Heparin(correct)
b. Chloramphenicol
c. Tetracycline
d. Warfarin
e. Diazepam
f. Aspirin

320. Risk factors for HSV2 in infant include all except;

a. Cervical transmission is commoner than labial transmission


b. Maternal antibodies for HSV1 protectsagainst HSV2(correct)
c. Maternal first episode is of greater risk than for infants
d. Head electrodes increases the risk of infection

321. Pregnancy induced hypertension. All true except;

a. Use of birth control pills increases the risk(correct)


b. Common in primigravida

322. Vomiting in pregnancy. All true except;

a. Hospital admission causes it(correct)


b. More in molar pregnancy
c. More in pregnancy induced hypertension

323. Most of the cause of infection;

a. Anemia which is most probably the cause during pregnancy


b. Retained placenta(correct)
c. Hemorrhage during pregnancy
d. Endometriosis

324. One of these drugs is safe in pregnancy;

a. Metronidazole is unsafe in 1st trimester


b. Chloramphenicol in last trimester
c. Erythromycin estolate is safe in all trimesters(correct)
d. Nitrofurantoin

325. Vulvovaginal candidiasis

a. Causes mucopurulent cervicitis


b. Frequently associated with systemic symptoms
c. May be diagnosed microscopically by mixing discharge with KOH(correct)
d. Is treated with doxycycline
e. Is one of the sexually transmitted infection

326. Bacterial vaginosis


a. Is a rare vaginal infection
b. Is always smptomatic
c. Is usually associated with profound inflammatory reacrion
d. Causes fishy discharge which results from bacterial amine production
e. Is treated with clotrimazole

327. Chlamydia trachomatis infections;

a. Are commonly manifest as vaginal discharge


b. Pap smear usually suggests inflammatory changes
c. Infection in the male partner present as urethritis
d. May ascend into the upper genital tract resulting in tubal occlusion
e. All of the above

328. Progestin only contraceptive pills;

a. Suppress ovulation
b. Increase cervical mucus
c. Associated with inceased incidence of breakthrough bleeding(correct)
d. May cause menorrhagia

329. Non-contraceptive use of combined oral contraception include;

a. Menorrhagia
b. Primary dysmenorrhea
c. Functional small ovarian cyst
d. All of the above(correct)

330. Possible mechanisms of action of intrauterine contraceptive device

a. Inhibition of implantation
b. Alteration of endometrium
c. Suppression of ovulation
d. All of the above(correct)

331. An Rh –ve ABO incompatible mother delivers an Rh +ve infant at term and does
not recieve Rh immune globulin. The probability of detection of anti-D antibody during
her next pregnancy ia about;

a. 2%
b. 5%
c. 10%
d. 15%
e. 16%(correct)
f. 25%
332. The class of antibody responsible for hemolytic disease of the newborn is;

a. Ig A
b. Ig G(correct)
c. Ig M
d. Ig E
e. Ig D

333. All of these are seen in uterus with autoimmune hypdors except;

a. Anemia
b. Hyperbilirubinemia
c. Kenicterus
d. Extramedullary hematopoiesis(correct)
e. Hypoxia

334. An Rh-ve woman married to an Rh+ve man should recieve Rh immune globulin
under which conditions?

a. Ectopic pregnancy
b. External cephalic version
c. Both(correct)
d. Neither

335. The most common cause of polyhydramnios is;

a. Immune hydrops
b. Non immune hydrops
c. Diabetes
d. Factors which impair fetal swallowing
e. Idiopathic(correct)

336. Generally accepted cut off values for plasma glucose on 1000gm, 3-hour glucose
tolerance test in pregnancy( according to the National Diabetes Goup) include all the
following except

a. Fasting glucose >90mg/dl(correct)


b. 1hour value 190mg/dl
c. Fasting glucose 105mg/dl
d. 2hour value 165mg/dl
e. 3hour value 145mg/dl

337. The prevalence of gestational diabetes in the general population is about;

a. 2%
b. 4%(correct)
c. 8%
d. 15%
e. 20%

338. Normal pregnancy 2nd trimester is characterized by all the following except

a. Elevated fasting plasma glucose


b. Decreased fasting plasma glucose(correct)
c. Elevated postprandial plasma insulin
d. Elevate postprandial plasma glucose
e. Elevated plasma triglycerides

339. Gestational diabetes is associated with;

a. Increased risk of spontaneous abortion


b. Increased risk of fetal cardiac malformation
c. Intrauterine growth restriction
d. Decreased head circumference-abdominal circumference ratio(correct)

340. Infants of mothers with gestational diabetes have an increaes risk of ;

a. Hpoglycemia
b. Hyperglycemia
c. Hypocalcemia
d. Hyperbilirubinemia
e. Polycythemia

341. Gestational diabetes is associated with an increased risk of all of the following
except;

a. C/S
b. Shoulder dystocia
c. Fetal macrosomia
d. Intrauterine fetal death
e. Intrauterine growth restriction(correct)
342. Infants of mothers with geststional diabetes are at increased risk of becoming;
a. Obese adults
b. Type II diabetes
c. Neither(correct)
d. Both
343. Control of gestational diabetes is accomplished with all these except;
a. Insulin
b. Diet
c. Oral hypoglycemic agent(correct)
d. Exercise

344. Compared with TypeII diabetes, Type I diabetes is associated with all of these
except;

a. Greater incidence of pre-eclampsia


b. Greater incidence of preterm delivery
c. Greater risk of maternal hypoglycemia
d. Greater risk of maternal diabetic ketoacidosis
e. Reduced risk of intrauterine growth restriction(correct)

345. Classical characteristics for genital herpes;

a. Painful ulcers and vessicles

346. Which of these is true regarding infertility

a. It is failure to conceive within 6months


b. Male factor > female factor
c. It could be due to high prolactin level(correct)
d. Rare to be due to anovulation
e. Only diagnosed by HSG

347. 32years old female patient presented with irregular menses, menses occur every
2months. O/E; everything is normal. Which of these is the least important test to ask
about first?

a. CBC
b. Pelvis U/S
c. Coagulation profile(correct)
d. DHES

348. Old female came with scales around her areola, she took steroid but no benefit.
On examination; normal and no masses. What is your next step?

a. Antibiotics
b. Antifungal
c. Mammography(correct)

349. In pregnant women;

a. Sulphonide not cause neonatal jaundice


b. Methyldopa contraindicate
c. Reflux esepggitis cause anemia(correct)
350. 19years old female with depression, anxiety, mood swinging, affecting her life. She
experience like this symptom every month before menstruation. What is the most
approved treatment?

a. SSRIs (correct)
b. Perogestron pach
c. OCP
d. Progestron tampon

351. Pregnant ladydevelops HTN. Drug of choice for HTN in pregnancy is?

a. a-methyldopa (correct)
b. hydralazine
c. thiazide
d. b-blocker

352. 35years prime, 16weeks gestation, PMH coming for her 1st checkup. She is excited
about her pregnancy, no history of any previous disease. Her BP after since rest was
160/100. After 1week her BP was 154/96. Most likely diagnosis;

a. Pre-eclampsia
b. Chronic HTN
c. Chronic HTN with superimposed pre-eclampsia
d. Transit HTN(correct)

353. Contraindication of breast feeding

a. HIV

354. Hemophilius ducreyi, asking for treatment. Give for;

a. All sexual partners


b. Symptomatic sexual partner
c. Family contact

Good luck!!!
DERMATOLOGY
1. Patient comes with history of tinea capitis. Treatment is:
a. Tar shampoo
b. Fluconazole
2. Child has fever and malaise and develops a popular rash that later becomes vesicular and
crusted
a. Varicella Zoster
3. Man went on vacation and noticed a white patch in his chest which later became clearer
after getting a sun tan that was spread on his chest. What is the diagnosis?
a. Pityriasis versicolor
b. Vitiligo
c. Pityriasis rosacea
4. Male with itching in groin with erythematous lesions and some have clear centres, The
diagnosis is:
a. Psoriasis
b. Tinea cruris
c. Erythrasma
5. Patient presents with midface pain, erythematous lesions, and vesicles on periorbital area
and forehead. Pain is mostly in the nose. Diagnosis is:
a. Rosella
b. HSV
c. HZV
6. Treatment of noninflammatory acne:
a. Retinoic acid
7. Treatment of comedones:
a. Topical retinoid
8. Treatment of papules or pustules:
a. Topical benzoyl peroxide plus topical antibiotics, mainly clindamycin or erythromycin
9. Treatment of severe acne:
a. Steroid injection or oral anitbiotics such as tetracycline or erythromycin may be
added
10. Baby with white papules in his face. What is your action?
a. Reassure mother that it will resolve spontaneously
b. Antibiotics
11. Patient with pustules, papules and telangiectasia lesions. Diagnosis is:
a. Rosacea
12. 15 year old boy with clear-centred, peripherally red patch in right lower leg; no fever.
Diagnosis is:
a. Contact dermatitis
b. Tinea corporis
c. Lyme disease
13. Mother brought baby with complaints of diaper rash. She has used corn starch, talc powder,
zinc ointment and 3 different types of corticosteroids with no benefit. The rash was well-
demarcated and scaly with satellite lesions. Most likely diagnosis is:
a. Candida rash
b. Seborrhoeic dermatitis
c. Allergic contact dermatitis
14. A female patient with wheals over the skin with history of swollen lips. Diagnosis is:
a. Chronic urticaria with angioedema
b. Solar dermatitis
c. Contact dermatitis
d. Cholinergic dermatitis
15. A child with honeycomb crust lesions. Culture showed Staph aureus. Diagnosis is:
a. Impetigo
16. Patient presented with 6 week history of itching and redness all over the body with wheals.
Diagnosis:
a. Chronic urticaria
b. Solar urticaria
c. Allergic urticaria resolved after 24-72 hours
17. Angioedema can be due to:
a. B-blocker
b. ACEI
18. Which of the following reduces the risk of post-therapeutic neuralgia:
a. Corticosteroid only
b. Valacyclovir only
c. Corticosteroid and valacyclovir
19. Examination of a nrewborn shows papules or pustules with erythematous base
a. Transient neonatal pustular melanosis
b. Erythema toxicum neonatarum
20. Patient with coloured pustules around the mouth. Herpes simplex type 1 virus was isolated.
Treatment is:
a. Oral antiviral
b. IV antiviral
c. Supportive
21. Treatment of herpes zoster in ophthalmic division:
a. Oral acyclovir alone
b. Acyclovir and prednisolone
c. Prednisolone
d. IV acyclovir
22. Treatment of scabies:
a. Permethrin
23. Male came with vesicles on forehead. To prevent post-herpetic,:
a. Oral acyclovir
b. Steroid
c. Oral acyclovir and steroid
d. Varicella vaccine
24. Patient with 2cm dome-shaped mass on dorsum of the hand covered with keratin. Most
likely diagnosis is:
a. Basal cell carcinoma
b. Malignant melanoma
c. Keratoacanthoma
25. Patient with haemorrhagic lesions in the mouth and papules in the face and back. He had
SOB, fever, cough and mediastinal mass. Diagnosis is:
a. Kaposi sarcoma
26. Male patient with hair loss started as fronto-temporal and moving forward the vertex.
Diagnosis is:
a. Androgenic alopecia
b. Tinea capitis
27. Rash all over the body except the face after a week of unprotected sexual intercourse
a. Charcoid
b. Secondary syphilis
28. Patient complaining of hypopigmented skin and nerve thickening. Diagnosis is:
a. Leprosy
29. Child with fever and malaise then develops popular rash, which later becomes vesicular and
crusted
a. Varicella zoster
30. Acanthosis nigricans associated with
a. PCOS
31. Patient with cystic nodule (acne) and scars. Best treatment:
a. Retinoin
b. Erythromycin
c. Doxycycline
32. Patient has diarrhoea, dermatitis and dementia:
a. Pellagra
33. Dermatomyositis. True statement:
a. Distal muscle weakness
b. Underlying malignancy
c. Generalized skin rash
34. 27 year old man with asymmetric oligoarthritis involving knee and elbow and painful oral
ulcer> 10years comes with arthritis and mild abdominal pain. Diagnosis is:
a. Behcet’s disease
b. SLE
c. Regional enteritis
d. Ulcerative colitis
e. Whipple’s disease
35. Dermatomyositis:
a. Proximal muscle weakness
b. Proximal muscle tenderness
36. Old male having back pain presents with vesicles from back to abdomen:
a. VSV
37. Hair loss is a side effect of the following medication:
a. Phenytoin
b. Carbamazepine
c. Valproic acid
d. Diazepam
38. Patients with symptoms of blepharitis and acne rosacea. Best treatment is:
a. Doxycycline
b. Erythromycin
c. Ceftriaxone
39. Child with fever and runny nose, conjunctivitis and cough. He then developed
maculopapular rash on his face, which descended to involve the rest of his body
a. EBV
b. Coxsackie virus
c. Rubella
d. Vaccini virus
40. Folliculitis treatment is:
a. Topical steroid
b. PO steroid
c. PO antibiotics
d. Topical AB
41. Most common association with acanthosis nigricans:
a. Hodgkins lymphoma
b. NHL
c. DM
d. Insulin resistance
e. Internal malignancy
42. Middle aged man with black spots on his thigh for years becoming darker with bloody
discharge. Best treatment is:
a. Wide excision
b. Incisional biopsy
c. Cryotherapy
d. Radiotherapy
e. Immunotherapy
43. Patient has symptoms of infection, desquamation of the hands and feet; BP 170/100mmHg.
Diagnosis is:
a. Syphilis
b. Toxic shock syndrome
c. Scarlet fever
44. Patient with early rheumatoid arthritis. Management to decrease limitation of movement:
a. Do not use steroids or analgesics
b. Use DMARDs like methotrexate or anti-TNF, hydroxychloroquine
45. Patient presented with bullae in his foot. Biopsy shows subdermal lysis, fluorescent stain
showed IgG. Diagnosis :
a. Bullous epidermolyisis
b. Pemphigoid vulgaris
c. Herpetic multiform
d. Bullous pemphigoid
46. 2 month old with scaling lesion on scalp and forehead. Diagnosis:
a. Seborrhoeic dermatitis
b. Erythema multiforme
47. Henoch-Scholein purpura affects:
a. Capillaries
b. Capillary and venule
c. Arteriole, capillary and venule
d. Artery to vein
48. Child with multiple painful swellings on the dorsum of hands, feet, fingers and toes; CBC
shows Hb=7; RBCs on peripheral smear are crescent-shaped. Long-term care:
a. Corticosteroids
b. Penicillin V
c. Antihistamines
49. Patient living in cold climate for a long time notices brown scaly lesion on his chest. The
lesions become hypopigmented when he moved to hot climate. Diagnosis:
a. Psoriasis
b. Vitiligo
c. Pityriasis versicolor
50. Urticaria; all true except:
a. Can be part of anaphylactic reaction
b. Not always due to immune reaction
c. Always due to deposition of immune-complexes in the skin (due to increased
permeability of capillaries)
d. Due to ingestion of drugs
e. Due to ingestion of strawberries
51. Neonate baby with rash over face and trunk, with blister formation. Diagnosis:
a. Rythema toxicum
52. Presence of vesicles, bulla and erythema on chest skin. Treatment is:
a. Acyclovir cream
b. Betamethasone cream
c. Floclvir
d. Erythromycin
53. The following drugs can be used for acne treatment except:
a. Ethinyl estradiol
b. Retin A
c. Vitamin A
d. Erythromycin ointment
e. Azelenic acid
54. Patient with scales in hair margin and nasal fold and behind ear with papules and irregular
erythema. Treatment is:
a. Nizoral cream
b. Atovit
c. Acyclovir
d. Antibiotic tetracycline or topical flagyl
55. Patient with erythematous lesions and vesicles on periorbital area and forehead with pain
around the nose. Diagnosis is:
a. Roseola
b. HSV
c. Herpes zoster
56. Seborrhoeic dermatitis is caused by:
a. Pityrosporum ovale
57. Patient complains of back pain and hypersensitivity of the skin of the back. Examination
reveals rashes on the back, tender, with a red base and distributed in belt-like pattern.
Diagnosis is:
a. Herpes zoster
b. CMV
58. Blistering skin rash is a feature of the following except:
a. Erythema herpiticum
b. Erythema multiforme
c. Erythema nodosum
d. Sulphonamide allergy
59. Scabies infestation; all true except:
a. Rarely involve head and neck
b. 5% indane is effective
c. Benzobenzoates are equally effective as 5% indane
d. Itching occurs 1 week after infestation
60. Dysplatic nevus syndrome; all are true except:
a. Autosomal dominant
b. Answer not written
61. Psoralin ultraviolet ray A (PUVA); all are true except:
a. Useful in vitiligo
b. Contraindicated in SLE
c. Used to treat some childhood intractable dermatosis
d. Increases the risk of basal and squamous cell carcinoma
62. Patient with eruptive purpuric rash, hepatosplenmegaly:
a. EBV infection
63. A lady with 9 weeks history of elevated erythematous wheals over all her body with lip
swelling. No history of recent travels, food allergy or drug ingestion. Diagnosis is:
a. Chronic angioedema and urticarial
b. Contact dermatitis
c. Solar dermatitis
d. Cholinergic dermatitis
64. Patient with acne take retinoids for management of acne. Side effect is:
a. No choices written
65. 70 year old man c/o fever, vesicular rash over forehead. Management is:
a. IV antibiotics
b. IV antivirals
c. Acyclovir
66. Patient has painful lesion. Diagnosis is:
a. Heroes zoster
b. Folliculitis
c. Cellulitis
67. Athlete who jogs on daily basis presented with groin rash and erythema:
a. Topical antibiotics
b. Topical antifungals
c. Topical steroids
68. 42 year old man presented with sudden eruptions all over the body with palms and feet.
Most likely diagnosis is:
a. Syphilis
b. Erythema nodosum
c. Erythema multiforme
d. Fixed drug eruption
e. Pityriasis rosacea
69. 10 year old boy presented with a 5 day history of skin lesion which is scaly and yellowish.
Diagnosis is:
a. Tinea corporum
70. Photo shows erythema at lower abdomen, groin and thighs. Diagnosis is:
a. Erythema
b. Seborrhoeic dermatitis
c. Tinea cruris
71. Children with eruption within 5 days on all skin:
a. Varicella
b. Eryhrema nodosum
c. Erythema multiforme
d. Fixed drug eruption
72. Sun burn hypertensive patient on hydralazine beside using sun protective:
a. Discontinue anti-hypertensive
b. Daily paths
c. Use mink oil
d. Avoid sun exposure
e. Frequent paths
73. Pituitary adenoma secretes:
a. ACTH
b. FSH
c. Prolactin
74. 32 year old patient presents with a mole; his father had moles. He now has metastasis to his
lungs, bones and liver. What will come to your mind about the malignant change of the
mole:
a. Irregular border
b. Presence in the thigh
c. Homogenous colour.
75. Nasal pain and rash:
a. Rosea
76. Picture of wart in hand:
a. HPV
77. Sunburn not responding to antisun cream and has spent much time near the sea (taking cold
showers after returning, give him prednisone orally)
a. Antifungal tinea versicolor
78. Erythema nodosum:
a. Painful red nodules
79. Child with eczema flareup. He is on steroids and having itches that disturbs his sleeping:
a. Give antihistamines
b. Topical steroids
80. Lichen planus, most common site is:
a. Scalp
b. Neck
c. Knee
d. Buttocks
81. Child with hair loss in the temporal area with microscopic finding. Diagnosis is:
a. Alopecia
b. Kerion
82. Acne topical antibiotic:
a. Clindamycin, if not inflammatory
b. Benzoyl or topical retinoic acid, if inflammatory
83. Female with problem in school-manual removal of her hair
a. Trichotillomani
84. Best treatment in acne rosea:
a. Amoxicillin
b. Clindamycin
c. Erythromycin
d. Doxycycline
e. Metronidazole then tetracycline
85. Picture of skin with purple flat-topped polygonal papules. Diagnosis:
a. Lichen planus
86. Male patient with scaly fine popular rash on front of scalp, nose and retroauricular area.
Treatment is:
a. Ketoconazole cream
b. Oral augmentin
87. Xanthoma:
a. On lateral aspect of upper eyelid
b. Hard plaque
c. Around arterioles
d. Not related to hyperlipidaemia
e. Deposited in dermis
88. 23 year old with history of URTI develops ecchymosis. Treatment is:
a. Local antibiotics
b. Local antivirals
c. Steroids
89. 2 month old infant with white plaue on tongue and greasy. Past history of chlamydia
conjunctivitis after birth treated by clindamycin. Treatment is:
a. Oral nystatin
b. Topical steroids
c. Topical acyclovir
d. Oral tetracycline
90. Newborn came with red lump on left shoulder. It is
a. Haemangioma
91. Patient presented with blepharitis and acne rosea but no keratitis. Best treatment:
a. Topical chloramphenicol
b. Topical gentamicin
c. Oral doxycycline
92. Oral and genital ulcer with arthritis:
a. Behcet disease
b. Syphilis
c. Herpes simplex
93. Most effective treatment for rosacea:
a. Clindamycin
b. Erythromycin
c. Topical steroids
94. Known case of ulcerative colitis with erythematous rash in lower limb. Most likely diagnosis:
a. Erythema nodosum
95. Known ulcerative colitis coming with skin lesions around the tibia with irregular margins.
Diagnosis is:
a. Pyoderma gangirenosum
96. 35 year old smoker with snow white patch on the tongue. Management is:
a. Antibiotics
b. Excisional biopsy
c. Close observation
97. Child with piece of glass, beans and battery deep in ear canal. What to do:
a. No irrigation
b. Best pick with forceps
98. All are true in black hairy tongue, except:
a. Hydrocortisone can be used
b. Advice patient not to brush his tongue
99. Hyperkeratotic scaly lesion over the extensor surface of knee and elbow. What to do to
avoid exacerbation:
a. Steroid
b. Avoid sun exposure
c. Avoid trauma
100. Baby with red macules and dilated capillary on the right side of the face:
a. Sturge-Webber syndrome or nevus flammeus
b. Cavernous haemangioma
101. 10 year old patient with erythematous scaly areas pruritic in face, scalp and flexor areas.
a. Atopic dermatitis
102. Type of acne pustule with discharge:
a. Inflammatory
103. Second degree burn in face and neck
a. Hospitalization
104. Face with red areas at angle of nose with erythema and scales on this area, chest and scalp:
a. Scabies
b. Atrophic dermatitis
c. Seborrhoeic dermatitis
105. Child with red rash on flexor surfaces:
a. Atopic dermatitis
106. Child with round palpable red rash on his right leg, no pain or itching for a long time:
a. Granuloma annular
b. Tinea corpora
c. Erythema nodosum
d. Migratory
107. Goal of early management of inflammatory acne:
a. To prevent physical scar
108. Laser therapy in derma (PUVA)
a. Used in treatment o eczema and psoriasis
109. An aged patient develops popular rash over buttocks. Examination shows a sinus discharge
at anal cleft. Diagnosis is:
a. Furunculosis
b. Hidradenitis suppurativa
110. Female with acne not responding to steroids and antibiotics. You decide to give her Ricotan;
what will you tell her before giving her this medication:
a. Causes birth defect
b. Increase in acne before decreasing it
111. Asthma + skin lesions:
a. Atopic dermatitis
112. Female with red rash under breast; after wash this rash with moist; what give:
a. Topical antibiotics
b. Antifungal powder
c. Solution
d. Steroids
113. Patient with family history of allergy has scaling skin and itching on face and antecubital
fossa. Diagnosis:
a. Seborrhoeic dernmatitis
b. Contact dermatitis
c. Atopic eczema
114. Young female has vulvar irritation. Her doctor advises her to stop bubble baths, but the
irritation continues. On examination, the irritation is waxy. Diagnosis:
a. Atopic dermatitis
b. Contact dermatitis
c. Linch simplex
d. Linch simplex chronicus
115. Baby with vesicles on face and honeycomb crust. Causative organism:
a. Staph aureus
116. Classical characteristic for genital herpes:
a. Painful ulcers and vesicles
117. Patient with cystic nodules and scars. Best treatment:
a. Retinoid
b. Erythromycin
c. Doxycycline
118. Lichen planus most commonly found in:
a. Scalp
b. Knee
c. Buttocks
d. Mouth
GASTROENTEROLOGY MCQs

1. Woman complaining of burning retrosternal pain with normal ECG what


is the treatment?
o PPI (Proton Pump Inhibitor)
2. 15 years male with history of 3 days yellow sclera, anorexia, abdominal
pain, LFT: T.bilirubin=253 Indirect=98 ALT=878 AST=1005, what is the
diagnosis?
o Gilbert disease
o Infective hepatitis
o Obstructive jaundice
o Acute pancreatitis
o Autoimmune hepatitis
3. Middle age woman presented with upper abdominal pain, increase by
respiration. On examination temperature 390C, right hypochondrial
tenderness, her investigations: Bilirubin & ALT normal, WBC 12.9, your
next step is:
o Chest x-ray
o Abdominal ultrasound
o Serum amylase
o ECG
o Endoscopy
4. Gastric lavage can be done to wash all of the following except:
o Drain cleanser
o Vitamin D
o Diazepam
o Aspirin
5. Drug addict swallowed open safety pins since 5 hours, presented to the
ER, x-rays showed the foreign body in the intestine. Which is the best
management:
o Shift to surgery immediately
o Discharge and give appointment to follow-up
o Admit and do serial abdominal x-rays and examination
o Give catharsis: MgSo4 250mg
6. Patient with hepatosplenomegaly and skin bruises and cervical mass
what is the initial investigation:
o Bone marrow
7. Which of the following is an indication of surgery in Crohn’s disease:
o Internal fistula OR intestinal obstruction
8. What is the contraindicated mechanism in a child swallowed a bleach
cleaner solution:
o Gastric lavage
9. Patient with vomiting and diarrhea and moderate dehydration, how to
treat:
o ORS only
10. Initial investigation in small bowel obstruction:
o Erect & supine abdominal X-ray
11. In which group you will do lower endoscopy for patients with iron
deficiency anemia in with no benign cause:
o Male all age group
o Children
o Perimenopausal women & male more than 59 years
o Women + OCP
12. Elderly women present with diarrhea, high fever & chills, other physical
examination is normal including back pain is normal, Diagnosis:
o Pyelonephritis
o Bacterial gastroenteritis
o Viral gastroenteritis
13. Patient presented to the ER with diarrhea, nausea, vomiting salivation,
lacrimation and abdominal cramps. What do you suspect?
o Organophosphate poisoning
14. Child with garlic smell:
o Alcohol toxicity
o Organophosphate poisoning
o Cyanide toxicity
15. Treatment of pseudomembranous colitis
o Metronidazole
o Vancomycin
o Amoxicillin
o Clindamycin
16. Patient had HBsAB +ve but the rest of the hepatitis profile was
negative. The diagnosis is:
o Immunization from previous infection, past exposure or
vaccination
o Carrier state
o Chronic hepatitis
o Active infection
17. 24 year old man presented with a4 month history of diarrhea with
streaks of blood & mucous. Ulcerative colitis was confirmed by
colonoscopy. The initial therapy for this patient:
o Oral corticosteroid
o Azathioprine
o Infleximabe
o Aminosalicylic acid
o Sulfasalazine
18. Which of the following organisms can cause invasion of the intestinal
mucosa, regional lymph node and bacteremia:
o Salmonella
o Shigella
o E. coli
o Vibrio cholera
o Campylobacter jejeni
19. Patient presented with severe epigastric pain radiating to the back. He
has past hx of repeated epigastric pain. Social history: drinking alcohol.
What’s the most likely diagnosis:
o MI
o Perforated chronic peptic ulcer
20. A female patient has clubbing, jaundice and pruritus. Lab results
showed elevated liver enzymes (Alkaline phosphatase), high bilirubin,
hyperlipidaemia and positive antimitochondrial antibodies. What’s the
most likely diagnosis:
o Primary sclerosing cholangitis
o Primary biliary cirrhosis
21. Patient came recently from Pakistan after a business trip complaining of
frequent bloody stool. The commonest organism causes this
presentation is:
o TB
o Syphilis
o AIDS
o Amebic dysentery
o E. coli
22. Erosive gastritis:
o Happened within one week of injury
o Happened within 24 hrs of injury
23. Patient with acute abdomen you will find:
o Rapid shallow breath
o Rapid prolonged breath
24. About hepatitis vaccine
o 3 doses only
25.Patient took high dose of acetaminophen presented with nausea &
vomiting, investigation shows increase alkaline phosphatase and
bilirubin, which organ is affected?
o Brain
o Gastro
o Liver
26. Old patient with cramp abdominal pain, nausea, vomiting and
constipation but no tenderness DX
o Diverticulitis
o Colon cancer
o Obstruction
27. Old male patient came with fever, abdominal pain, diarrhea, loss of
weight, positive occult blood, labs shows that the patient infected with
streptococcus bovis, what will you do?
o Give antibiotic
o ORS
o Abdominal X-ray
o Colonoscopy
o Metronidazole
28. Patient came with chest pain, burning in character, retrosternal,
increase when lying down, increase after eating hot food, clinical
examination normal, what is the diagnosis?
o MI
o Peptic ulcer
o GERD
29. Benign tumors of stomach represent almost:
o 7%
o 21%
o 50%
o 90%
30. 40year old with mild epigastric pain and nausea for 6 months,
endoscopy shows loss of rugal folds, biopsy shows infiltration of B
lymphocytes, treated with antibiotic, what is the cause?
o Salmonella
o H. pylori
31. All of the following exaggerate the gastric ulcer except
o Tricyclic antidepressant
o Delay gastric emptying
o Sepsis
o Salicylates
o Gastric outlet incompetence
32. Old patient with history of recent MI complaining of severe abdominal
pain, distention, bloody diarrhea, slightly raised serum amylase.
Diagnosis is:
o Ischemic colitis
33. Old patient with positive occult blood in stool
o Colonoscopy
34. Adult patient with history of sickle cell anemia, he at risk of
o Infarction
35. After dinner 4 of family members had vomiting & diarrhea. What is the
causative organism?
o Salmonella
o Staphylococcus
o C. diff
36. Vitamin C deficiency will affect
o Collagen synthesis
o Angiogenesis
o Epithelization
o Migration of macrophage
37. Patient with perianal pain, increase during night and last for few
minutes
o Proctalgia fugax
o Ulcerative colitis
38. Young patient came with peptic ulcer. Which of the following doesn’t
cause it:
o Sepsis
o Delayed gastric emptying
o TCA
o Aspirin
o Pyloric sphincter stricture
39. Drug abuser, showed RNA virus what is the diagnosis:
o HBV
o HCV
o HEV
o HDV
40. Patient with cirrhosis, ascites, lower limb edema best to give:
o Thiazide
o Spirinolactone
41. Young male known case of sickle cell anemia presented with abdominal
pain & joint pain. He is usually managed by hospitalization. Your
management is:
o In-patient management & hospitalization
o Out-patient management by NSAID
o Hydration, analgesia & monitoring
o Narcotic opoids
42. Patient with celiac sprue he should take:
o Carbohydrate free diet
o Protein free diet
o Gluten free diet
43. First sign of MgSO4 overdose:
o Loss of deep tendon reflex
o Flaccid paralysis
o Respiratory failure
44. 70 year old presented with weight loss, fatigue, anemia, upper
quadrant pain without any previous history, the stool showed high fat
he is a known smoker:
o Acute pancreatitis
o Chronic pancreatitis
o Pancreatic carcinoma
45.About alcohol syndrome
o Leads to facial anomaly and mental retardation
o Reduce to 1 glass of wine to decrease the risk of alcohol syndrome
o Wine will not cross the placenta
46. What is the most common cause of chronic diarrhea
o Irritable bowel syndrome
47. Patient with dysphagia to solid and liquid, and regurgitation, by barium
there is non peristalsis dilatation of esophagus and air-fluid level and
tapering end. What is the diagnosis?
o Esophageal spasm
o Achalasia
o Esophageal cancer
48. Patient with nausea, vomiting and diarrhea developed postural
hypotension. Fluid deficit is:
o Intracellular
o Extracellular
o Interstitial
49. 25 year old Saudi man presented with history of mild icterus,
otherwise ok, hepatitis screen: HBsAg +ve, HBeAg +ve, anti HBc Ag +ve
(this should be core anti-body, because core antigen doesn’t leave
hepatocyte to the blood), the diagnosis:
o Acute hepatitis B
o Convalescent stage of hepatitis B
o Recovery with seroconversion hepatitis B
o Hepatitis B carrier
o Chronic active hepatitis B
50.23 year old female presented with finding of hyperbilirubinaemia,
normal examination, investigation shows total bilirubin=3.1, direct
bilirubin=0.4, the most likely diagnosis:
o Gilbert disease
o Criglar najjar syndrome 1
o Dublin Johnson syndrome
o Rotor’s disease
o Sclerosing cholangitis
51. Patient diagnosed with obstructive jaundice best to diagnose common
bile duct obstruction:
o ERCP
o US
52. 48 years female patient with abdominal pain, nausea, vomiting
tenderness in right hypochondrial area your diagnosis is:
o Acute cholecystitis
53. 50 year old male with 2 year history of dysphagia, lump in the throat,
excessive salivation, intermittent hoarseness & weight loss. The most
likely diagnosis is:
o Cricopharyngeal dysfunction
o Achalasia
o Diffuse spasm of the oesophagus
o Scleroderma
o Cancer of the cervical esophagus
54.Gastroesophageal Reflux disease best diagnosed by:
o History
o Physical examination & per-rectal examination
o History & barium meal
o History & upper GI endoscopy
55. Irritable bowel syndrome all EXCEPT
o Abdominal distention
o Mucous PR
o Feeling of incomplete defecation
o PR bleeding

56.Regarding H.pylori eradication:


o Clarithromycin for 1 week
o Bismuth, ranitidine amoxil for 2 weeks
o PPI 2 weeks, amxilor 1 week clarithromycin
57. One type of food is protective against colon cancer:
o Vitamin D
o Fibers
58.70 year old woman presented with 3 days history of perforated ulcer,
she was febrile, semi comatose and dehydrated on admission. The
BEST treatment is:
o Transfuse with blood, rehydrate the patient, perform vagotomy
and drainage urgently
o Insert a NGT & connect to suction, hydrate the patient, give
systemic antibiotics and observe
o Insert a NGT & connect to suction, hydrate the patient, give
systemic antibiotics and perform placation of the perforation
o Hydrate the patient, give systemic antibiotics and perform
hemigastrectomy
o None of the above
59.Patient was diagnosed to have duodenal ulcer and was given ranitidine
for 2 weeks and now he is diagnosed to have H.pylori. What is your
choice of management?
o Omeprazol, clarithromycin & amoxicillin
o Bismuth + tetracycline + metronidazole
o Metronidazole amoxicillin
o Omeprazole + tetracycline
60. 28 year old lady presented with history of increased bowel motion
in the last 8 months. About 3-4 motions/day. Examination was normal.
Stool analysis showed cyst, yeast, nil mucus, culture: no growth. What is
the most likely diagnosis?
o Inflammatory bowel disease
o Irritable bowel disease
o Diverticulitis
61. 40 year old man presented to the ER with 6 hour history of severe
epigastric pain radiating to the back like a band associated with nausea.
No vomiting, diarrhea or fever. On examination the patient was in
severe pain with epigastric tenderness. ECG normal, serum amylase was
900u/l, AST and ALT are elevated to double normal. Which of the
following is the least likely precipitating factor to this patient’s
condition?
o Hypercalemia
o Chronic active hepatitis
o Chronic alcohol ingestion
o Hyperlipidaemia
o Cholethiasis
62. Patient had abdominal pain for 3 months. What will support the
pain due to duodenal ulcer?
o Pain after meal 30-90 min
o Pain after meal immediately
o Pain after fatty meal
o Pain radiating to the back
63. The single feature which best distinguishes Crohn’s disease from
ulcerative colitis is:
o Presence of ileal disease
o Cigarette smoking history
o Presence of disease in the rectum
o Non-caseating granulomas
o Crypt abscesses
64. 45 year old man presented with anorexia, fatigue and upper
abdominal pain for one week. On examination he had tinge of jaundice
and mildly enlarged tender liver. Management includes all EXCEPT:
o Liver ultrasound
o ERCP
o Hepatitis markers
o Serum alkaline transferase
o Observation and follow up
65. 30year old man presented with upper abdominal pain and dyspepsia.
Which of the following doesn’t support the diagnosis of peptic ulcer:
o Hunger pain
o Heart burn
o Epigastric mass
o Epigastric tenderness
o History of hematemesis
66. Hepatitis most commonly transferred by blood:
o HBV
o HAV
o HCV
o None of the above
67. All of the following organisms causes diarrhea with invasion except:
o Shigella
o Yersenia
o Salmonella
o Cholera
o Campylobacter
68. Premalignant lesions have:
o Pedunculated polyps
o Villous papilloma (adenoma)
o Polypoid polyp
o Juvenile polyp
69. Patient had abdominal pain and found to have gastric ulcer all are
predisposing factor except:
o Tricyclic antidepressant
o NSAIDS
o Delayed gastric emptying
o Pyloric sphincter incompetence
o Sucralfate
70. In the neck, esophagus is:
o Posterior to the trachea
o Anterior to the trachea
o Posterior to the vertebral column
71. Patient with hepatitis B then he said which one of the following
antigens appear in the window period?
o HBsAg
o HBcAg
o Anti HBe
o Anti HBc antibody Ig M against HBcAg
72. Treatment of erosive gastritis?
o Antibiotics
o H2 blocker
o Depend on the patient situation
o Total gastrectomy
o Sucralfate
73. Patient old with WBC 17000 and left iliac fosa tenderness and fever
most likely has:
o Diverticulitis
o Colon cancer
o Crohn’s disease
74. Which of the following features is related to Crohn’s disease:
o Fistula formation
o Superficial layer involvement
75. 60 year old male patient complaining of dysphagia to solid food. He is a
known smoker and drinking alcohol, he has weight loss. What’s the
most likely diagnosis?
o Esophageal cancer
o GERD
o Achalasia
76. Which is true about gastric lavage?
o It is safer than ipecac if the patient is semiconscious
o It is done to the patient in the right decubitus position
o It is useless for TCA if the patient presented after 6 hrs
77. Which of the following conditions is contraindicated to use ibuprofen?
o Peptic ulcer disease
78. Patient come with jaundice, three days after the color of jaundice
change to greenish what is the cause?
o Oxidation of bilirubin to bilverdin which is greenish color
79. Patient with hypercholesteraemia, he should avoid:
o Organ meat
o Avocado
o Chicken
o White egg
80. Overcrowded area, contaminated water, type of hepatitis will be
epidemic:
o Hepatitis A
o Hepatitis B
o Hepatitis C
81. Celiac disease severe form involve:
o Proximal part of small intestine
o Distal part of small intestine
o Proximal part of large intestine
o Distal part of large intestine
82. GERD which cancer is the patient at risk of contracting?
o Adenocarcinoma
83. High risk of developing colon cancer in young male is:
o Smoking, high alcohol intake, low fat diet
o Smoking, low alcohol intake, high fat diet
o Red meat, gardner’s disease (Gardner syndrome)
o Inactivity, smoking
84. Patient with primary biliary cirrhosis, which drug helps the
histopathogy of the liver?
o Steroid
o Interferon
o Ursodiol
85.A man travelled to Indonesia and had rice and cold water and ice cream.
He is now having severe watery diarrhea and severely dehydrated. What
is the most likely he has:
o Vibrio cholera
o Clostridium difficile
o Clostridium perfringens
o Dysentery
o Shigella
86. Patient with peptic ulcer using anti-acid, presented with forceful
vomiting food particle:
o Gastric outlet obstruction
87. 75 year old female with 2 days history of MI is complaining of abdominal
pain, vomiting, bloody stool, x-ray shows abdominal distention with no
fluid level, serum amylase is elevated. Dx:
o Ulcerative colitis
o Acute pancreatitis
o Ischemic colitis
o Diverticulitis
88. Stop combined OCP if the patient has:
o Chronic active hepatitis
o Breastfeeding
o Varicose veins
o Gastroenteritis
89. All the following are differentials of acute abdomen except:
o Pleurisy (Diaphragmatic pleurisy sometimes has been incorrectly
diagnosed as acute disorder of the abdomen)
o MI
o Herpes zoster (visceral type cause acute abdomen)
o Polyarteritis nodosa (cause acute abdomen through ischemia)
o Pancreatitis
90. 6 month old baby presented to the clinic with 2 day history of
gastroenteritis. On examination: decreased skin turgor, depressed
anterior fontanelle & sunken eyes. The Best estimate of degree of
dehydration :
o 3%
o 5%
o 10%
o 15%
o 25%
91. Man with history of alcohol association with
o High MCV
o Folic acid deficiency
o B12 deficiency
o Hepatitis
92. In irritable bowel syndrome, the following mechanism,
contraction and slow wave myolectricity seen in:
o Constipation
o Diarrhea
93. Kwashiorkor disease usually associated with
o Decrease protein intake, decrease carbohydrate
o Increase protein, increase carbohydrate
o Decrease protein, increase carbohydrate
94. 22 year old male patient was presented by recurrent attacks of
diarrhea, constipation and abdominal pain relieved by defecation but no
blood in stool, no weight loss. What is the diagnosis
o Irritable bowel syndrome
95.Young healthy male has abdominal pain after basketball. Examination
fine except for left paraumbilical tenderness. What to do?
o Abdominal USS
o Flat plate graph
o Send home & reassess within 48hrs
96. Prophylaxis of cholera
o Good hygiene, sanitation and oral vaccine, in epidemic public:
mass single dose of vaccine & tetracycline
97. Chronic diarrhea is a feature of:
o HyperNatremia
o HyperCalcemia
o HypoMagnesaemia
o Metabolic Alkalosis
98. Teacher in school presented with 3 days history of jaundice &
abdominal pain, 4 of school student had the same illness in lab, what is
true regarding this patient?
o Positive for hepatitis A IgG
o Positive hepatitis A IgM
o Positive hepatitis B core
o Positive hepatitis B c anti-body
99. Which of the following features of ulcerative colitis distinguishes
it from Crohn’s disease
o Possible maliganant transformation (both but more in UC)
o Fistula formation (common in CD)
o Absence of granulomas
o Colon involvement (Both)
100. Inflammatory bowel disease is idiopathic but one of the following
is possible underlying cause:
o Immunological
101. Which of the following is true regarding varicella vaccine during
breast feeding:
o It is safe
o No breast feeding except after 3 days of immunization
102. Girl with amenorrhea for many months. BMI is 20 and is stable
over 5 years. Diagnosis:
o Eating disorder
o Pituitary adenoma

Ortorhinolaryngology.

1. 56yr old presents with vasomotor rhinitis


A. Local antihistamine (correct)
B. Local decongestant
C. Local steroid
D. Systemic antibiotic
2. 9yrs old presented with ear pain, red tense tympanic membrane,and negative rhines
test with positive weber test with lateralization (conductive loss) for 2 days only
a. Otitis media (correct)
b. Otosclerosis
c. Cholesteatoma
3. A child was treated for otitis media with three different antibiotics for 6 weeks but
without improvement . which antibiotic is best for treatment next?
a. Amoxicillin
b. Penicillin
c. Cephalosporin
d. Amoxicillin and clavulanic acid (correct)
e. Erythromycin and sulfamethoxazole
4. Patient presented with ear pain red tympanic membrane apparent vessels with limited
mobility of the tympanic membrane. Whats the most likely diagnosis
a. Acute otitis media (correct)
b. Tympanic cellulitis
c. Mastoiditis
5. Most common cause of otorrhea
a. Acute otitis media (correct)
b. Cholesteatoma
c. Leakage of cerumen
d. Eustachian tube dysfunction
6. Patient with difficulty getting air. Nasal exam showed unilateral swelling inside the nose.
Whats the initial treatment for this patient
a. Decongestant
b. Sympathomimetics
c. Corticosteroids (correct)
7. Nasal decongestant (vasoconstrictive) can cause
a. Rhinitis sicca
b. Rebound phenomena (correct)
c. Nasal septum perforation
8. Patient with pain and discharge on examination he feels pain on moving pinna, there is
normal tympanic membrane with erythematous auditory canal
a. Otitis media
b. Otitis externa (correct)
9. Patient with recurrent congested nose and conjunctivitis , what would you give?
a. Antihistamine and oral decongestant.
10. Epistaxis treatment
a. Sit upright and forward with mouth open and firmly press on nasal alar for
5mins.
11. One of the steps in managing epistaxis
a. Packing the nose
b. Press the fleshy part of nostrils (correct)
c. Put patient in lateral lying position
12. Patient is febrile 38.5, ear ache discharge paresthesia and hemiparesis on same side
a. HZV (correct)
b. Epidural abcess
c. Subdural hematoma
13. Young patient with congested nose, sinus pressure tenderness and green nasal
discharge. Has been treated thrice with broad spectrum antibiotics previously , what is
ur next action?
a. Give antibiotics
b. Nasal corticosteroid (correct)
c. Give antihistamine
d. Decongestant
14. Old man with cognitive deficit what will you screen for
a. IQ test
b. Involuntary movement test
c. Memory score test (correct)
d. Hearing test
15. Young man came with nasal bleeding from posterior septum not known to have any
medical disease or bleeding disorder , whats your management
a. Tampon in posterior septum
b. Screen for blood and coagulation
c. Inject septum by vasoconstrictor
d. Spray anaesthetic or vasoconstrictor (correct)
16. What is the best diagnostic test for maxillary sinusitis
a. Ct scan (correct)
b. Xray
c. Torch examination
d. Mri
e. Uss
17. Which of the following is an indication for tonsillectomy
a. Sleep apnea (correct)
b. Assymptomatic large tonsils
c. Peripharyngeal abscess
d. Retropharyngeal abscess
18. A 45yr old lady was complaining of dizziness sensorineural hearing loss on her left ear
(8th nerve palsy) tingling sensation and numbness on her face loss of corneal reflex. MRI
showed a dilated internal ear canal (or CT shows intracranial mass). The diagnosis is
a. Acoustic neuroma (correct)
b. Glue ear
c. Drug toxicity
d. Herpes zoster
e. Cholesteatoma
19. A child presented with earache. On examination, there was a piece of broken glass deep
in the ear canal. The mother gave a hx of broken glass in the kitchen. Best mgt is:
a. Refer to ENT
b. Remove by irrigation of a stream of solution into the ear
c. Remove by forceps , don’t irrigate (correct)
d. Instill acetone into the external auditory canal.
20. A 15 yr old boy presents with 5 days hx of pain behind the left ear and 3 days hx of
swelling over mastoid. He had a hx of AOM treated by amoxicillin but was not
completed. On examination he has tenderness over the mastoid bone with swelling, TM
shows absent cone reflex and mild congestion. Dx?
a. AOM
b. Serous otitis media
c. Acute mastoiditis (correct)
d. Glue ear
21. Most common cause of hearing loss in children
a. Chronic serous otitis media (correct)
b. Eustachian tube dysfunction
c. Ototoxic drugs
22. 23 year old lady with 1month hx of nasal discharge and nasal obstruction, she
complained of pain on the face throbbing in nature, referred to the supraorbital area
worsened by head movt walking and stopping. O/e tender antrum with failure of
translumination. Most likely Dx:
a. Frontal sinusitis
b. Maxillary sinusitis (correct)
c. Dental abscess
d. Chronic atrophic rhinitis
e. Chronic sinusitis
23. Treatment of cholesteatoma is
a. Antibiotic
b. Steroid
c. Surgery (correct)
d. Grommet tube
24. Child with ear pain with positive pump test for tympanic membrane, treatment is
a. Maryngiotomy (correct)
b. Amoxicillin/potassium
25. Child with unilateral nasal obstruction with bad odour (offensive)
a. Unilateral adenoid hypertrophy
b. Foreign body (correct)
26. Child came with inflammation and infection of the ear. Most likely complication is
a. Labrynthitis
b. Meningitis
c. Encephalitis
d. Mastoiditis
27. Most common site of malignancy in paranasal sinuses
a. 90% maxillary and ethmoid sinuses
28. 2yr old child with ear pain and bulging TM what is the Dx?
a. Otitis media (correct)
b. Otitis externa
c. Otomycosis
d. Bullous myringitis
29. First step in mgt of epistaxis
a. Pinching fleshy part of the nose (correct)
b. Adrenalin
c. Nasal packs
d. Not interfering
30. Case of temporal arteritis, what is the treatment of choice
a. Corticosteroids
31. The most common cause of coughin adults is
a. Asthma
b. Gerd
c. Postnasal drip
32. A 5yr old child came with earache and o/e there is fluid in the middle ear and adenoid
hypertrophy. Beside adenoidectomy on mgt, which also should you do
a. Myringotomy (correct)
b. Grommet tube insertion
c. Mastoidectomy
d. Tonsillectomy
33. Boy 3 days after flu symptoms develop conjunctivitis with occipital and neck LN
enlarged. Dx is?
a. Adenovirus (correct)
b. Streptococcus
c. HSV
34. 50 yr old with uncontrolled DM complains of black to brown nasal discharge. Dx is
a. Mycomyosis (correct)
b. Aspergillosis
c. Foreign body
35. Glue ear (secretory OM, OM with effusion or serous OM)
a. Managed by grommet tube (correct)
b. Leads to sensorineural hearing loss
c. Pus in middle ear
d. Invariably due to adenoid
36. Most prominent symptom of AOM
a. Pain (correct)
b. Hearing loss
c. Discharge
d. Tinnitus
37. All are true about hoarseness in adult except
a. Due to incomplete apposition of vocal cords
b. If >3weeks needs laryngoscopy
c. If due to overuse, advise to whisper a few weeks
d. Commonly seen in bronchial CA
e. f eature of myedema (correct)
38. regarding tinnitus all are true except
a. a symptom that is not experienced by children (correct)
b. present in anaemia (fe and b12 def)
c. as salicylate complication that improves with drug withdrawal
d. if associated with deafness it improves if hearing loss improves
39. what is the commonest cause of ortorrhoea
a. otitis externa (correct)
b. csf ortorrhoea
c. liquefied eczema
d. Eustachian tube dysfunction
40. Regarding aphthous ulceration in the mouth, all are true except
a. There is no treatment for acute ulcer (correct)
b. Tetracycline suspension helps in healing
c. There is immunological role in its development
d. Mostly idiopathic in origin
41. Patient had hoarseness of voice for three weeks what is the next thing to do?
a. Throat swab
b. Laryngoscopy (correct)
42. A lady with epistaxis after cauthery of the nose, all tru except
a. Don’t snuff for 1-2days
b. Use of nasal packing if bleeding starts again
c. Use of aspirin for pain (correct)
43. Px is complaining of right side pharynx tenderness, o/e px has inflamed right tonsil and
redness around tonsil with normal left tonsil. Dx is?
a. Parenchymal tonsil
b. Quinse parapharyngeal abscess
c. Peritonsillar abscess (hot potato voice) (correct)
44. Child after swimming in pool came complaining of right ear tenderness. o/e px has
external auditory canal redness tenderness and discharge . mgt is
a. Antibiotics drops gentamicin or cipro avoid aminoglycosides
b. Systemic antibiotics only if cervical LN or cellulitis
c. Steroid drops only if chronic
d. Antibiotic and steroid drops (correct)
45. Child came with inflammation and infection of the ear the most likely complication is
a. Labrynthitis
b. Meningitis is most common Intracranial complication but for extracranial is post
auricular abscess (correct)
c. Encephalitis
46. Anosmia
a. Frontal
b. Occipital
c. Temporal (correct)
d. Parietal
47. Px suffers sensorineural loss, vertigo dizziness 3yrs ago, and now developed numbness
and weakness of facial msc. Dx:
a. Menieres disease (correct)
b. Acoustic neuroma
c. Acute labrynthitis
48. Px with seasonal nasal discharge, watery, what is the first mgt
a. Decongestatnt
b. Antihistamine (correct)
c. Steroid
49. Px presented with nausea and nystagmus with tinnitus and inability to walk unless he
concentrates well on a target object. His cerebellar fxn is intact
a. Benign positional vertigo (correct)
b. Meniere dx (vertigo, tinnitus hearing loss, aural fullness)
c. Vestibular neuritis (nausea and vomiting, inability to stand, vertigo)
50. 5yr old recently adopted child, parents brought him to you with white nasal discharge,
he’s a known case of SCA. What will you do to him
a. Give prophylactic penicillin
51. Submandibular swelling and pain during eating, best investigation
a. Xray
b. Uss
c. CT (correct)
d. MRI
52. Rt ear pain with plugging of tympanic membrane
a. Secretory otitis media
53. Ranula
a. Forked uvula
b. Thyroglossal cyst
c. Swelling @floor of the mouth (correct)
54. All are speech disorders except
a. Stuttering
b. Mumbling
c. Cluttering
d. Palila
55. smelly unilateral nasal discharge is a feature of
a. Adenoid
b. Choanal atresia
c. Foreign body (correct)
d. RT atrophy
56. The most common cause of epistaxis in children is
a. Polyps
b. Trauma (ie nose picking) (correct)
c. Dry air’
d. Thrombocytopenia
57. Swallowed FB will be found In all of the following except
a. Stomach
b. Tonsil (correct)
c. Pharyngeal pouch
d. Piriform fossa
58. Adenoids
a. Can be a chronic source of infection
b. Causes snoring
c. Located at the bacvk of the nasopharynx 1 inch above the uvula
d. Involved in immune system reaction
e. All of the above (correct)
59. All are normal in association with teething except
a. Rhinorrhea
b. Diarrhea (correct)
c. Fever >39c
d. Irritability
60. All are features of tonsillar abscess except
a. Deviation of uvula to affected side.
61. Child presents with rhinorrhoea and sore throat for 5 days, middle ear effusion, o/e no
redness of ear. Cause of effusion?
a. Otitis media
b. Upper respiratory infection
62. Patient is a smoker and alcoholic with difficulty in swallowing and neck mass. Right
investigation
a. Indirect laryngoscope (true)
b. Neck CT
c. Head CT
d. Biopsy
e. Aspiration
63. Child falls from stairs came with mild injury to the nose, no bleeding and edema in the
nasal septum, best treatment;
a. Nasal packing
b. Reassure
c. Analgesia
d. Refer to ENT (he will give analgesia) (correct)
64. 16 yr old female becomes deaf suddenly. Her mother became deaf @30yrs. Dx;
a. Otosclerosis (correct)
b. Acoustic neuroma
c. Tympanic perforation
65. Regarding barret oesophagitis which is correct
a. Risk of adenocarcinoma (correct)
b. Risk of squamous cell CA (correct if no hx of GERD)
66. 35 yr old smoker o/e shows white patch on the tongue, what is the mgt
a. Antibiotics
b. No treatments
c. Close observation
d. Excision biopsy
67. Px presented with ear pain, red tympanic membrane, apparent vessels with limited
mobility of the TM. Most likely Dx;
a. AOM (correct)
b. Tympanic cellulitis
c. Mastoiditis
68. Waking up from sleep cant talk no fever, can cough normal vocal cord; likely Dx
a. Functional Aphonia (loss of speech without attributable cause)
69. Px after swimming in a pool (clear Dx of otitis externa) Rx;
a. Nothing
b. Amphotericin b
c. Steroid
d. Ciprofloxacin drops (correct)
70. Post partum female with recurrent attack of conductive hearing loss, on CT, there is
adhesion in the semicircular canal. Dx;
a. Otosclerosis (correct)
b. Miners
c. Tuberous sclerosis
71. Purulent discharge from middle ear, how do you treat
a. Systemic AB (correct)
b. Local AB
c. Steroid
72. Child with URTI then complained of ear pain. o/e there is hyperaemia of YM and +ve
insufflations. Has received two antibiotics before. Best treatment;
a. Augumentin (correct)
b. Azithromycin
c. Ciprofloxacin
73. Px with sore throat, anorexia, loss of appetite, enlarged tonsils with petechiae on palate
and uvula, mild tenderness of spleen and liver. What is the Dx
a. Infectious mononucleosis
74. Urti with meningococcus type A, best Rx
a. Rifampicin
b. Penicillin, ampicillin, chloramphenicol, ceftriaxone. (correct)
75. 28yr old with AOM. He was treated with amoxicillin, came after 3wks for follow up,
there was fluid collection behind TM, no blood. What do u do next?
a. Watchful waiting (correct)
b. Myringotomy
76. Urti with streptococcus type A best Rx
a. Penicillin for 10days.
77. The most common cause of cough in adult is
a. Asthma
b. GERD
c. Postnasal drip (correct)
78. 5yr old seen In ER with fever and sore throat, which of the following suggests viral
etiology
a. Presence of thin membrane over the tonsils
b. Palpable tender cervical LN
c. Petechial rash over hard or soft palate
d. Absence of cough
e. Rhinorrhea of colourless secretion (correct)
79. 4yr old presented with 2day hx of shortness of breath, a seal like cough with no
sputumand mild fever, o/e he did not look ill or in distress
a. Acute epiglottitis
b. Croup (correct)
c. Angioedema
80. Child with rt ear pain, and tendernesson pulling ear, no fever, o/e inflamed edematous
rt ear canal with yellow discharge. Dx;
a. Otitis media
b. Otitis externa (correct)
c. Cholesteatoma
81. Child with decreased hearing, her grandmother has deafness, rhine and weber revealed
bone conduction more than air conduction (osteosclerosis). Plan;
a. Reassure
b. Refer her to hearing aid
c. Prescribe hearing instrument
d. Refer her to ENT (correct)
82. AOM criteri
a. Should not be with effusion (correct)
b. Rapid sign and symptom
83. Child came to you with barking cough, stridor and by examination you see steeple sign.
What is your diagnosis
a. Epiglottitis
b. Croup (correct)
84. 50yr old male, smokes 40pcks/year, develops painless ulcer on the lateral border of the
tongue which is rolled in with indurated base and easily bleeds. Dx;
a. Squamous cell carcinoma (correct)
b. Aphthous ulcer
c. Syphilis
85. Px develops nasal discharge with frontal headache
a. Acute sinusitis (correct)
b. Migraine
c. Temporal arteritis
d. Temporal
86. 55yr old male px presented with just mild hoarseness, o/e there is a mild cervical mass.
Best investigation
a. Indirect laryngoscope (correct)
b. CT brain
c. CT neck
87. Old px presented with ear pain headache hemiparesis. most likely cause;
a. Epidural abscess
b. Spinal abscess
c. Subdural haematoma
88. Px has snoring during sleep and on examination there is large tonsil what will you do for
him
a. Weight reduction (correct)
b. Adenoidectomy
89. Which of the following dosent cause ear pain
a. Pharyngitis
b. Otitis
c. Dental carries
d. Vestibular neuritis (correct)
90. Bad breath smell with sickly smell, no dental caries or inflammation. Likely cause
a. Cryptic tonsillitis
b. Sojreens syndrome
91. Old man complains of progressive hearing loss, mostly propounded when he listens to
radio, nil such symptoms in the past. Rhine and webers tests result in bilat sensorineural
hearing loss. Dx:
a. Meniere disease
b. Otosclerosis
c. Noice induced deafness
d. Hereditary hearing loss
92. Patient with a large nodule in the nose which is painful and telangectasia on the face.
What do u give
a. Deoxycycline (not sure) (correct)
b. Clindamycin
c. Retinoid
93. Patient with perforated TM with foul whitish discharge. Diagnosis
a. Otosclerosis
b. Otitis externa
c. Cholesteatoma (correct)
94. Young male had pharyngitis thencough and fever what is the most likely organism
a. Staph aureus
b. Streptococcus pneumonia (correct )
95. 7yrs old with SOB and wheezing . he was sitting in bed leaning forward with drooling and
stridor
a. Epiglottitis (correct)
b. Bronchial asthma
96. Child presented with dysphagia sore throat postnasal drip drooling of saliva rhonchi and
fever of 38.5c.. the Rx is
a. Hydrocortisone injection immediately
b. Call ENT for intubation (correct )
c. Admit in icu
d. Give antibiotics and send him home
97. Child with epiglotitis will present with all of the following except
a. Fever
b. Dysphagia
c. Like to lie in supine position (correct)
d. Stridor
98. Most common site of malignancy in paranasal sinuses
a. Maxillary sinus
99. Child is having a croup early morning. The most common cause is
a. Post nasal drip
100. Patient is post rhinoplasty. Presented with brown discharge with foul odour from
the wound . what could be the mgt
a. Debridement and antibiotics
101. All the following are present in otitie media except
a. Signs and symptoms of inflammation
b. Signs and symptoms of effusion (correct)
c. High grade fever
d. Pain
102. 4 years old patient comes with cystic swelling behind lower lip varying in size , has
bluish discolouration
a. Ranula (ruptured salivary gland duct usually caused by local trauma)
103. Generalized skin rash associated with lymphnode enlargement
a. EBV
104. Offensive white ear discharge with white rigid tympanic membrane, likely diagnosis
a. One of the choices is spicteccusis
105. Enlarged unilateral tonsils
a. Peripharyngeal abscess
106. One of these causes conductive hearing loss
a. AOM (correct)
b. Syphilis
c. Meniere disease
107. 5yr old with hx of fever and swelling of the face and to both ears (parotid gland
enlargement) what is the most common complication apart from orchitis
a. Encephalitis’
b. Mastoiditis
c. Meningitis (correct)
108. All are features of tonsillar abscess except
a. Deviation of uvula to affected side
109. Patient taking treatment for TB came with imbalance and hearing loss which drug is
responsible
a. INH
b. Streptomycin (correct)
c. Rifampicin
d. Ethambutol
e. Pyrazinamide.
110. Patient with URTI says “ I see flash when I sneeze” why?
a. Mechanical irritation (correct)
b. Chemical irritation
111. Old patient with abnormal ear sensation and fullness, hx of vertigo and progressive
hearing loo. Investigation showed low frequency sensorial hearing loss. Dx;
a. Acoustic neuroma
b. Neuritis
c. Meniere’s disease. (correct)
112. Patient came with peeling, redness, waxy appearance in the scalp margins behing
the ear and nasal fold. Best Rx is
a. Topical antifungal (correct)
b. Antibiotic
c. Steroid
113. Adult patient came with AOM received amoxicillin for 1week. During f/u after 3wks
u found fluid behind TM
a. Give AB for 10days
b. Antihistamine
c. Follow up after 1month, can resolve spontaneously (correct)
d. Give AB.
Basic Sciences MCQs

1. In a cachetic patient, the body utilizes the protein of the muscle


o to provide Amino Acid and protein synthesis
2. Which of the following describes the end of early inflammatory phase
o formation of scar
o formation of ground base of collagen
o the end of angiogenesis
3. Anatomy of facial artery after leaving mastoid
o superficial to mandibular vein and external carotid
o deep to external carotid
o superficial to external carotid
4. Seperation of chromatid occur in
o Anaphase
o Metaphase
o Telophase
5. Adult polycystic kidney disease mode of inheritance
o Autosomal dorminant
6. Which of the following organs is likely to receive a proportionately
greater increase in blood flow
o Kidneys
o Liver
o Heart
o Skin
o None of the above
7. Scenario of trauma, on face examination there is shifted mouth angle,
loss of sensation of anterior tongue, which CN is affected:
o Facial nerve
o Trigeminal nerve
8. Link the suitable treatment with organism
o Shigella 3rd generation of cephalosporin or trimethoprim-
sulfamethoxazole
o Salmonella ciprofloxacin, 3rd generation Cefotaxim
o Campylobacter erythromycin
o Giardia The most common treatment for giardiasis is
metronidazole (flagyl) for 5-10
9. Cheese tyramine:
o MAOI
10. The most unwanted side effect of anti-cholinergic drugs is:
o Constipation & dry mouth
11. The best way to prevent infection in Medical practice:
o Wear gloves
o Wash hands
o Wear mask
o Wear gown
12. A patient on IV line developed fever due to infection. The most
common source of bacterial contamination of IV cannula
o Contamination of fluid during the manufacturing process
o Contamination of fluid during cannula insertion
o Contamination during injection of medication
o Seeding from remote site during intermittent bacteremia
13. Blood culture show gram negative rod shape that grows only on
charcoal free fungal organism is:
o Staph. Aureus
o Chlamydia
o Klebsiella
o Mycoplasma
14. Most common side effect of atropine is:
o Urinary incontinence
o Dryness
o Bradycardia
15. Which of the following shift the O2 dissociation curve to the right:
o Respiratory alkalosis
o Hypoxia
o Hypothermia
16. In the neck, esophagus is:

o Posterior to the trachea

o Anterior to the trachea

o Posterior to vertebral column


17. Which of the following is a treatment for giardiasis:

o Prazequantel

o Mebendazole

o Metronidazole

o Albendazole

18. A lot of bacteria produce toxins which are harmful. Which one of the
following is used in amiddirs

o Botulism

o Tetanus

o Diphtheria

o Staph aureus

19. Tyramine cause hypertension crises with:

o TCA

o MAOI

o SSRI

20. Methergine contraindicated:

o Asthma

o HTN and pregnancy

o Gastric disease

21. In IDA, which of the following iron studies is most specific:

o Iron level

o TIBC

o Ferritin level

22. Treatment of Chlamydia


o Doxycycline

23. Family went to a dinner party after 6 hours they all had symptoms of
abdominal pain, nausea, vomiting and dehydration. Some of them
recovered while others needed hospitalization. What’s the most likely
organism?

o Giardia

o Staph aureus

o Salmonella

o C. perfiringis

o C. boyulism

24. 25 year old male who recently came from India presented with a 3
day history of left knee pain & swelling, 1 day history of right wrist
swelling. On examination it was swollen, tender, red with limitation of
movement, 50cc of fluid was aspirated from the knee. Gram stain
showed gram positive diplococcic. What’s the most likely organism?

o Brucella

o Neisseria meningitides

o Streptococcus pneumonia

o Staph aureus

o Strept pyogenes

25. Vertigo, inability to perceive termination of movement & difficulty in


sitting or standing without visual due to toxic reacts that likely to occur
in 75% of patient with long term use of:

o Penicillin

o Tetracycline

o Amphotericin B
o Streptomycin

o INH

26. Which of the following antibiotics has the least activity against S.
aureus?

o Erythromycin

o Clindamycin

o Vancomycin

o Dicloxacillin

o First generation cephalosporins

27. Furosemide increase the excretion of:

o Na+

o K+

o Phosphorus

28. All of the following signs or symptoms are characteristics of an


extracellular fluid volume deficit EXCEPT:

o Dry, sticky oral mucous membranes

o Decreased body temperature

o Decreased skin tugor

o Apathy

o Tachycardia

29. Anticoagulant effect of heparin is based on:

o Potentiation of anti-thrombin III

o Activation of plasmin into plasminogen

o Inactivation of ionized calcium


o Reduction of available factor VII

30. The length of the trachea in adult is:

o 11-12cm

o 24cm

o 20cm

o 4cm

31. All of the following drugs are advised to be given to elderly patient,
EXCEPT:

o Cimetidine

o Thyroxin

o Digoxin

o Chloropromide

32. Heparin anticoagulant action depend on:

o Potentiating of anti-thrombin III

o Change plasmin to plasminogen

o Affect prothrombin

o Affect ionized Ca++

33. Entamoeba histolytica cysts are destroyed best by:

o Boiling

o Iodine added to water

o Chlorine added to water

o Freezing

34. All of the following cause gastric irritation, except:

o Erythromycin
o NSAIDS

o Sulcrafate

o Diclofenac

o Penicillin

35. What is the most risk of antihypertensive drugs on elderly patient:

o Hypotension

o Hypokalemia

o CNS side effect

36. Digoxin toxicity:

o Tinnitus

o Pleural effusion

o Nausea

o All of the above

37. Which one of these drugs is administered orally:

o Amikacin

o Neomycin

o Gentamycin

o Streptomycin

o Tobramycin

38. Chronic use of steroid will give:

o Osteomalacia

o Myopathies of pelvic girdle

o Increased risk of breast Ca


o Hypoglycaemia

39. All of the above are anti-arrhythmic drugs, except:

o Xylocain

o Digoxin

o Quinidine

o Amiodarone

o Procainamide

40. All of the following are true about paracetamol poisoning, except:

o Metabolic acidosis

o Hypohlycemia

o Brochospasm

o Liver failure

o Acute renal tubular necrosis

41. Diagnosis of hemochromatosis:

o Serum ferritin

o Transferrin saturation

42. Beriberi caused by deficiency of

o Vitamin B1

o Vitamin B2

o Vitamin B3

43. 14 year old female with BMI 32.6 (associated big chart):

o Overweight

o Obese
o Normal weight

44. Recent study revealed that anti-psychotic medications cause the


following complication:

o Weight gain

o Alopecia

o Cirrhosis

45.Which one of the anti-TB medications cause tinnitus, imbalance

o Streptomycin

o Isoniazide

o Pyrazinamide

46. At what level is lumbar puncture (LP) done at:

o L2-L3

o L3-L4

o L5-S1

47. Patient present with high blood pressure (systolic 200), tachycardia,
mydriasis, sweating, what

o Anti-cholinergic

o Sympathomimetic

o Tricyclic antidepressant

o Organophosphorus compounds

48. All are complications of long term use of phenytoin, EXCEPT

o Ataxia

o Osteoporosis

o Osteomalacia
o Macrocytosis

49. Physiological cause of hypoxemia

o Hypoventilation

o Improper alveolar ventilation

o Perfusion problem

o Elevated 2,3 DPG

50.Calcium channel blocker drugs like verapimil, diltiazem, nifedipine are


all effective in all, EXCEPT:

o Prinzmetal angina

o Hypertension

o Atrial tachycardia

o Ventricular tachycardia

o Effort angina

51. Epidemic disease in poor sanitation areas affecting children and young
adults:

o Hepatitis A

o Hepatitis B

o Hepatitis C

o Hepatitis D

52.One of the anti-psychotics causes ECG changes, Leukopenia, drooling

o Resperidone

o Clozapine

o Amisulpride

53. Man using sildenafil, to prevent hypotension you should not use
o Nitrate

o B blocker

o ACIE

o CCB

54. Deep laceration in the anterior aspect of the wrist, causing injury to the
median nerve the result is:

o Claw hand

o Drop hand

o Inability to oppose the thumb to the fingers


Endo
cri
nol
ogy
1
.Pat ient,
knownc a
seofDMt ype2o ni
nsuli
n,hi
sblo
o d
sugarmeas ur
ementasfol
lowing;mor
ning=285mg/dl
,
at3pm=1 65mg/dl
,atdi
nnerti
me=95 mg/dl
.Whatwi l
l
beyo urmanagement?
oI
ncr
eas
eeve
ningdo
seo
flo
ngAc
tingi
nsul
in(correct)
oDe
cre
asee
veni
ngdo
seo
fsho
rta
cti
ngi
nsul
in
oDe
cre
asee
veni
ngdo
seo
flo
nga
cti
ngi
nsul
in
oI
ncr
eas
eeve
ningdo
seo
fsho
rta
cti
ngi
nsul
in.
2
.Pat i
entknowncaseofI
DDMpr es
entedwithDKA;
K=6mmo l
/landbl
oodsugar3
50mg/dl.Yo
uwillgi
ve
hi
m
oI
vfl
uid
oI
vfl
uida
ndi
nsul
in(
cor
rec
t)
oSo
dium bi
car
bona
te
3
.Pati
entinc
reasefo
otsi
ze3
9>>>41
.5andi
ncr
eas
esi
ze
o
fhandandjoi
nt.Whi
chho
rmone
oT
hyr
opi
ne
oPr
ola
cti
n
oACT
H
oSo
matot
ropi
chormo
nea
lsokno
wna
sgr
owt
h
ho
rmo
ne(cor
rec
t)
4.
Typi
cals
ympt
omso
fdi
abet
icket
oac
ido
sis
.whati
sthe
mechani
sm
oNoi
nsul
in+f
ata
cidut
il
iz
ati
ono
fke
tone
s
5
.Pat ientcamewi thwhiti
shdischar
gef
romthenippl
e,
herinvestigat
ionshowpitui
tar
yadeno
ma,whic
h
hormo neresponsi
blefo
rthis
oPr
ola
cti
n
6.
T4hi
gh,
freeT
3hi
gh,
TSHl
ow;di
agno
sis
oI
mmunet
hyr
oidt
is
7
.Yo
ungmal
ewi
thuni
lat
eralgy
nec
omas
tia
oSt
ops
oyapr
oduc
ts
oCo
mpr
ess
ionbr
aatni
ght
oI
twi
llr
eso
lve
dbyi
tse
lf(
cor
rec
t)
8
.42
yearo
ldwi
tht
hyr
oidmas
s,whati
sthebes
ttodo
oF
NA
9.
Hypothyro
idpat
ientonthyroxinhadanor
exi
a, dr
ycough
anddy s
pneaandleftventr
iculardysf
unct
ion.Shehad
normalTSHandT 4l
evels,
hy perpho
sphat
emi aand
hypocal
cemia.T
hediagnosisis
oPr
ima
ryhy
popa
rat
hyr
oidi
sm
oSe
conda
ryhy
popa
rat
hyr
oids
im(
cor
rec
t)
oHy
popi
tui
tar
ism
oUnc
ont
rol
le
dhy
per
thy
roi
dis
m
1
0. Pat i
entwit
hDMt y
pe2hasc o
nservat
ive
managementst
il
lco
mplai
ningo
fweightgainand
po
lyur
ia,
give
oI
nsul
inSho
rta
cti
ng
oMe
tfo
rmi
n(c
orr
ect
)
oL
onga
cti
ngi
nsul
in
1
1. 3 4y earol
dfe malepat
ientpresent
edwith
t
ermi
nalhairwit
hmal ehairdi
stri
butio
nandhas
f
emal
egeni t
alorganstheunderl
yingproc
essi
s
oPr
ola
cti
nove
rse
cre
tio
n
oAndr
oge
nove
rse
cre
tio
n(c
orr
ect
)
1
2. F emalepati
entpresent
edwi t
hs ymptomsof
hyper
thy
roi
dism,tenderneckswell
ing,anddi
scomf
ort
.
ShehadlowTSHandhi ghT4level
.Thediagno
sis
;
oSuba
cut
ethy
roi
dit
is(
cor
rec
t)
oT
hyr
oidno
dul
e
oGr
ave
sdi
sea
se
1
3. Panc
reat
iti
s
oAmy
las
eiss
lowl
yri
si
ngbutr
ema
inf
orda
ys
oAmyla
sei
smo
res
pec
ifi
cbutl
esss
ens
iti
vet
han
l
ipa
se
oRans
oncri
teri
ahasseve
rit
y(pr
edi
cti
ve)i
nac
ute
pa
ncrea
tit
is(
corr
ect
)
oPai
nisi
ncr
eas
edbys
itt
inga
ndr
eli
eve
dbyl
yi
ng
do
wn
oCo
ntr
ace
pti
vepi
ll
sisa
sso
cia
ted
1
4. Pr
imar
yhy
per
aldo
ster
oni
sm as
soc
iat
edwi
th
oHy
per
nat
remi
a
oHy
poma
gne
semi
a
oHy
poka
lemi
a(c
orr
ect
)
oHy
per
kal
emi
a
1
5. Pat i
entpresentwit
ht hi
spict
ureonl
ynoot
her
manif
estat
ion‘
organomegalyorl
ymphadeno
pat
hy’
whatist
hediagnosis
oMo
nonuc
leo
sis
oGo
ite
r(c
orr
ect
)
oL
ympho
ma
1
6. T
hyr
oidc
anc
erc
anbef
rom
oHy
pot
hyr
oidi
sm
oGr
ave
sdi
sea
se(
cor
rec
t)
oT
oxi
cno
dul
e
1
7. Pat i
entiscomplai
ningo
fir
ri
tat
ion,
tac
hy c
ardi
a
,
nightsweati
ng.Labsdones
howedTSH=normal,
T
4=high,di
agnosi
sis
oGr
ave
sdi
sea
se(
cor
rec
t)
oSe
conda
ryhy
pot
hyr
oidi
sm
oHa
shi
mot
othy
roi
dit
is
1
8. 8 year
soldboywhic
his6yearol
dheightandbo
ne
s
cano
f5 .
5year
s,whati
sthediagno
sis
oSt
ero
id
oGe
net
ic
oHy
poc
hodr
opl
asi
a
ohy
pot
hyr
oids
im(
cor
rec
t)
1
9. hi
rsut
ism as
soc
iat
edwi
thwhi
cho
fthef
oll
owi
ng
oa
nor
exi
a(c
orr
ect
)
oj
uve
nalhy
pot
hyr
oidi
sm
odi
gox
int
oxi
ci
ty
oc
/oc
itr
ate
2
0. 60y earo
ldmalecomplai
no fdec
reasedl
ibi
do,
decr
eas
edejacul
ati
on,
FBS=6.5mmo l
,incr
eas
eprol
act
in,
nor
malFSHandL H,
whatisthenextstep?
oT
est
ost
ero
nel
eve
l
oDM
oNLF
BG
oCTo
fthehe
ad(
cor
rec
t)
2
1. Si nglet
hyroi
dnodul
eshowedhi
ghi
odi
neupt
ake
,
whati
sthebes
tt r
eat
ment?
oRa
dioi
odi
ne1
31
oSe
ndho
me
oAnt
ithy
roi
dme
dic
ati
on(
cor
rec
t)
oEx
cis
ioni
fpr
ese
nt
2
2. T
hyr
oto
xic
osi
sinc
ludeal
loft
hef
oll
owi
ngex
cept
oNe
uro
pat
hy
oHy
per
gly
cemi
a(c
orr
ect
)
oPe
riphe
ral
pro
xima
lne
uro
pat
hy
2
3. T
hemo
stac
tivef
orm
oT
4
oT
3(c
orr
ect
)
oT
SH
2
4. 45 yearol
dpresent
edwithpol
yurea,urineanal
ysi
s
sho
wedgluc
osur
iaandnegat
iveketo
ne ,
F BS1 4mmo l
.
Whati
sthebestmanagementofthi
spat ient?
oI
nte
rme
dia
teI
Mins
uli
nti
lls
tabl
e
oNPHo
rle
ntei
nsul
in3
0mgt
hendi
et
oSul
pho
nyl
ure
a
oDi
abe
ticdi
eto
nly
oMe
tfo
rmi
n(c
orr
ect
)
2
5. A3 0yearoldteacherco
mplaini
ngofexcessi
ve
waterdrinkingandf r
equentofuri
nati
on,
on
examinationnormal.YoususpectDMandr eques
t
FBS=6.8,thediagnosi
sis
oDM
oDI
oI
mpa
ire
dfa
sti
nggl
uco
se(
cor
rec
t)
oNLbl
oods
uga
r
oI
mpa
ire
dgl
uco
set
ole
ranc
e
2
6. 42 y
earol
dfemal
epr
esentedwi
th6monthhist
ory
o
fmal
ais
enausea
,vo
mit
ing.l
abNA=127
,K=4.9,
UREA=1
5 ,
creati
nine=1
35,HCO3
=13
,gl
uco
se=2
.7mmo
l.
themostli
kelydiagnosi
s
oHy
pot
hyr
oidi
sm
oPhe
ochr
omo
cyt
oma
oHy
povo
lemi
aduet
ovo
mit
ing
oSI
ADH
oAddi
si
ondi
sea
se(
cor
rec
t)
2
7. I
nDKA,
use
oSho
rta
ndi
nte
rme
dia
tea
cti
ngi
nsul
in(
cor
rec
t)
oL
onga
cti
ngi
nsul
in
2
8. Met
for
minwhi
chi
str
ue;
oCa
usehy
pogl
yce
mia
oCa
usewe
ightga
in
oSuppr
essgl
uco
neo
gene
sis
(co
rre
ct)
2
9. Hy per
prol
act
inemi
aas
soc
iat
edwi
thal
loft
he
f
oll
owi
ngexcept
oPr
egna
ncy
oAc
rome
gal
y
oOCP
oHy
pot
hyr
oidi
sm
3
0. Pat ientcametoyouandyoufindhi
sBPt obe
160/100,heisnotonanymedicati
onyet
.Lab
i
nves ti
gationsho
wedcreat
ini
ne(normal
),NA1 45
(13
5-
145),K=3.2(
3.5
-5.
1),
HCO3=30(
21-3
0).Whatisthe
diagnosi
s
oEs
sent
ialhy
per
tens
ion
oPhe
ochr
omo
cyt
oma
oAddi
si
ondi
sea
se
oPr
ima
ryhy
per
aldo
ste
roni
sm(
cor
rec
t)
3
1. A46y earo l
dman, akno wnc aseofdiabetesfor
thelas t5mo nths .Hei smai ntainedo nMet for
min
800mgpoT I
D, dietco ntr
o landus edt owal kdai
lyfor
30mi n.onex aminat i
o nunr emarkabl e.Some
i
nves tigat
io nsho wt hefo ll
owing;F BS=7.4mmo l/
l,2
hr
PP=8 .6mmo l/
l,HBA1 c6. 6,Totalcho l
esterol5.
98mmo /l
,
HDL C=0.92 mmo l/l,L
DL C=3 .8
8mmo l
/l,t
rigl
yceri
des
2.84mmo /
l(0.34-2.27
)bas edonevi dencet hefoll
owing
concerninghi smanagementi strue;

oT
hegoalofmanage
menti
stol
owe
rthe
t
ri
gly
ceri
desf
irs
t.
oT
hego
alo
fma
nage
menti
stor
educ
ethe
HBA1
c
oT
hedrugofchoi
cet
ore
acht
hego
ali
s
f
ibr
ates
(co
rrec
t)
oT
hego
alo
fma
nage
menti
sCDCa
t2.
6mmo
l
oT
hego
alofma
nage
menti
sto
talc
hol
est
ero
lat
5
2mmol.
3
2. Regar di
ngthec
rit
eri
aofthediagno
siso
fdi
abet
es
mel
li
tus
,thefo
ll
owi
ngaretr
ueexcept;
oSympt
omaticpati
entpl
usca
sua
lpl
asmagl
uco
se
7.
6mmol/
lisdia
gnosti
cofDM
oF
PG7 .
0mmol/lpl
us2hrpo
st7
5gm gl
uco
se
1
1.1
mmo l
/li
sdiagno
sti
cofDM
oF
PG5
.5mmo
l/l
=no
rma
lfa
sti
nggl
uco
se
oFPG7.
0mmo l
/l
=pr
ovis
io na
ldiagnosiso
fDMa
nd
mustbeco
nfi
rmedina notherset
tingin
asy
mptomati
cpat
ient.(corr
ect)
o2hrpos
t75
gm gluc
ose7.
6mmol
/land
<11.
1mmol
/l=i
mpair
edgl
uco
setol
era
nce
.
3
3. 3 6year
sol
dfemalewit
hF BS=14mmol/land
gl
uco
s ur
ia,
wit
houtket
onesinuri
ne,thet
reatmenti
s
oI
nte
rmi
tte
ntI
Mins
uli
nNPH
oSul
pho
nyl
ure
a
oDi
abe
ticdi
eto
nly
oMe
tfo
rmi
n
3
4. A3 0yearmal
epres
entedwit
hpol
yuri
a,negat
ive
ket
ones
,rando
m bl
oodsugar2
80mg/
dl.Management
oNo
thi
ngdo
ne,
onl
yobs
erve
oI
nsul
in3
0iuNPH+di
etc
ont
rol
oDi
eta
nde
xer
cis
e(c
orr
ect
)
oOr
alhy
pogl
yce
mic
3
5. T
hyr
oidc
anc
eras
soc
iat
edwi
th
oEuo
thy
roi
d(c
orr
ect
)
oEuo
thy
roi
d
oHy
pot
hyr
oid
oGr
ave
s
3
6. Ol dpati
enttakehypergl
ycemicdr
ugsand
devel
opedgo
ut,whatis
nr espons
ibl
edrugs
;
oF
rus
emi
de
oT
hia
zide
(co
rre
ct)
3
7. Pat ho
logi
calr
esultf
romthyro
idt
iss
ues
howed
papi
ll
arycarci
noma,thenex
tstep;
oSur
gic
alr
emo
val
(co
rre
ct)
oAppl
yra
dio
act
iveI
131
oGi
veAnt
ithy
roi
ddr
ug
oF
oll
owupt
hepa
tie
nt.
3
8. Ac ervi
c all
ymphnodeisfoundt obereplacedwith
awelldif
ferenti
atedthy
roidt
issue.Attheo perat
ion,
ther
earenopal pabl
elesi
onInthet hyr
oidgland.The
oper
atio
no fc ho
icei
s;
oT
ota
lthyr
oide
cto
mya
ndmo
dif
ieddi
sse
cti
on
r
adi
cal
oTo
talt
hyroi
decto
myandr
adi
calne
ck
di
ssec
tio
n(cor
rect
)
oT
ota
lthy
roi
dec
tomy
oThyr
oidl
obe
cto
mya
ndr
emo
valo
fal
llo
call
ymph
nodes
oT
hyr
oidl
obe
ctomyandist
hmust
hec
tomyand
r
emoval
ofal
ll
ocale
nla
rgedl
ymphnode
s.
3
9. Whi
chi
str
ueabo
utDMI
KSA?
oMo
stl
yar
eIDDM
oMo
stNI
DDMa
reo
bes
e(c
orr
ect
)
40. F
emal
eco
mewi
thmani
fes
tat
ionso
f
hypothyr
o idism,sl
eepi
ng,myxedema,col
dintol
erance,
nows hes ufferfr
om di
ffi
cul
tyinbreathi
ng,wheezi
ng,
TSH=normal ,T4normal
,Ca=decrease,
phosphorus =nor
mal,ALP=normal,whatisy
o ur
diagnos
is?
oSe
conda
ryhy
popa
rat
hyr
oidi
sm
41
. Patientcomeswi
thdi
arr
hea,
confusi
onand
mus
cleweaknesshes
uff
ersfr
om what?
oHy
poka
lemi
a(c
orr
ect
)
oHy
per
kal
emi
a
oHy
per
cal
cemi
a
42
. T hefir
sts
tepi
nthemanagemento
fac
ute
hyper
cal
caemi
asho
uldbe
oCor
rec
tio
no fdefi
ci
tofe
xtr
ace
ll
ula
rfl
uid
vo
lume(cor
rect
)
oHe
modi
al
ysi
s
oAdmi
nis
tra
tio
noff
rus
emi
de
oAdmi
nis
tra
tio
nofmi
thr
amy
cin
oPa
rat
hyr
oide
cto
my.
43
. T
ype1di
abet
ic,
tar
getHBA1
C
o9
o8
o6.
5(c
orr
ect
)
44. 1 9year
so l
dat hlete,hi
sweighti
ncr
eas
e45po unds
i
nlast4mo nt
hs.Inex aminati
on,hei
smuscul
ar,
BP13
8/89,whatist hec ase
?
oAl
coho
l
oCo
cai
nea
bus
e
oAna
bol
i
cst
ero
idus
e(c
orr
ect
)
45
. Adul thadahi
storyo
fpalpi
tati
on,sweati
ngand
neckdis
comfor
tfo
r10days,
ladCBCno r
mal ,
ESR=80,
TSH0.01,F
T4hi
gh,whati
sthediagnos
is?
oGr
ave
sdi
sea
se(
cor
rec
t)
oSuba
cut
ethy
roi
dit
is
oSuba
cut
ethy
roi
dit
is
oSuba
cut
ethy
roi
dit
is
46. Ol ddiabeti
cpatientwhosti
llhavehyper
gly
c emi
a
des
piteinc
reaseins
ul i
ndose
,theproblem wi
thinsul
in
onobesepati
entis
oPo
str
ece
pto
rre
sis
tanc
e
47
. F emaleco
metothecli
nicwit
hherbabyof
6mo
nt hs
,shehadt
remorandothers
ignIfo
rgo
tit
,
whi
cho
fthef
oll
owi
ngi
smo
stl
ikel
ydi
agno
sis
?
oHa
shi
mot
o
oPo
stpa
rtum t
hyr
oidi
ti
s(c
orr
ect
)
oHy
per
thy
roi
dis
m
oSuba
cut
ethy
roi
dit
is
oHy
ope
rthy
roi
dis
m
48
. Di abet
icpat
ientoni
nsul
inandMetfo
rmi
nhas
rena
limpai
rment.Whati
syournexts
tep?
oSt
opMe
tfo
rmi
nanda
ddACEi
nhi
bit
or
49. Thefol
lo
wingmo
rec
ommo
nwi
tht
ype2DMt
han
t
ype1DM
oWe
ightl
oss
oGr
adua
lons
et
oHe
redi
tar
yfa
cto
rs(
cor
rec
t)
oHL
ADR3
+-DR4
5
0. Pat i
entwaspres ent
edbyt r
emor,
fever
,
pal
pit
ati
on,di
agnos
edasc as
eofhyper
thyro
idi
sm,
whatisyo
urini
ti
altreatment.
oSur
ger
y
oRa
dioi
odi
ne
oBe
tabl
ocke
rs(
cor
rec
t)
oPr
opy
lthi
ora
cil
5
1. Pat ientwithtr
uncalo
besi
tyeasybrui
sing,
hy
per
tensio
n,buff
alohump.Whatisthedi
agnosis
?
oCus
hing
5
2. Bl
oods
ugari
nDMt
ype1i
sbes
tco
ntr
oll
edby
oSho
rta
cti
ngi
nsul
in
oL
onga
cti
ng
oI
nte
rme
dia
te
oHy
pogl
yce
mica
gent
s
oBa
sala
ndbo
lusi
nsul
in(
cor
rec
t)
5
3. Wel lknowncaseof DMwaspr esent
edtot
he
ERwithdro
ws i
ness,
int
heinves
tigati
on;blo
od
sugar
=400mg/dl,
Ph=7.
05.
whatisy o
urmanagement?
o1
0uni
tsi
nsul
in+400c
cofde
xtr
ose
o0.
1uni
t/kgo
fins
uli
n,
SC
oNAHCO3
oOnel
it
ero
fno
rma
lsa
li
ne(
cor
rec
t)
5
4. Pr egnantpati
entcamewithnec kswell
ingand
mul
ti
plenodul
arnontendergo
iter
,thenexteval
uati
on
i
s;
oT
hyr
oidbi
ops
y
oGi
veAnt
ithy
roi
dme
dic
ati
on
oRa
dia
tio
nio
dine
oT
SHa
ndf
reeT
4,o
rjus
tfo
ll
owup.(
cor
rec
t)
5
5. Ol dpati
entwi
thnec ks
welli
ng,no
dular
,disf
igur
ing
wi
thhist
oryofmuscl
eweakness,co
ldint
o l
eranc
e,
ho
ars
eness,whati
sy ourmanagement?
oL
evo
thy
rox
ine
oCa
rba
maz
ole
oT
hyr
oidl
obe
cto
my(
cor
rec
t)
oRa
dio
-ac
tivei
odi
ne
5
6. Pr egnantwo
manwi t
hsy
mpt
omso
f
hy
per
thyr
oidi
sm,TSHl
ow;
oPr
opy
lthi
our
aci
l(
cor
rec
t)
oRa
dio
-ac
tive
oPa
rti
alt
hyr
oide
cto
my
5
7. Yo urec ei
vedacal
lfrom af
atherwhohasas o
n
diagno
s edrecentl
ywit
hDM- 1forsi
xmo nths,hesaid
thathef oundhissonl
yingdownunconsci
o usinhi
s
bedroom,whaty ouwil
ltel
lhimifheisseekingfor
advi
ce?
oBr
inghi
mass
oona
spo
ssi
blet
oER
oCa
llt
hea
mbul
anc
e
oGi
vehi
m hi
sus
ual
dos
eofi
nsul
in
oGi
vehi
mIMgl
uca
gone
oGi
vehi
msuga
rinf
lui
dpe
ror
al.(
cor
rec
t)
5
8. Di abet i
cpatientonmedicat
ionf
oundunc
ons
cio
us,
hisbl
oods ugarwa s60,whati
sthemo s
tcommon
causetothisproblem?
oSul
fony
lur
eas
e(c
orr
ect
)
oBi
gua
nide
s
5
9. 40y r
soldmalepr
esent
edwi
thl
argehands
,
hepat
omegal
y,di
agno
sis
oAc
rome
gal
y(c
orr
ect
)
oGi
gant
ism
60. T
hec
aus
eofi
nsul
inr
esi
stanc
eino
bes
eis
;
oI
nsul
inr
ece
pto
rski
nas
eac
tivi
ty
oNumbe
rofi
nsul
inr
ece
pto
r
oCi
rcul
ati
ono
fant
i-
ins
uli
n
oI
nsul
inpr
oduc
tio
nfr
omt
hepa
ncr
eas
oPo
st–r
ece
pto
rac
tio
n(c
orr
ect
)
61
. Pat ientwit
hDMpr esent
edwithl
imit
edor
decreasedrangeofmovementpas
siveandac
tiveo
fal
l
dir
ec t
ionofshoul
der
oF
roz
ens
houl
der
(co
rre
ct)
oI
mpi
ngme
nts
yndr
ome
oOs
teo
art
hri
ti
s
62
. F emalenotmarri
edwithno
rmali
nves
tigat
ion
exc
eptFBS=142,
RBS=196,
whati
sthetr
eatment
oGi
vei
nsul
inSC
oAdvi
ceno
tbe
comema
rri
ed
oBa
rri
erc
ont
rac
ept
ivei
sgo
od
oBMI
cont
rol
(co
rre
ct)
63
. Yo ungdi
abeti
cpat
ientcamewithabdominalpai
n,
vomi t
ingandketoness
mel l
edfr
om hismouth.Whatis
frequentcause?
oI
nsul
inmi
sma
nage
ment
(co
rre
ct)
oDi
etmi
sma
nage
ment
64. 70y
rsaudi
cdi
abet
icmal
esuddenl
yfel
ldo
wn,
thi
s
c
oul
dbe;
oMa
ybet
hepa
tie
nti
shy
per
tens
ivea
ndhe
de
vel
ope
das
udde
nri
sei
nBP
oHemightha
dfo
rgo
thi
sor
alhy
pogl
yce
mic
dr
ug(
corr
ect
)
oSudde
nICHwhi
chr
ais
ehi
sICP.
65
. Patientpr
esentwi
thc o
nsti
pati
on‘
hypot
hyr
oidi
sm’
toc
onf
irmthatthepat
ienthashypo
thy
roi
dis
m
oT
4
oT
SH(
cor
rec
t)
oF
reeT
4
66. Whi cho
fthefol
lo
wingmedi
cat
ionss
houl
dbe
avo
idedi
ndi
abet
icnephr
opa
thy?
oNi
fedi
pine
oL
osa
rta
n
oL
isi
nopr
il
oT
hia
zide
(co
rre
ct)
67
. Whi c
hoft
hef
oll
owi
ngi
ndi
cat
ebeni
gnt
hyr
oid
les
ion?
oL
ympha
deni
ti
s
68
. Pat i
entcometoy o
ut ocheckup,hehasDMhi s
bloodsugari
swellc
o nt
rol
led,buthi
sBPis1 3
8/86,
all
otherphysi
calex
aminati
ons hownoabnormal
ity
i
ncl
udi
ngneurol
ogi
calexaminati
on,
heisfol
lo
wing
r
egul
arl
yinopht
hal
mo l
ogycl
inic
,whatyo
uwi l
lputi
n
y
ourpl
antomanaget hispat
ient
oGi
vingACEi
nhi
bit
orgo
alf
orBPf
orDM;
138
/80
69. F emalepat
ientwithhypot
hyroi
dis
m, TSHhigh,but
hedidno
tgivethetotalT4no
rfree,
pulsenormal,BP
nor
mal,s
heisont hy
roxin,
whatyouwilldo
oI
ncr
eas
ethy
rox
inf
oll
owa
fte
r6mo
nths
oI
ncr
eas
ethy
rox
inf
oll
owa
fte
r3mi
ont
hs(
cor
rec
t)
oDe
cre
aset
hyr
oxi
nfo
ll
owa
fte
r6mo
nths
oDe
cre
aset
hyr
oxi
nfo
ll
owa
fte
r3mo
nths
7
0. Al
lcaus
eshy
per
pro
lac
tinemi
aex
cept
oPr
egna
ncy
oAc
roma
gal
y
oMe
thy
ldo
pa
oAl
lo
pur
ino
l(c
orr
ect
)
oHy
pot
hyr
oidi
sm
7
1. DMt
ype1
oHL
ADR4
7
2. Di
ffer
entbet
weenpr
imar
yands
eco
ndar
y
hy
per
aldo
ster
oni
sm
oI
ncr
eas
ere
nni
nins
eco
nda
ry
7
3. 5 0yr
swithunc
ontr
oll
eddiabet
es,complai
nof
bl
acktobro
wnnasaldi
schar
ge.Sodiagnos
isis;
oMy
cos
is(
cor
rec
t)
oAs
pir
glo
sis
oF
ore
ignbo
dy
7
4. Whi c
hho r
moneaf
fec
tthebi
leac
idandl
ower
ing
t
hec
hol
ester
ol
oCho
lec
yst
oki
nn
7
5. T
hyr
oidno
dul
esno
nmal
ignant
oMul
ti
ple
7
6. Mec
hani
smo
fcus
hingdi
seas
e
oI
ncr
eas
eACT
Hfr
om pi
tui
tar
yade
noma
oI
ncr
eas
eACT
Hfr
oma
dre
nal
.
2
3
4
5
6
7
8
9
10
Scanned by CamScanner
Scanned by CamScanner
Scanned by CamScanner
Scanned by CamScanner
Scanned by CamScanner
Scanned by CamScanner
Scanned by CamScanner
Scanned by CamScanner
Scanned by CamScanner
Scanned by CamScanner
Scanned by CamScanner
Scanned by CamScanner
Scanned by CamScanner
Scanned by CamScanner
Scanned by CamScanner
Scanned by CamScanner
Scanned by CamScanner
Scanned by CamScanner
Scanned by CamScanner
Scanned by CamScanner
Scanned by CamScanner
Scanned by CamScanner
Scanned by CamScanner
Scanned by CamScanner
Scanned by CamScanner
Scanned by CamScanner
Scanned by CamScanner
Scanned by CamScanner
Scanned by CamScanner
Scanned by CamScanner
Scanned by CamScanner
Scanned by CamScanner
Scanned by CamScanner
Scanned by CamScanner
Scanned by CamScanner
Scanned by CamScanner
Scanned by CamScanner
Scanned by CamScanner
Scanned by CamScanner
Scanned by CamScanner
Scanned by CamScanner
Scanned by CamScanner
Scanned by CamScanner
Scanned by CamScanner
Scanned by CamScanner
Scanned by CamScanner
Scanned by CamScanner
Scanned by CamScanner
Scanned by CamScanner
Scanned by CamScanner
Scanned by CamScanner
Scanned by CamScanner
Scanned by CamScanner
Scanned by CamScanner
Scanned by CamScanner
Scanned by CamScanner
Scanned by CamScanner
Scanned by CamScanner
Scanned by CamScanner
Scanned by CamScanner
Scanned by CamScanner
Scanned by CamScanner
Scanned by CamScanner
Scanned by CamScanner
Scanned by CamScanner
Scanned by CamScanner
Scanned by CamScanner
Scanned by CamScanner
Scanned by CamScanner
Scanned by CamScanner
Scanned by CamScanner
Scanned by CamScanner
Scanned by CamScanner
Scanned by CamScanner
Scanned by CamScanner
Scanned by CamScanner
Scanned by CamScanner
Scanned by CamScanner
Scanned by CamScanner
Scanned by CamScanner
Scanned by CamScanner
Scanned by CamScanner
Scanned by CamScanner
Scanned by CamScanner
Scanned by CamScanner
Scanned by CamScanner
Scanned by CamScanner
Scanned by CamScanner
Scanned by CamScanner
Scanned by CamScanner
Scanned by CamScanner
Scanned by CamScanner
Scanned by CamScanner
Scanned by CamScanner
Scanned by CamScanner
Scanned by CamScanner
Scanned by CamScanner
Scanned by CamScanner
Scanned by CamScanner
Scanned by CamScanner
Scanned by CamScanner
Scanned by CamScanner
Scanned by CamScanner
Scanned by CamScanner
Scanned by CamScanner
Scanned by CamScanner
Scanned by CamScanner
Scanned by CamScanner
Scanned by CamScanner
Scanned by CamScanner
Scanned by CamScanner
Scanned by CamScanner
Scanned by CamScanner
Scanned by CamScanner
Scanned by CamScanner
Scanned by CamScanner
Scanned by CamScanner
Scanned by CamScanner
Scanned by CamScanner
Scanned by CamScanner
Scanned by CamScanner
Scanned by CamScanner
Scanned by CamScanner
Scanned by CamScanner
Scanned by CamScanner
Scanned by CamScanner
Scanned by CamScanner
Scanned by CamScanner
Scanned by CamScanner
Scanned by CamScanner
Scanned by CamScanner
Scanned by CamScanner
Scanned by CamScanner
Scanned by CamScanner
Scanned by CamScanner
Scanned by CamScanner
Scanned by CamScanner
Scanned by CamScanner
Scanned by CamScanner
Scanned by CamScanner
Scanned by CamScanner
Scanned by CamScanner
Scanned by CamScanner
Scanned by CamScanner
Scanned by CamScanner
Scanned by CamScanner
Scanned by CamScanner
Scanned by CamScanner
Scanned by CamScanner
Scanned by CamScanner
Scanned by CamScanner
Scanned by CamScanner
Scanned by CamScanner
Scanned by CamScanner
Scanned by CamScanner
Scanned by CamScanner
Scanned by CamScanner
Scanned by CamScanner
Scanned by CamScanner
Scanned by CamScanner
Scanned by CamScanner
Scanned by CamScanner
Scanned by CamScanner
Scanned by CamScanner
Scanned by CamScanner
Scanned by CamScanner
Scanned by CamScanner
Scanned by CamScanner
Scanned by CamScanner
Scanned by CamScanner
Scanned by CamScanner
Scanned by CamScanner
Scanned by CamScanner
Scanned by CamScanner
Scanned by CamScanner
Scanned by CamScanner
Scanned by CamScanner
Scanned by CamScanner
Scanned by CamScanner
Scanned by CamScanner
Scanned by CamScanner
Scanned by CamScanner
Scanned by CamScanner
Scanned by CamScanner
Scanned by CamScanner
Scanned by CamScanner
Scanned by CamScanner
Scanned by CamScanner
Scanned by CamScanner
Scanned by CamScanner
Scanned by CamScanner
Scanned by CamScanner
Scanned by CamScanner
Scanned by CamScanner
Scanned by CamScanner
Scanned by CamScanner
Scanned by CamScanner
Scanned by CamScanner
Scanned by CamScanner
Scanned by CamScanner
Scanned by CamScanner
Scanned by CamScanner
Scanned by CamScanner
Scanned by CamScanner
Scanned by CamScanner
Scanned by CamScanner
Scanned by CamScanner
Scanned by CamScanner
Scanned by CamScanner
Scanned by CamScanner
Scanned by CamScanner
Scanned by CamScanner
Scanned by CamScanner
Scanned by CamScanner
Scanned by CamScanner
Scanned by CamScanner
Scanned by CamScanner
Scanned by CamScanner
Scanned by CamScanner
Scanned by CamScanner
Scanned by CamScanner
Scanned by CamScanner
Scanned by CamScanner
Scanned by CamScanner
Scanned by CamScanner
Scanned by CamScanner
Scanned by CamScanner
Scanned by CamScanner
Scanned by CamScanner
Scanned by CamScanner
Scanned by CamScanner
Scanned by CamScanner
Scanned by CamScanner
Scanned by CamScanner
Scanned by CamScanner
Scanned by CamScanner
Scanned by CamScanner
Scanned by CamScanner
Scanned by CamScanner
Scanned by CamScanner
Scanned by CamScanner
Scanned by CamScanner
Scanned by CamScanner
Scanned by CamScanner
Scanned by CamScanner
Scanned by CamScanner
Scanned by CamScanner
Scanned by CamScanner
Scanned by CamScanner
Scanned by CamScanner
Scanned by CamScanner
Scanned by CamScanner
Scanned by CamScanner
Scanned by CamScanner
Scanned by CamScanner
Scanned by CamScanner
Scanned by CamScanner
Scanned by CamScanner
Scanned by CamScanner
Scanned by CamScanner
Scanned by CamScanner
Scanned by CamScanner
Scanned by CamScanner
Scanned by CamScanner
Scanned by CamScanner
Scanned by CamScanner
Scanned by CamScanner
Scanned by CamScanner
Scanned by CamScanner
Scanned by CamScanner
Scanned by CamScanner
Scanned by CamScanner
Scanned by CamScanner
Scanned by CamScanner
Scanned by CamScanner
Scanned by CamScanner
Scanned by CamScanner
Scanned by CamScanner
Scanned by CamScanner
Scanned by CamScanner
Scanned by CamScanner
Scanned by CamScanner
Scanned by CamScanner
Scanned by CamScanner
Scanned by CamScanner
Scanned by CamScanner
Scanned by CamScanner
Scanned by CamScanner
Scanned by CamScanner
Scanned by CamScanner
Scanned by CamScanner
Scanned by CamScanner
Scanned by CamScanner
Scanned by CamScanner
Scanned by CamScanner
Scanned by CamScanner
Scanned by CamScanner
Scanned by CamScanner
Scanned by CamScanner
Scanned by CamScanner
Scanned by CamScanner
Scanned by CamScanner
Scanned by CamScanner
Scanned by CamScanner
Scanned by CamScanner
Scanned by CamScanner
Scanned by CamScanner
Scanned by CamScanner
Scanned by CamScanner
Scanned by CamScanner
Scanned by CamScanner
Scanned by CamScanner
Scanned by CamScanner
Scanned by CamScanner
Scanned by CamScanner
Scanned by CamScanner
Scanned by CamScanner
Scanned by CamScanner
Scanned by CamScanner
Scanned by CamScanner
Scanned by CamScanner
Scanned by CamScanner
Scanned by CamScanner
Scanned by CamScanner
Scanned by CamScanner
Scanned by CamScanner
Scanned by CamScanner
Scanned by CamScanner
Scanned by CamScanner
Scanned by CamScanner
Scanned by CamScanner
Scanned by CamScanner
Scanned by CamScanner
Scanned by CamScanner
Scanned by CamScanner
Scanned by CamScanner
Scanned by CamScanner
Scanned by CamScanner
Scanned by CamScanner
Scanned by CamScanner
Scanned by CamScanner
Scanned by CamScanner
Scanned by CamScanner
Scanned by CamScanner
Scanned by CamScanner
Scanned by CamScanner
Scanned by CamScanner
Scanned by CamScanner
Scanned by CamScanner
Scanned by CamScanner
Scanned by CamScanner
Scanned by CamScanner
Scanned by CamScanner
Scanned by CamScanner
Scanned by CamScanner
Scanned by CamScanner
Scanned by CamScanner
Scanned by CamScanner
Scanned by CamScanner
Scanned by CamScanner
Scanned by CamScanner
Scanned by CamScanner
Scanned by CamScanner
Scanned by CamScanner
Scanned by CamScanner
Scanned by CamScanner
Scanned by CamScanner
Scanned by CamScanner
Scanned by CamScanner
Scanned by CamScanner
Scanned by CamScanner
Scanned by CamScanner
Scanned by CamScanner
Scanned by CamScanner
Scanned by CamScanner
Scanned by CamScanner
Scanned by CamScanner
Scanned by CamScanner
Scanned by CamScanner
Scanned by CamScanner
Scanned by CamScanner
Scanned by CamScanner
Scanned by CamScanner
Scanned by CamScanner
Scanned by CamScanner
Scanned by CamScanner
Scanned by CamScanner
Scanned by CamScanner
Scanned by CamScanner
Scanned by CamScanner
Scanned by CamScanner
Scanned by CamScanner
Scanned by CamScanner
Scanned by CamScanner
Scanned by CamScanner
Scanned by CamScanner
Scanned by CamScanner
Scanned by CamScanner
Scanned by CamScanner
Scanned by CamScanner
Scanned by CamScanner
Scanned by CamScanner
Scanned by CamScanner
Scanned by CamScanner
Scanned by CamScanner
Scanned by CamScanner
Scanned by CamScanner
Scanned by CamScanner
Scanned by CamScanner
Scanned by CamScanner
Scanned by CamScanner
Scanned by CamScanner
Scanned by CamScanner
Scanned by CamScanner
Scanned by CamScanner
Scanned by CamScanner
Scanned by CamScanner
Scanned by CamScanner
Scanned by CamScanner
Scanned by CamScanner
Scanned by CamScanner
Scanned by CamScanner
Scanned by CamScanner
Scanned by CamScanner
Scanned by CamScanner
Scanned by CamScanner
Scanned by CamScanner
Scanned by CamScanner
Scanned by CamScanner
Scanned by CamScanner
Scanned by CamScanner
Scanned by CamScanner
Scanned by CamScanner
Scanned by CamScanner
Scanned by CamScanner
Scanned by CamScanner
Scanned by CamScanner
Scanned by CamScanner
Scanned by CamScanner
Scanned by CamScanner
Scanned by CamScanner
Scanned by CamScanner
Scanned by CamScanner
Scanned by CamScanner
Scanned by CamScanner
Scanned by CamScanner
Scanned by CamScanner
Scanned by CamScanner
Scanned by CamScanner
Scanned by CamScanner
Scanned by CamScanner
Scanned by CamScanner
Scanned by CamScanner
Scanned by CamScanner
Scanned by CamScanner
Scanned by CamScanner
Scanned by CamScanner
Scanned by CamScanner
Scanned by CamScanner
Scanned by CamScanner
Scanned by CamScanner
Scanned by CamScanner
Scanned by CamScanner
Scanned by CamScanner
Scanned by CamScanner
Scanned by CamScanner
Scanned by CamScanner
Scanned by CamScanner
Scanned by CamScanner
Scanned by CamScanner
Scanned by CamScanner
Scanned by CamScanner
Scanned by CamScanner
Scanned by CamScanner
Scanned by CamScanner
Scanned by CamScanner
Scanned by CamScanner
Scanned by CamScanner
Scanned by CamScanner
Scanned by CamScanner
Scanned by CamScanner
Scanned by CamScanner
Scanned by CamScanner
Scanned by CamScanner
Scanned by CamScanner
Scanned by CamScanner
Scanned by CamScanner
Scanned by CamScanner
Scanned by CamScanner
Scanned by CamScanner
Scanned by CamScanner
Scanned by CamScanner
Scanned by CamScanner
Scanned by CamScanner
Scanned by CamScanner
St
ati
sti
csandf
ami
lymedi
ci
ne

1
.Po
sit
ivepr
edi
cat
iveval
ue
oPa
tie
ntwhoha
shi
ghr
iskf
act
ora
ndpo
sit
ivet
est
.
2
.F emaleco
metofamil
yphys
icianaskabo
utdi
ett
hat
decr
easeCVD,
(shehasf
amilyhist
ory
)?
oI
ncr
eas
efr
uita
ndve
get
abl
e
oDec
reaset
hei
ntakeo
fmeatanddi
ary.(
cor
rec
t)
oDe
cre
aset
heme
ata
ndbr
ead.
3
.Mo
stdi
ffi
cul
tmet
hodt
opr
event
edi
ntr
ans
mis
sio
n
oPe
rso
ntope
rso
n
oVe
cto
r
oDr
opl
et
oAi
rfl
ow.(
cor
rec
t)
4.
Nul
lhy
pot
hes
is
oT
hee
ffe
cti
sno
tat
tri
but
edt
ocha
nce
.
oThereissi
gni
fi
cantdi
ffe
renc
ebe
twe
ent
het
est
ed
popula
tio
ns
oT
her
eisnosi
gnifi
cantdi
ffer
enc
ebe
twe
ent
he
t
est
edpopul
ati
ons..(co
rrec
t)
5
.Thes
pec
ifi
ci
tyi
s
oWhenthepe
rso
ndo
esha
vet
hedi
sea
sewi
th
po
sit
ivet
est
oWhentheper
sondo
esha
vet
hedi
sea
sewi
th
ne
gati
vetes
t
oWhenthepe
rso
ndo
esno
tha
vet
hedi
sea
sewi
th
po
sit
ivet
est
oWhentheper
sondoesnotha
vet
hedi
sea
sewi
th
ne
gati
vetes
t..(co
rrec
t)
6.
Whati
sthebes
twayo
fheal
theduc
ati
on
oMa
ssme
dia.(
cor
rec
t)
oI
nte
rna
lta
lk
oI
ndi
vidua
lappr
oac
h.
7
.Thebes
twayt
opr
eventho
usemi
te
oCo
vert
hepi
ll
owswi
thi
mpe
rme
abl
eco
ver
oWa
sht
hec
lot
hesi
nho
twa
ter.(
cor
rec
t)
oRe
moveo
ldc
arpe
ts
8
.Childnewl
ydiagnosedwit
has t
hmaandal
ler
gyt
omi
st
dus
twhatyouwi l
ladvi
sehisparent
oAdvi
cet
ore
movea
llt
hec
arpe
tsa
ndr
ugs
oCo
verhi
sbe
dandbe
ll
owwi
thi
mpe
rme
abl
eco
ver
oWa
sht
hec
lot
hesa
ndl
ine
ninho
twa
ter.(
cor
rec
t)
oHumi
dho
usewi
th8
0%humi
dit
y
oCo
oli
ngc
lot
hes
.
9.
Whati
sthedef
ini
ti
ono
fst
andar
ddevi
ati
on
oMe
asur
eme
nto
fva
rie
ty
1
0. At
tri
but
abl
eri
sk
oMeas
ureme
ntofhavetheex
posedandnote
xpo
sed
wi
ththo
seexpo
sedandhavethedi
sease
.
1
1. Oneo
fthes
eno
tli
vevac
cine
oHBV .(
cor
rec
t)
oOPV
oMMR
1
2. Bes
tpr
event
iono
fdus
tmi
tes
oCo
oli
ngc
lot
hes
oHumi
dho
usewi
th8
0%humi
dit
y
oBo
il
ingc
lot
hesa
ndl
i n.(
ne cor
rec
t)
1
3. L
ikel
iho
odr
ati
oofadi
seas
einc
idenc
eis0.
3means
oL
argei
ncr
eas
e
oSma
lli
ncr
eas
e
oNoc
hange
oSma
llde
cre
as.(
e c
orr
ect
)
oL
argede
cre
ase
1
4. T o
wno f15000po
pulati
on,i
n2 009numbero f
del
iver
ieswas1 05
,5ofthem ar
es ti
llbi
rth,
4dieinfir
st
month,2diebefor
efi
rstbir
thday.I
f7 00mo veoutand
250mo vei
n.whatistheperi
natalmo rt
ali
tyrat
e.
o9
o8
o4
o6.(
cor
rec
t)
1
5. Ataday carecentr
e1 0outof50hadredey
einthe
fi
rstweekanother30develo
ps amec o
ndi
tio
ninthe
next2weeks.Wha tistheattac
krate.
o40
o60
0.(
o8 cor
rec
t)
o2
0
1
6. Bef oregi
vinginfl
uenz
avac
cine,
yous
houl
dkno
wif
t
hepat
iental
lergytowhic
h
oShe
ll
fi
sh
oEgg.(
cor
rec
t)
1
7. Yo uhaveanappoi
ntmentwit
hy o
urpatientat
10am whoisanewlydi
agnosedDM,youcamel ateat
11am bec
auseyouhaveanot
hercompli
catedpatient
,
whatareyougoi
ngt os
aytoc o
ntr
olhi
sanger
oTo
ldhi
mt ha
tther
eisa
not
herpa
tie
ntwhor
eal
ly
needy
ourhel
p.
1
8. Whati sthemostc
ommo
nmedi
calpr
obl
em f
aced
i
npr
imar
yhealt
hcar
eis?
oCo
rya .(
z cor
rec
t)
oUT
I
oHy
per
tens
ion
oDi
abe
tes
1
9. T
hegr
eat
estmet
hodt
opr
eventt
hedi
seas
e
oI
mmuni
zat
ion
oGe
net
icc
ouns
eli
ng
oEnvi
ronme
ntmo
dif
ica
tio
n
oT
ryt
ocha
ngebe
havi
oro
fpe
opl
eto
war
d
he
alh(
t c
orr
ect
)
oSc
ree
ning

2
0. I nastudytheyares
elec
tingthe10thf
ami
lyi
n
eac
hgroup,whatisthet
ypeofstudy
oSy
ste
mics
tudy
oNo
nra
ndo
miz
eds
tudy
oSt
rat
ifi
eds
tudy.(
cor
rec
t)
2
1. Yo uwereworkinginac l
ini
cwi t
hac onsultantwho
prescri
bedadr ugthatwasc ontrai
ndicat
edt othe
patient(thepati
entwasal l
ergi
ct othatdrug)buty ou
didn’ti
nterfer
eandas sumedt hathekno wsbet ter
thany ou.Whichoft hefo
ll
owingy ouhaveviolated
oPr
ofe
ssi
ona
lco
mpe
tenc
e
oQua
li
tyo
fca
ringpa
tint.(
e cor
rec
t)
oHo
nes
ty
oPa
tie
ntr
ela
tio
nshi
p
oMa
int
aini
ngt
rus
t
2
2. Phy
sic
ian’
sca
rel
ess
nes
siskno
wnas
oMa
lpr
act
ice
oCr
imi
nal
negl
ect.(
cor
rec
t)
oMa
lfe
asa
nce
oNo
nfe
asa
nce
2
3. Yo uarereadingapo pul
ationstudythatst
ates
t
hat90%o flungcanc erpatientar
es mo ker
swhil
e3 0%
o
flungcancerpatientareno ns mokers.Whatisthe
s
pecif
ici
tyofusings mo kingasapr edi
c t
orofl
ung
c
ancer?
o1
0%
o40%
o3
0%
0%.(
o7 cor
rec
t)
o90%
2
4. Whati sthemostimpo
rtantf
acto
rinat
temptt
o
s
ucc
ess
fulcess
ati
onofsmokingi
s?
oT
hes
moke
rde
sir
etos
tops
mo ng.(
ki cor
rec
t)
oT
hepha
rmacol
ogi
cal
age
ntsus
edi
nthes
moki
ng
c
ess
ati
onpr
ogram
oF
reque
nto
ffi
cevi
si
t
oPhy
sic
iana
dvi
cet
ost
ops
moki
ng
oEvi
denc
eofha
zar
dso
fsmo
king
2
5. Whati
sthemo
stpo
wer
fulepi
demi
ol
ogi
cst
udy
?
oRe
tro
spe
cti
vec
asec
ont
rols
tudy
oCo
hor
tst
udy
oHi
sto
rict
imeda
ta
oSe
conda
ryda
taa
nal
ysi
s.(
cor
rec
t)
2
6. Evi
denc
ebas
emedi
ci
ne
oPr
act
iceme
dic
inea
sint
hebo
ok
oPr
act
icea
cco
rdi
ngt
othede
par
tme
ntpo
li
cy
oPra
ctic
ea c
cordi
ngtoa
vai
la
bles
cie
nti
fi
c
evi
dence.
(co
rrec
t)
oPr
act
icea
cco
rdi
ngt
ofa
cil
it
y
oPr
act
ice
sac
cor
dingt
ola
tes
tpubl
is
hda
ta.
2
7. Pat ientwi
thcancer.Youwantt
obr
eakbadnews
whi
cho
ft hefo
ll
owingistrue.
oI
nfo
rm hi
sfa
mil
y
oI
nfo
rm hi
ma ccor
dingt
ohi
smo
ralba
ckgr
ounda
nd
r
eli
gin.(
o co
rrec
t)
oL
ets
oci
als
ervi
cei
nfo
rm hi
m
oDono
tte
llhi
m.
2
8. F orhealt
heduc
ati
onpr
ogr
amst
obes
ucc
ess
fulal
l
ar
etr
ueexcept
oHuma
nbe
havi
ormus
tbewe
llunde
rst
ood
oI
nfo
rma
tio
nsho
uldbef
romc
ult
ura
lba
ckgr
ound
oDo
cto
rsa
reo
nlyt
hehe
alt
heduc
ato
rs.(
cor
rec
t)
oMe
tho
dsi
ncl
udepi
ctur
esa
ndvi
deo
s(ma
ssme
dia
)
oI
nvo
lves
oci
etyme
mbe
rsa
tea
rlys
tage
.
2
9. Bat
ter
edwo
men
oMul
ti
plevi
si
tmul
ti
plec
ompl
ai
nt
3
0. Rel
ati
ver
isk
o2
0
3
1. Pat i
entwit
hf ami
lyhi
sto
ryofcoro
naryart
ery
di
seas
e,hisBMI
-28cametoyouaski
ngfortheadvi
ce
oSt
art8
00c
alo
riei
nta
keda
il
y
oDe
cre
asec
arbo
hydr
ateda
yti
me
oI
ncr
eas
efa
tandde
cre
asepr
ote
in
oSt
artwithdecr
eas
e________kc
alo
riepe
rkgpe
r
wee (
k.co
rrec
t)
3
2. Pat i
enthasfamil
yhi
stor
yofDMhei
sover
wei
ght
t
hepr
opermanagementf
orhi
mis
oGe
ner
alr
educ
tio
ninc
arbo
hydr
ate
s
oDe
cre
ase5
00kc
alf
ore
ver
ykg
oSt
opc
arbo
hydr
ate
sst
artf
atdi
et
3
3. F
irs
tst
epi
nepi
demi
cst
udyi
s
oVe
rif
yingdi
agno
sis
3
4. Rando
miz
edc
ont
rolt
rial
sbec
omes
tro
ngeri
f
oYo
ufo
ll
owmo
ret
han5
0%o
ftho
sei
nthes
tudy
oSyst
emicassi
gnmentpr
edi
cta
bil
it
yby
pa
rti
cipa
nt.
s(c
orr
ect)
3
5. Mo t
herwor
ryabo
utradi
ati
onfrom mi
cro
wavei
f
ex
pos
edtoherc
hil
d.Whatyoutel
lher
oNotal
lra
dia
tio
naredange
rousa
ndmi
cro
wave
oneo
fthem..(co
rrec
t)
oMi
cro
wavei
sda
nge
rouso
nchi
ldr
en
oMi
cro
wavei
sda
nge
rouso
nadul
t
3
6. Whati
sthemo
sti
mpo
rtanti
nco
uns
eli
ng
oEx
cludephy
sic
ali
ll
nes
s
oEs
tabl
is
hingr
appo
rt.(
cor
rec
t)
oF
ami
ly
oSc
hedul
eda
ppo
int
ment
3
7. I
nbr
eaki
ngbadnews
?
oF
indo
utho
wmuc
hthepa
tie w.(
ntkno cor
rec
t)
oF
indo
utho
wmuc
hthepa
tie
ntswa
ntst
okno
w
3
8. As tudydo net oassesstherisko flo
ngt aki
ng
calci
um int wogro upsthediseasedgr oupwithcalci
um
pluscontrolaccordingtogeographi
c alloc
atio
n, s
ite,
andpo pulati
on.Ita dds(??
)thistypeo fstudy;
oCo
hor
t.(
cor
rec
t)
oCa
sec
ont
rol(
ret
ros
pec
tive
)
oCo
rre
lat
ions
tudy
3
9. Def
ineepi
demi
ol
ogy
oThestudyofthedistri
butiona nddeter
mina
ntof
heal
threlat
ede vents(i
ncludingdise
ases)a
ndthe
appl
icat
ionofthisstudytot hec o
ntr
olofdi
sea
ses
andtheothershea l
thproblems .
40. Al adyc
amet oyourcl
inic
,s ai
dthatshedoes
n’t
wantt
odomammo gr
am andpr efer
redtodobr
east
sel
fex
aminat
ion.Whatisyourresponse?
oMa
mmo
gra
m wi
llde
tec
tde
ept
umo
r
oSel
fex
amina
tio
nandma mmo
gra
mar
e
co
mpleme
ntary.(
cor
rec
t)
oSe
lfe
xami
nat
ioni
sbe
stt
ode
tec
tea
rlyt
umo
r.
41
. Cas
eco
ntr
oldes
cri
pti
on
oStar
twitht
heo
utc
omet
henf
oll
owt
her
isk
fa
ctors
42
. Avac
cinat
ionf
orpr
egnantl
adywi
thDT
oGi
veva
cci
nea
ndde
li
ver
ywi
thi
n24hr
s
oCo
ntr
aindi
cat
edi
npr
egna
ncy
oNo
tco
ntr
aindi
cat
edi
npr
egna
nc.(
y c
orr
ect
)
43
. BM13
0;
oObe
se
44. Ifyouseepat
ientandy o
ufac
edif
fic
ultytoget
ac
cur
ateinf
ormat
ionfrom hi
m.Whati
st hebesttodo
?
oAs
kdi
rec
tque
sti
on.(
cor
rec
t)
oAs
kope
nque
sti
on
oCo
ntr
olwa
yofdi
scus
sio
n
45
. Pat i
entcamewit
hmajordepr
essi
onsodur
ing
communi
cati
onwit
hpati
entyouwil
lfi
nd;
oHy
poma
nia
oL
atemo
rni
nga
wake
oL
osso
fey
eco
nta
ct..(
cor
rec
t)
46. Pati
entwanttoqui
tsmoki
ngyout o
ldhimtha
t
s
ympt
omso fni
cot
inewit
hdr
awalpeakedaft
er
o1
-2da
ys
o2
-4da
ys.(
cor
rec
t)
o5
-7da
ys
o8
-10da
ys
47
. Whati stheshapeofadi
str
ibut
iongr
aphs
eeni
na
nor
maldi
stri
buti
oncurve.
oBe
lls
hape
d
48
. Pat i
enttaki
ngbupr
opi
ont
oqui
tsmo
king.Whati
s
thes
ideeff
ect?
oAr
rhy
thmi
a
oSe
izur
e
oXe
ros
tomi
a
oHe
ada
c .(
he c
orr
ect
)
49. Adul
ttogi
vevar
icel
lavac
cine;
o2do
se2we
eksa
par
t
o2do
ses4we
eksa
par
t.(
cor
rec
t)
o2do
ses6mo
nthsa
par
t
o3do
ses4we
eksa
par
t
5
0. Whi leyo
uar ei
nthecl
ini
cy o
ufi
ndthatmany
pat
ient
spresentwi
thredf
o l
li
cul
arc
onj
uncti
vit
is
(
Chl
amy
dia)y
ourmanagementi
s;
oI
mpr
ovewa
ters
uppl
yands
ani
tat
ion.(
cor
rec
t)
oI
mpr
oves
ani
tat
iona
ndde
str
oyi
ngo
ftheve
cto
r
oEr
adi
cat
iono
fther
ese
rvo
ira
ndde
str
oyi
ngt
he
ve
cto
r
oDe
str
oyt
heve
cto
randi
mpr
ovet
hes
ani
tat
ion
5
1. Whi
chi
str
ueabo
utDMi
nKSA.
oMo
stl
yar
eIDDM
oMo
stl
yNI
DDMa
reo
bes
e.(
cor
rec
t)
5
2. Abo
utannuali
nfl
uenz
avac
cinat
ion
oDr
if
t
5
3. T
hebes
tadvi
cet
opat
ientt
ravel
li
ngi
s;
oBo
il
edwa
te.(
r c
orr
ect
)
oI
ce
oWa
ter
oSa
lada
ndunde
rco
oke
dse
ashe
ll
s
5
4. Epi
demi
ccur
ve
oGr
aphinwhic
hthenumbe rofnewc a
se sofa
di
sea
seispl
otte
da gai
nstanint
e r
valofti
met o
de
scri
beaspeci
fi
ce pi
demicoroutbr
ea k.
5
5. Endemi
cmeans
;
5
6. Bes tsentencetodescr
ibespec
ifi
ci
tyo
fsc
reeni
ng
t
esti
sthepopulat
ionofpeopl
ewho;
oAr
enegati
veo fdi
sea
sea
ndt
esti
s
ne
gat
i .
ve(cor
rect)
oAr
epo
sit
iveo
fdi
sea
sea
ndt
esti
sne
gat
ive
oAr
epo
sit
ivec
ompa
ringt
oto
tal
othe
rpe
opl
e
oNe
gat
ivedi
sea
se,
pos
iti
vet
est
oPo
sit
ivedi
sea
se,
nega
tivet
est
.
5
7. T hewaytodet
erminetheac
cur
acyofocc
ultbl
ood
t
estf
or1
1,000o
ldpat
ientsi
sbymeasur
ing;
oSe
nsi
ti
vi
ty.(
cor
rec
t)
oSpe
cif
ici
ty
oPo
sit
ivepr
edi
cti
veva
lue
oNe
gat
ivepr
edi
cti
veva
lue
5
8. T
ruenegat
ivet
esti
sbes
tdes
cri
bedasf
oll
owi
ng;
oNotsuspe
ctedt
oha
vet
hedi
sea
set
hata
ctua
ll
y
do
e sno
tha v
e.
5
9. I ndevel
opi
ngc
ount
ryt
opr
eventdent
alc
ari
es,
it
addt
owater
;
oF
lor
i .(
de c
orr
ect
)
oZi
nc
oCo
ppe
r
oI
odi
de
60. Gar denerhasr
ecurrentco
njunc
tivi
ti
shecan’
t
avoi
dexpo
s ur
etoenvir
onme nt
.Inor
dertodecr
eas
ethe
sympto
msi ntheeveni
ng, GPshoul
dadvisehi
mto
oCo
ldc
ompr
ess
ion
oEy
eir
ri
gat
ionwi
thvi
nega
rso
lut
ion
oCo
nta
ctl
ens
es
oAnt
ihi
sta
mine
s.(
cor
rec
t)
61
. Mo stef
fecti
vemeasur
etoprevents
preadof
inf
ec t
ionamo ngheal
thc
arewor
kersandpatient
sina
nursery;
oWashhandbefor
eanda
fte
rex
ami
ninge
ach
pa
tie
nt.
(co
rrec
t)
oWe
argo
wna
ndgl
ove
sbe
for
eent
eri
ngt
henur
ser
y
oWe
ars
hoec
ove
r
62
. 1 0year
so l
dchil
dbroughtbyhi
sparent
sbecause
theywereco
ncernabouthi
sweight
,heeatsalotof
fastf
oodandFrenc
hf r
ies,
yourmai
nconcernt
o
managet
hispat
ienti
s;
oHi
spa
rent
sco
nce
rna
bouthi
swe
ight
oHi
sBMI
>3.(
3 c
orr
ect
)
oF
ami
lyhi
sto
ryo
fhe
artdi
sea
se
oEa
tingha
bit(
fas
tfo
od,
Fre
nchf
ri
es)
63
. 1 2yearsol
dbo ybroughtbyhispar entf o
rrouti
ne
evaluat
ion,
heiso bes ebutother
wiseheal thy,his
parentwanttome asurehischol
ester
o llevel,whati
s
thebestindic
atoro fmeas ur
ingthi
sc hi l
dc hol
ester
ol?
oHi
spa
rentde
sir
e
oF
ami
lyhi
sto
ryo
fea
rlyCVA
oHi .(
ghBMI c
orr
ect
)
64. Whicho
ft hefol
lo
winginc
reas
esthequali
tyo
fthe
r
ando
miz
edcontro
ll
eds t
udyandmakeitstr
onger
;
oSyst
ema t
ica
ssignmentpr
edi
cta
bil
it
yby
pa
rti
cipa
nts.
(co
rrec
t)
oOpe
nal
l
oca
tio
n
oI
ncl
udi
ngonlyt
hepa
rti
ci
pant
swhor
ece
ive
dthef
ull
i
nte
rvent
ion
oF
oll
owi
nga
tle
ast5
0%o
fthepa
rti
ci
pant
s
oGi
vings
imi
la
rint
erve
nti
ont
osi
mil
argr
oups
65
. Us i
ngthefo
ll
owingc
lassi
fi
cati
on,r
elat
iver
iskof
tho
sewit
hris
kfacto
rtotho
sewithoutri
skfact
oris
oA/
A+B,
C/C+D.(
cor
rec
t)
oA/
A+B
oC/
C+D
66. Co mparingt
heprospec
tiveandr
etr
ospec
tive
s
tudi
es,allaret
rueex
cept;
oRet
ros
pec t
ivea
ret
ypi
cal
lymo
rebi
ase
dtha
n
pr
ospect
ive.
oRet
ros
pec t
ives
tudi
esa
ret
ypi
cal
lyqui
cke
rtha
n
pr
ospect
ive
oPr
o spec
tiveall
ocat
ionofper
sonintogro
up
dependso nwhetherhehasthedis
e a
seor
no
t (
.cor
rect)
oPr
ospe
cti
vec
ostmo
ret
hanr
etr
ospe
cti
ve
oE-e
ffe
cti
smo
rei
dent
ifi
abl
einpr
ospe
cti
ve
67
. Di
agr
am ,
int
erpr
eti
t
oF
ema
lesa
remo
res
usc
ept
ibl
etoo
ste
opo
ros
is
68
. F emaleunderwentabdominaloper
atio
ns hewent
tophysi
ci
anforcheck,ul
tras
oundrevealmetalthi
ng
ins
ideabdomen(missi
ngdur i
ngoperati
on),
whatwill
y
oudo
?
oCa
llt
hes
urge
ona
nda
skhi
m wha
ttodo
.
oCa
lla
tto
rne
yanda
ska
boutl
ega
lac
tin.(
o cor
rec
t)
oT
ellhert
hati
soneo
fpo
ssi
blec
ompl
ic
ati
onso
f
o
pe r
ati
on
oDo
n’tt
ellhe
rwha
tyo
ufo
und.
69. Whenaper so
nispredi
cat
ednott
ohaveadi s
ease
hei
scal
led(
negati
ve)
.Thenwhati
s(t
ruenegat
ive).
oWhenape
rso
nispr
edi
cte
dtoha
veadi
sea
se,
he
ha
sit
oWhenaper
soni
spr
edi
cte
dtoha
veadi
sea
se,
he
do
esnotha
veit
.
oWhenaper
soni
spr
edi
cte
dno
ttoha
veadi
sea
se,
hehasi
t
oWhenape
rsonispre
dic
tednott
oha
veadi
sea
se,
hedo
esnothavei
t..(c
orr
ect
)
oWhe
nri
skc
anno
tbea
sse
sse
d.
7
0. Regardi
ngs
tandar
der
roro
fthemean(
SEM)
,whi
ch
i
str
ue?
oSEMi
sobs
erv
ati
ona
roundt
heme
an
oSt
anda
rdde
via
tio
nisme
asur
eofr
eli
abi
li
tyo
fSEM
oI
sbi
gge
rtha
nSD
oSEMi
sca
lcul
ate
dass
qua
rer
ooto
fva
ria
nc.
e
(
cor
rec
t)
oSt
andar
dde vi
ati
ona
dva
nta
gec
anbema
th
mani
pul
ate
d
7
1. T
hes
tro
nges
tty
peo
fepi
demi
ol
ogi
cals
tudi
esi
s;
oPr
ospe
cti
vec
oho
rts
tudi
es.(
cor
rec
t)
oRe
tro
spe
cti
vec
ont
rolc
ases
tudi
es
oCr
osss
ect
iona
l
oT
ime
li
ne
7
2. Mo therbr
oughther10y
earso l
dobesebo
ytot
he
f
ami
lypr
acti
cecl
ini
c,whatisy
o uradvi
ce;
oSa
medi
eta
ryha
bit
sonl
yex
erc
ise
oF
atf
reedi
et.(
cor
rec
t)
oMul
ti
fac
tor
iali
nte
rve
nti
ons
7
3. F emal epatientdevelopeds uddenlossofvi si
on

bo t
hey es ’
whi l
es hewaswal kingdo wnthes treet,
als
o
complainingo fnumbnes sandt i
ngli
nginherf eet,
thereisdiscrepanc ybetweent hecomplaintandt he
fi
nding, onexami nati
onreflex
esandankl ejerks
preserved,thereisdec r
easeint hesensatio
nand
weaknes
si nthelowermus
clesno
tgo
ingwi
thanat
omy
,
whatisyouract
ion?
oCa
llo
pht
hal
mol
ogi
st
oCa
llne
uro
logi
st
oCa
llps
ychi
atr
ist
oReas
sureherandaskhe
rabo
utt
he
st
resso
rs(
.cor
rec
t)
7
4. Sames cenar
ioo
fthepr
evi
ousques
tio
n,whati
s
y
ourdi
agno
sis
oCo
nve
rsi
ondi
sor
der.(
cor
rec
t)
oSo
mat
ofo
rm di
sor
der
7
5. F or
cingt
hec hi
ldtogotothetoi
letbef
orebedt
ime
andi
nthemo r
ning,yo
uwillco
ntro
ltheprobl
em of;
oEnur
esi
s
7
6. Pat ientwithhear
tdi
seas
eco
mpl
aino
flo
werl
imb
i
schemi
a,youradvi
ce;
oRe
fert
oca
rdi
ol
ogy
oRe
fert
ova
scul
ars
urge
ry
oSt
arthe
par
in.(
cor
rec
t)
7
7. Pat ientwithsever
eheadacheanddec
reas
ein
vi
sualac
uity,pupi
lisdi
lat
ed,sotr
eatment
?
oPi
lo
car
pindr
opa
ndo
pht
hal
mol
ogyr
efe
rrd.
e (
cor
rec
t)
oEr
got
ami
ne
oNSI
D
7
8. Heavysmo kercamet oyouaskingaboutot
her
c
anc
er,
notl
ungc ancer
,thats
mo kingincr
easei
tsri
sk;
oCo
lon
oBl
adde
r.(
cor
rec
t)
oL
ive
r
7
9. Maj
orai
mofPHCi
nSaudiAr
abi
a;
oTopr
ovidec
ompr
ehe
nsi
vema
ter
nal
andc
hil
d
heal
th
8
0. Apat i
enthastender
,re
dnes
snodul
eonlac
rimal
ducts
ite.Bef
oreref
err
edhimtoopht
hal
molo
gist,
what
youwi
lldo;
oT
ropi
cals
ter
oid
oT
opi
cala
nti
bio
tic
s
oOr
ala
nti
bio
tic
s.(
cor
rec
t)
oNo
thi
ng.
8
1. 1 7
y ear
sol
d,shemi
ss edhers
econddoseof
vari
cel
lavacci
ne,
thefi
rstoneabout1y
earago
,what
youwilldo?
oGi
vehe
rdo
ubl
edo
seva
cci
ne
oGi
vehe
rthes
eco
nddo
seo
nl.(
y c
orr
ect
)
oRe
vac
cina
tef
roms
tar
t
oSe
eifs
heha
sant
ibo
dya
nda
cta
cco
rdi
ngl
y
8
2. T her
eisoutbreako
fdi
phther
iaandt
etanusi
n
c
ommunit
y,r
egardingt
opr
egnantwoman;
oCo
ntr
aindi
cat
iont
ogi
veDTva
cci
ne
oI
fex
pos
ed,
ter
mina
tepr
egna
ncyi
mme
dia
tel
y
oI
fex
pos
ed,
ter
mina
tea
fte
r72
hrs
oGi
veDTva
cci
nea
nywa
y..(
cor
rec
t)
8
3. Mo t
herwhoi
sbreastf
eedi
ngands
hewantt
otake
MMRvacci
ne,
whati
sy o
uradvi
ce
oCa
nbegi
vens
afe
lydur
ingl
act
ati
on.(
cor
rec
t)
oConta
inli
veba
cte
riat
hatwi
llbet
rans
mit
tedt
o
thebody
oSt
opbre
ast
fee
dingf
or7
2hr
saf
tert
aki
ngt
he
va
cci
ne.
8
4. Chi l
dwithpo
sit
iveski
ntesto
fTBandpr
evi
ousl
yit
wasnegati
ve,
whati
st het
reat
mentoft
hisc
hil
d?
oI
NHa
lone
oI
NH+r
if
ampi
ci
n
oI
NH+r
if
ampi
ci
n+s
tre
pto
myc
in
oNot
rea
tme
nt
oF
ullr
egi
menf
o B.(
rT cor
rec
t)
8
5. Mal epati
entknownc aseofDM2comewith
HBA1C;
8%,heistaki
ngmet for
minandgli
benc
lami
de
tor
egul
atetheblo
o dsugarneed;
oI
nsul
in.(
cor
rec
t)
oMe
tfo
rmi
nanda
car
bos
e
8
6. Epi demiol
ogicalst
udyforsmo kersai
dther
ei s
1
0000personi
nt hearea,
atstartofthestudytherei
s
2
000smo ke
r,attheendo ft
hes tudy,t
hereis1
000
s
moker,t
heincidenceofthi
sstudyis
0%.(
o1 cor
rec
t)
o1
2.5
%
o2
0%
o3
0%
8
7. Pat ientpres
enttoyou,whenyous eehi
scase,yo
u
di
sco
vert hatpati
enthaster
minalstageofchr
onic
i
ll
ness,
ho wt omanagethi
spatient
;
oMa
kehi
m got
otheho
me
8
8. F emalepat i
entknowntoyo usince3yearsagohas
I
BS,shedidn’tagreewi
thyouabo utthat
,y oudoallthe
i
nvesti
gatio
nno things
uggest
iveo thert
hant hat,she
want
sy outor ef
erher,
atthi
sc ase,whaty ouwil
ldo
oYouwi
llr
espo
ndtoherandr
efe
rhe
rtot
hedo
cto
r
tha
tshewants.(
cor
rec
t)
oYouwi
llr
espo
ndt
ohe
randr
efe
rtot
hedo
cto
rtha
t
yo
uwant
8
9. Pat i
entwit
hdiabet
esandhyper
tens
ion,
whi
chone
o
fant
ihy
pert
ensi
vemedicat
iony
ouwanttoaddf
ir
st?
oACE.(
cor
rec
t)
oBe
tabl
ocke
r
oCa
lci
um c
hanne
lbl
ocke
r
oAl
phabl
ocke
r
90. Theni
fpat
ients
til
lhy
per
tens
ive,
whati
sthenex
t
c
hoi
ce?
oBe
tabl
ocke
r.(
cor
rec
t)
oT
hia
zide
oARB
oCa
lci
um c
hanne
lbl
ocke
r
91
. Yo
ungmanwi
thpl
eur
isybes
tmanagement
;
oNSAI
DS
oAc
eta
mino n.(
phe cor
rec
t)
oCo
rti
sone
92
. Pat i
enthadpainintheback,neck,abdo
menand
upperli
mb.Yo ugavethepati
entaf o
llowupint
he
cli
nic
,buts t
il
lthepat
ienti
sc o
mpl ai
ningandco
ncer
n
ofthepain.Whatisyourdi
agnosi
s?
oChr
oni
cpa
ins
yndr
ome
oSo
mat
iza
tio
ndi
sor
de.(
r c
orr
ect
)
93
. Yo ungmanc o
mewi thheadache,heisdes
cri
bing
thatt
hisheadacheist
hewo rstheadacheinhi
sli
fe,
whatofthefol
lo
wingwillbelesshelpf
ul?
oAs
kingmo
rede
tai
lsa
boutt
hehe
ada
che
oDoMRI
orCTs
can
o Skul
lxr
ay.(
cor
rec
t)
oL
P
94. Howt
opr
eventas
thmai
nchi
ldvi
aadvi
ce,
mot
her
t
odo
;
oWa
shc
lot
heswi
thho
twa
ter
oPr
eve
ntdus
t.(
cor
rec
t)
oCha
ngebl
anke
t
95
. Bes
tme
tho
dtopr
eventpl
aguei
s;
oHa
ndwa
sh
oKi
llr
ode
nt.(
cor
rec
t)
oAvo
idc
ont
actwi
thpe
opl
e
96. 73yearsol
dpat i
ent,f
armer,c
omingwith
c
ompl
aintsofdr yeye,
heisas mokerf
or20year
sand
s
mokes2packs/day,yo
urreco
mmendat i
on;
oAdvi
sehi
mtoe
xer
cis
e
oSt
ops
mo ng.(
ki cor
rec
t)
oWe
ars
uns
cre
en
97
. Out br
eakando nepati
entco
metodot
uber
cul
in
tes
t,andi
tsnegat
ive,whattodo?
oBCG
oI
soni
zed.(
cor
rec
t)
98
. Sec
ondar
ypr
event
ioni
nbr
eas
tcanc
er?
oNOa
nswe
rwa
swr
it
ten.
99. Sec
ondar
ypr
event
ioni
sbes
tef
fec
tivei
n
oDM.(
cor
rec
t)
oL
euke
mia
oPr
e-e
cla
mps
ia
oMa
labs
orpt
ion
1
00. Sec
ondar
ypr
event
ioni
sleas
tli
kel
yofbenef
iti
n;
oBr
eas
tca
nce
r
oL
euke
mi.(
a c
orr
ect
)
oDM
oT
oxe
miao
fpr
egna
ncy
1
01. Anex
ampl
eofs
eco
ndar
ypr
event
ioni
s
oDe
tec
tio
nofa
sympt
oma
ticdi
abe
ticpa
tie
nt.
(
cor
rec
t)
oCo
rona
ryby
pas
sgr
aft
oMe
asl
esva
cci
nat
ion
oRube
ll
ava
cci
nat
ion
1
02. Al
lar
epr
imar
ypr
event
iono
fanemi
aex
cept
oHe
alt
heduc
ati
ona
boutf
oodr
ichi
nir
on
oI
ro
nfo
rti
fi
edf
oodi
nchi
ldho
od
oL
imi
tat
iono
fco
wmi
lkbe
for
e12
mont
hso
fage
oGe
net
ics
cre
eni
ngf
orhe
redi
tar
yane
mi.(
a c
orr
ect
)
oI
ro
n,fo
li
caci
dsuppl
eme
nti
npr
egna
ncya
nd
po
stnat
al
1
03. Whati
sthedef
ini
ti
ono
fepi
demi
calc
urve
oGraphi
cre
gis
tra
tio
nofdi
sea
set
hro
ughape
rio
dof
ti
me
1
04. Whati sthenameofques
tio
nnai
ret
hat
dif
ferent
iat
esbetweenpr
imar
yandseco
ndar
ysl
eep
apnea?
oPo
lys
omno
gra
phy
1
05. Per
inat
almo
rtal
it
y;

oI
ncl
udesa
lls
til
lbi
rt
haf
tert 0thwe
he2 eko
f
pr
egna
ncy
oI
ncl
ude
sal
lne
ona
tal
dea
thsi
nthef
ir
st8we
eko
f
l
if
e
oI
ncl
udesal
lsti
ll
bir
tha
ndf
ir
stwe
ekne
ona
tal
dea
th.
(co
rrec
t)
oSpe
cif
ica
ll
yne
ona
talde
aths
oI
sus
ual
l
yde
athpe
r10000l
ivebi
rt
hs
1
06. Yo uaskedtomanageanHIVpati
entwhowas
i
nvolvedinacaracci
dent
,youknowthatthispat
ienti
s
adrugaddictandhasext
ramari
talr
elat
ions
.Whatar e
yo
ugo ingtodo?
oCo
mplet
eis
ola
tio
noft
hepa
tie
ntwhe
nhei
sint
he
ho
spi
tal
oYouhavetheri
ghtt
olo
oka
fte
rthepa
tie
ntt
o
pr
otec
ty o
urse
lf
oYouwil
lmana
gethi
semer
ge nc
ycasewit
hta
kena
ll
ther
ecommende
dprec
aut
ions.(c
orr
ect
)
oYouwil
lre
por
thi
mtol
ega
laut
hor
it
iesa
fte
r
re
cove
ry
oT
ellhi
sfa
mil
ytha
thi
sisHI
Vpo
sit
ive
1
07. St
ronges
tmet
hodt
opr
eventt
hedi
seas
e.
oI
mmuni
zat
ion
oCha
ngehe
alt
hbe
havi
oro
fPPI
s
1
08 . 3 2year
so ldladyworkasaf il
eclerkdevel
oped
suddeno nseto flo
wbac kpainwhens hewasbending
onf i
lesmo der atel
yseverefor3daysdurati
on.Ther
eis
noevi denceo fnerverootcompressi
on.Whatist he
properac ti
on
oBe
dre
stf
or7t
o10da
ys
oT
rac
tio
n
oNa
rco
tica
nal
ges
ia
oEa
rlya
cti
vit
ywi
thr
etur
ntowo
rk.(
cor
rec
t)
oCTs
canf
orl
umbr
osa
cra
lve
rte
bra
e
1
09. Yo
uhaver
ecei
vedt
heCTs
cano
na3
4year
sol
d
mo thero ft
hreewhohadamal ignantmel ano ma
remo ved3y earsago.Originally, itwasac ler
k‘ sl
evel
andt hepr o
gno si
swasex c el
lent .Thepat ientc amet o
youro ff
iceoneweekagoc ompl ainingo fches tpainand
abdo minalpain,ACTs cano ft hec hestandabdo men
revealedmet astati
clesionst hro ugho utthelungsa nd
theabdo men.Shei siny ouro fficeandy ouhavet o
deli
vert hebadnewso fthes igni ficantspreado fthe
cancer.Thef i
rststepinbr eakingnewsi sto ;
oDel
ive
rthene
wsal
li
no nebl
owa ndge
tito
verwi
th
asqui
ckl
yasi
shumanl
ypossi
ble
oF
ireawar
ningsho
ttha
tso
meba
dne
wsi
s
c
oming.
(co
rrec
t)
oF
indouthowmuc
hthepa
tie
ntkno
wsi
fho
wmuc
h
c
hoos
ei t
.
oF
indo
utho
wmuc
hthepa
tie
ntwa
ntst
okno
wit
oT
ellt
hepa
tie
ntno
ttowo
rry
1
10. Regardi
ngs
moki
ngc
ess
ati
on,
thef
oll
owi
ngar
e
t
rueex
cept
oThemoste
ffe
cti
veme
tho
dofs
moki
ngc
ont
roli
s
heal
the
ducat
ion
oT
hereisst
rongevi
denc
ethatacupunctur
eis
e
ffe
c t
ivei
ns mo
kingcess
ati
on.(corr
ect)
oAnt
ismo
kinga
dvi
cei
mpr
ove
ssmo
kingc
ess
ati
on
oNic
oti
nere
plac
eme
ntt
her
apyc
aus
es40-
50%o
f
smoker
stoqui
t
oT
herel
apser
atei
shi
ghwi
thi
nthef
ir
stwe
eko
f
a
bst
inenc
e.
1
11. I
nci
denc
eisc
alc
ulat
edbyt
henumbero
f
oOl
dca
sesdur
ingt
hes
tudype
rio
d
oNe
wca
sesdur
ingt
hes
tudype
rid.(
o cor
rec
t)
oNe
wca
sesa
tapo
inti
nti
me
oOl
dca
sea
tapo
inti
nti
me
oEx
ist
ingc
ase
satas
tudype
rio
d
1
12. Co
mmuni
cabl
edi
seas
eco
ntr
oll
edby
oCo
ntr
olt
hes
our
ceo
finf
ect
ion
oBl
ockt
hec
aus
alo
ftr
ans
mis
sio
n
oPr
ote
ctt
hes
usc
ept
ibl
epa
tie
nt
oAl
loft
hea
bov
e.(
cor
rec
t)
oNo
neo
fthea
bove
1
13. T r
eat
mentofc
ont
act
sisappl
iedi
nal
loft
he
f
oll
owi
ngexc
ept
oBi
lha
rzi
asi
s
oMa
lar
ia.(
cor
rec
t)
oHo
okwo
rm
oF
il
ari
asi
s
1
14. I
nis
chemi
chear
tdi
seas
e
oPreval
enc
eist
henumbe
rofc
ase
sdi
sco
ver
ed
ye
ar l
y
oI
nci
denc
eisne
wca
sesy
ear
ly
oT
he r
eisass
ociat
ionbetwe
enhyper
tens
iona
nd
i
schemichear
tdiseas
e.(cor
rec
t)
oSmoki
ngi
sana
bso
lut
eca
useo
fis
che
miche
art
di
seas
e
1
15 . Prospec
tiveVSr
etr
ospec
tives
tudi
esal
lar
etr
ue
EXCEPT
oRet
ros
pec t
ivestudi
esha
vemo
rebi
ast
han
pr
ospect
ivestudies
oI
nprospecti
vestudi
es,
thos
ewhoe ntert
hegr
oup
dependwhe t
herthe
yhavethedi
seas
eo r
no
t .
(co
rrec
t)
oPr
ospe
cti
ves
tudi
esa
ree
xpe
nsi
ve
1
16. Mal epat
ientc
omplai
no fex
cruc
iat
ingheadache,
awakenhi
mf r
oms l
eepever
ynightwit
hburning
s
ensat
ionbehi
ndlef
teye,
lacr
imati
onandnas
al
c
onges
tion.Whati
seff
ecti
veintr
eati
nghi
m;
oEr
gona
vine
oSuma
tri
pta
nSC
oMe
thl
ypr
edni
sol
one
oNSAI
D
oOx
yge
n.

You might also like